Sei sulla pagina 1di 122

Introduction to Renal Dysfunction 

 
Review of Renal Basics 
Major Anatomical Features   Kidney:   Glomeruli 
of Urinary Tract     Tubules 
 Interstitium 
 Vasculature 
 Calyces 
 Collecting System   Ureters 
   Bladder 
 Urethra 
 
 Calyces 
Major Functions of the   Glomerular filtration 
Kidney   Excretion 
o Metabolic by‐products, drugs, toxins 
 Electrolyte and acid‐base homeostasis 
 Blood pressure regulation 
 Volume homeostasis 
 Endocrine regulation 
o Erythropoietin, vitamin D, renin 
How do we know a patient   History  
has a  “kidney problem”   Physical Exam 
 Laboratory Evaluation 
o Serum electrolytes 
o Serum BUN, creatinine 
o Urinalysis 
 Imaging 
o Ultrasound 
o CT scan 
 Kidney Biopsy 
Ways to categorize kidney   
dysfunction  Site of lesion   Glomeruli 
 Tubules 
 Interstitium 
 Vasculature 
Nature of Factors   Immunologic 
 Metabolic 
 Infiltrative 
 Infectious 
 Hemodynamic 
 Genetic 
Chronicity   Acute 
 
 Chronic 
Kidney Dysfunction ‐   Acute    Acute Kidney Injury (AKI) 
Chronicity    o Abrupt decrease in kidney function 
 Chronic    Chronic Kidney Disease (CKD) 
  o Evidence of kidney dysfunction/pathology >3 months 
 End Stage   End Stage Renal Disease (ESRD) 
  o Last stage of CKD 
 
 Need for chronic renal replacement therapy 
Stages of Chronic Kidney   Chronic kidney disease is defined as either kidney damage or GFR < 60 mL/min/1.73 m2 for >3 months. 
Disease   Kidney damage is defined as pathologic abnormalities or markers of damage, including abnormalities in blood or urine tests or 
imaging studies 
Stage  Description  GFR (mL/min/1.73 m2) 
1  Kidney damage with normal or increased GFR  >90 
2  Kidney damage with mild decrease GFR 60‐89
3  Moderate decrease GFR 30‐59
4  Severe decrease GFR  15‐29 
 
5  Kidney failure  <15 (or dialysis) 
Approach to the Patient   Renal disease 
with Renal Disease   Determine chronicity 
 Assess level and rate of change of renal function 
 Categorize disorder into specific renal syndromes 
o Prerenal 
o Intrinsic renal 
 Acute tubular necrosis 
 Glomerular 
 Chronic glomerulonephritis 
 Nephrotic syndrome 
 Acute glomerulonephritis 
 Rapidly progressive glomerulonephritis 
 Asymptomatic hematuria and/or proteinuria 
 Vascular 
 Tubulointerstitial 
o Postrenal 
Features of Renal   Too much or too little urine production 
Dysfunction   Azotemia, uremia 
 Proteinuria, hematuria, stone formation 
 Abnormal urinary sediment 
 Electrolyte and/or acid‐base imbalance 
 Hypertension, abnormal volume state 
 Anemia, metabolic bone disease 
 Fever, eosinophilia, rash, pain 
 Toxicity from decreased renal clearance of drugs, endogenous substances (e.g. insulin) 
Too Much or Too Little   Normal Urine Output   
Urine  o 1500ml/24 hour 
 Oligura 
o <500cc/24 hours 
 Anuria 
o Absence of urine output 
 Polyuria 
o >3000ml/24 hours 
Azotemia and Uremia   Azotemia: elevation in renal indices 
o BUN, technically 
o Usually reflects decrease in GFR 
 Uremia: when azotemia gives rise to clinical manifestations and biochemical abnormalities 
o Clinical Syndrome 
 Fatigue, anorexia, nausea, mental status changes, itching 
 Serositis (pericarditis, pleural effusions) 
 Platelet dysfunction 
Proteinuria   Normal Urinary Protein 
o Total protein: < 150mg/24 hours 
o Albumin: < 30mg/24 hours 
 Nephrotic Range Proteinuria 
o “Nephrosis”: > 3 – 3.5gm/24 hours 
 Quantitation of Proteinuria 
o 24hr urinary collection 
o Random urine protein/creatinine ratio 
Hematuria   Gross vs. Microscopic (≥ 2 RBC/hpf) 
 Glomerular vs. Nonglomerular 
 Upper vs. Lower Urinary Tract 
 Imposters 
o Free Hemoglobin 
o Myoglobin 
o Menstrual contamination 
Nephrolithiasis   Formation of stones in the collecting system 
 Manifests clinically as renal colic and hematuria 
Abnormal Urinary   Hematuria 
Sediment   Dysmorphic RBC’s  
 Pyuria 
 Casts 
o RBC casts 
 Glomerulonephritis 
o WBC casts 
 Pyelonephritis 
o Tubular casts 
Types of Renal Tubular   Epithelial casts (muddy brown casts) 
Casts  o Acute tubular necrosis 
 Fatty casts 
o Lipiduria, usually seen in nephrotic syndrome 
 Granular casts 
o Chronic kidney disease 
o Nonspecific finding otherwise 
 Hyaline casts 
o Do not imply over “pathology” 
o Dehydration, exercise, diuretic therapy 
 Waxy casts 
o Advanced kidney disease 
 
Electrolyte/Acid‐Base   Common imbalances in renal disease 
Imbalance  o Hyper/hyponatremia 
o Hyper/hypochloremia 
o Hyperkalemia 
o Hyperphosphatemia 
o Hypocalcemia 
o Anion gap metabolic acidosis 
o Non anion gap metabolic acidosis 
Hypertension, abnormal   Causes of Hypertension 
volume state  o Primary or “Essential” hypertension 
o Secondary causes 
o Anatomic/vascular causes 
 Endocrinopathies 
 Renal diseases, volume overloaded states 
 Pregnancy related diseases 
 Medications 
Anemia, metabolic bone   Renal Hormone Regulation 
disease  o Erythropoietin: regulates HGB/HCT concentration 
o Vitamin D: regulates calcium absorption, maintains normal levels of calcium and phosphorus 
o Advanced kidney disease: 
 Chronic erythropoietin deficiency 
 Anemia of chronic disease, typically normocytic 
 Iron transport/storage dysregulation also contributes 
 Chronic hyperphosphatemia, decreased renal activation of vitamin D3 (1,25‐OH form) 
 Hyperparathyroidism, renal osteodystrophy 
Fever, eosinophilia, rash,   Nonspecific symptoms/signs 
pain   Seen in many forms of renal dysfunction 
o Urinary tract infection 
o Nephrolithiasis 
o Renal infarction 
o Papillary necrosis 
o Acute interstitial nephritis 
o Renal cell carcinoma 
o Vasculitis (SLE, Wegener’s, e.g.) 
o Cholesterol atheroembolic disease 
Toxicity from decreased   Drug Toxicity from Renal Dysfunction 
renal clearance of drugs,  o Often unanticipated by clinicians when managing AKI/CKD 
endogenous substances  o May be presenting feature of AKI/CKD 
(e.g. insulin)  o Can happen with endogenous substances as well 
 Insulin: prolonged renal clearance, resultant hypoglycemia 
o Management 
 Drug dosage adjustment 
Glomerular Disease   Diseases of the glomerulus can present as one of five clinical syndromes:  
o Acute glomerulonephritis 
o Rapidly progressive glomerulonephritis 
o Chronic glomerulonephritis 
o Nephrotic syndrome 
o Asymptomatic urinary abnormalities 
Glomerulonephritis   A number of disorders lead to glomerular injury that presents with some combination of 
o Hematuria 
o Proteinuria 
o reduced GFR 
o hypertension.  
What is “Nephritic   Diseases that cause active inflammation of the glomerulus associated with  
Syndrome”?  o Hematuria (red cells and red cell casts) 
o Azotemia 
o Hypertension 
o Oliguria 
o Sub‐nephrotic range proteinuria 
What is “Nephrotic   Diseases that cause massive leakage of protein across the glomerular basement membranes 
Syndrome”?   Criteria 
o >3.5gm protein/24 hours 
o Hypoalbuminemia 
o Edema 
o Hyperlipidemia 
o Fat bodies in the urine/fatty casts 
Acute Tubular Necrosis   Acute tubular injury 
o Damage to tubular epithelial cells 
Prerenal   Decrease effective arterial flow 
Postrenal   Urine flow is obstructed 
o Ureters, bladder, urethra 
 
Renal Equations   How to Measure/Estimate GFR 
o Creatine Clearance from 24 hour urine collection 
 GFR = [U(Cr) * V] / P(Cr) 
o Cockcroft‐Gault Formula 
 GFR = [(140‐Age) * Weight(kg) * 0.85(if Female)] / [72 * Serum Creatine (mg/dL)] 
o Modification of Diet in Renal Disease (MDRD) Equation 
 eGFR = 186 * (Serum Creatinine)^(‐1.154) * (Age)^(‐0.203) * 1.212(if Black) * 0.742(if Female) 
 Amount of Sodium or Urea Excreated 
o Fractional Excretion of Sodium (FeNa) 
 FeNa = [UNa/PNa] / [UCr/PCr] * 100% 
o Fractional Excretion of Urea (FeUrea) 
 FeUrea = [UUrea/PUrea] / [UCr/PCr] * 100% 
o NOTE: can help in determining cause of acute kidney injury 
 FeNa < 1% or >2% is helpful, but between 1‐2% is indeterminate 
 FeUrea <35% or >35% is helpful 
o Diseases associated with Fractional Excretion of Sodium 
 Prerenal azotemia 
 Reduction in the amount of glomerular filtrate entering each nephron increases the retention of salt and water, resulting in 
a lower fractional excretion of sodium (FeNa) 
 Acute Tubular Necrosis (ATN) 
 Nephrons excrete a large fraction of their filtered sodium and water, resulting in a higher FeNa 
Urinalysis   Urinalysis (Chemstrip Analysis) 
o Color:     Yellow 
o Clarity:    Clear 
o Glucose:    Negative 
o Bilirubin:    Negative 
o Ketones:    Negative 
o Specific Gravity:    1.003 – 1.030 
o Blood:     Negative 
o pH:      5.0 – 8.0 
o Protein:    Negative 
o Urobilinogen:    0.2 – 1.0 
o Nitrite:    Negative 
o Leukocyte Esterase:  Negative 
 Urinalysis (Microscopic Analysis) 
o RBC:      0 – 2 /hpf 
o WBC:      0 – 5 /hpf 
o Casts:     Negative 
o Crystals:    Negative 
 
Histology for Pathology 
Urinary Histology 1 
Name origins:   Kidney in Latin means “Renes” and in Greek means “Nephros” 
 
Identify elements of the   Structure of the Kidney 
gross and microscopic  o The kidneys are located in the retroperitoneum between the 12th Thoracic vertebra to 
structure of the kidney and  the 3rd Lumbar vertebra 
analyze the relationship  o Mass = 115‐170 g (M>F) 
between them   The male kidney is heavier than the female kidney 
   Dimensions = 11‐12 x 5‐7.5 x 2.5‐3 cm 
o Kidneys – paired, bean shaped 
o Ureters – paired 
o urinary bladder 
o Urethra, male versus female 
 Function of the Kidney: 
o Filter blood & reabsorb nutrients 
o Control water, ion, and salt balance of the body 
o Maintain acid‐base balance of the blood (ex. pH) 
o Excrete metabolic wastes (ex. urea and uric acid), toxins, drug components 
o Secrete hormones (ex. renin, erythropoietin) 
o Produce calcitriol 
 Which is the active form of vitamin D and associated with the absorption of dietary calcium into the blood 
 Identify developmental   Kidney development through a series of successive phases:  
stages of kidney and their  o pronephros  most immature 
position  o mesonephros 
  o metanephros  persists as the definitive adult kidney 
 Kidney Embryologic Development  
o 3 stages: pronephros, mesonephros, metanephros 
 cranial to caudal direction followed by “ascend” 
 Pronephros: 
 The pronephros forms early in development, at 22 days post‐coitum (DPC) in 
the cervical region of the embryo. 
 Mesonephros: 
 develops by the formation of mesonephric tubules from the intermediate mesoderm, it is the principal excretory organ 
during early embryonic life (4—8 weeks).  
 It gradually degenerates, although parts of its duct system become associated with the male reproductive organs. 
 Metanephros: 
 arises caudal to the mesonephros at five weeks of development; it is the permanent and functional kidney in higher 
vertebrates.  
 It is derived from the intermediate mesoderm.  
 The ureteric bud arises as a diverticulum from the Wollfian duct, close 
to the entrance to the cloaca and grows towards and inside the 
metanephric mesenchyme. 
 mesonephric duct comes in contact with cloaca, grows cranially as 
ureteric bud 
 ureteric bud & metanephros reciprocally induce growth, forming 
kidney 
 the kidney “ascends”  during this process, the kidney takes new arterial supply from the aorta, new venous drainage 
into the vena cava 
o developmental abnormalities are relatively common 
 important in pediatric nephrology 
 On gross kidney identify:   Kidney structure: bean shaped  
cortex, medulla, papillae  o Capsule – on the outer surface 
at apex of medullary  o Cortex (outer) 
minor calyces, major   Cortex 
calyces, renal pelvis    Renal columns also known as septa of Bertin  which forms cortical tissue on either side of the 
  medullary pyramids 
o Medulla (inner) 
 Medulla divided into several conical pyramids is a papillae 
 The papillae leads to the minor calyces 
 Minor calyces then lead into 2‐3 major calyces 
 Major calyces lead to the expanded upper end of the ureter, renal pelvis (latin: pelvis=basin) 
o Hilum 
 pelvis, ureter, renal artery, vein 
 
 
 
 
 
Describe the renal blood   Kidney blood supply:  
supply: renal artery, arcuate  o Kidneys are small organs   0.5% of total body weight but receives 25% of the cardiac output 
arteries, interlobular  o Blood Flow 
arteries, differences   Start with the renal artery which then divides into the anterior and posterior divisions 
between cortical and   The anterior or posterior division then lead to the segmental arteries  
medullary blood supply,   The segmental arteries then lead to the interlobar arteries. 
define arterioles (afferent   Where the interlobar arteries then extend into either side of medullary pyramid 
and efferent), peritubular   At the border of the cortex and the medulla, the interlobar arteries curve to form Arcuate 
capillary plexus  arteries. 
   Arcuate artery then enters the renal cortex to from the Interlobular artery. 
 The Interlobular artery forms branches called Arteriole, which are at a right angle to the 
Interlobular artery 
 The Afferent Arteriole enters the Bowman’s Capsule at the vascular pole to form the Glomerular 
tuft and then exits as Efferent Arteriole. 
 Following the exit from the glomeruli, the Efferent arterioles in the cortex form peritubular 
capillary plexus 
 In contrast, efferent arterioles that are exiting glomeruli that are near the medulla called 
juxtamedullary, take a deep dive into the medulla to form the straight vessels called Vasa Recta 
 The Vasa Recta supply blood to the tubules and specifically to the Loop of Henle 
 Correlation with pathology: 
o The is more blood delivered to the cortex than to the medulla 
 Cortex = 90% of blood supply 
 Medulla = 10% of blood supply 
 medulla is relatively a‐vascular and is VERY susceptible to changes in the supply of blood  low oxygenation 
o tubular capillary beds are derived from the efferent arterioles 
 which are susceptible to acute tubular necrosis (injury) or papillary necrosis 
 Arterioles: 
o Afferent arterioles arise from interlobular vessels and supply 
the glomeruli 
o Efferent arterioles arise from glomerular capillaries 
 Glomerulus: 
o Glomerulus in paraffin section: Bowman’s capsule (Bc) + 
glomerular tuft* 
 The round structure is Bowman’s capsule (green circle) 
 Inside Bowman’s capsule is the glomerular tuft 
 Outside you can see the tubules (yellow circle) 
 
Urinary Histology 2 
   The kidney: glomerulus 
o Glomeruli are rounded structures labeled as G 
o Tubules  
o are located in between the glomeruli and are small round or oval structures labeled as T 

 Higher magnification: 
o Glomeruli labeled as G  
o Tubules can be seen in the cross section (TC) or in the longitudinal section (TL) 
o Vessel (V) shows a medium sized artery 
o Interstitum (I) is the area in between the tubules and in the normal kidney the interstitum 
is inconspicuous (not clearly visible) 

 Zooming into the glomerulous 
o Where the glomerular tuft (black outline) is surrounded by Bowman's Capsule 

 Glomerulus – Overview    Afferent arterioles arise from interlobular vessels 
  o Then enters the bowman's capsule forming the glomerular 
tuft 
 Efferent arterioles arise from glomerular capillaries 

 
 
 
Glomerulus – Structure   The entire glomerular tuft is supported by mesangial cells lying 
  between the capillaries.  
 Basement membrane–like “mesangial matrix” forms a meshwork in 
which the “mesangial cells” are embedded.  
o Mesangial cells have three functions:  
 Contractile: 
 To regulate blood flow and filtration 
 Similar to vascular smooth muscle cells 
 Provides structural support to the vessels  
 Capable of proliferation   generating both matrix and collagen  
 Phagocytic: 
o Endothelium: 
 There is a thin layer of fenestrated endothelial cells 
 Where the fenestra is not covered by a membrane 
o Basement membrane: 
 Separates the glomerular compartments 
 Endo‐capillary: containing mesangial and endothelium cells 
 Extra‐capillary: located in the urinary space and contains two lays of epithelial cells 
 visceral (aka “podocytes” or foot processes) – anchored on glomerular basement membrane 
 parietal – line Bowman’s capsule 
Pathology of Glomeruli   To understand the pathology of the glomeruli and the prototypic diseases, we have to go beyond 
the detail of light microscopic H&E slides to electron microscopy 
o Electron microscopy are normally black and white 
o Capillaries (labeled C) 
o In between the capillaries is the Mesangial area with Mesangial cells (labeled MC) 
o Red Box: Shows the foot processes 
 Area of the red box zoomed 
o Electron microscopy of the glomerular capillary wall (aka glomerular filtration 
barrier): 
 fenestrated endothelium without diaphragms 
 glomerular basement membrane –  
 lamina rara interna – outter layer next to the Endothelium 
 lamina densa – middle layer 
 lamina rara externa – outer layer next to the foot processes 
 visceral epithelial cells (podocytes) with foot processes 
 in between each foot process called the filtration slits with thin slit diaphragm 
 Note: other capillaries in the kidney are fenestrated with diaphragms 
Glomerular filtration   Glomerular basement membrane: = combined basal lamina of glomerular 
barrier: Glomerular  endothelium & podocytes 
basement membrane  o Lamina rara externa & Lamina rara interna = heparan sulphate (‐) 
o Lamina densa: collagen type IV, laminin (size) 
 Characterized as: 
o High permeability to water & small solutes 
o Impermeability to proteins 
 Filtration barrier Factors 
o size barrier: 
 <70,000 MW and <10 nm in diameter proteins pass easily 
o charge dependent restriction: 
 where the Lamina rara externa & Lamina rara interna are negatively charged 
 Excluding negatively charged molecules such as albumin 
 Glomerular function:  
o Provides the first stage in the filtering process of the blood carried out by the 
nephron in its formation of urine 
 Pathology: 
o Proteinuria – loss of protein 
 Which could be due to loss of charge or injury to the slit membrane 
o Hematuria – bleeding through the glomerular filtration barrier 
 Which could be due to a loss of integrity or “structural” damage to the GBM 

Immunofluorescence:    many kidney diseases are immune‐complex or antibody/abnormal protein deposition mediated 
   Immunofluorescence used to detect such deposits 
o IgG, IgA, IgM 
o Kappa and lambda light chains (polyclonal versus light chain restriction) 
o C3, C1q – complement involved in immune complex formation 
o Fibrinogen – necrosis, coagulation 
o Albumin – filtration injury with protein loss 
 Frozen sections: better sensitivity and specificity than paraffin sections IHC 
 
 
 
 
Nephron:   Nephron: functional & structural unit 
  o Function = formation of urine 
o Structure: 
 glomerulus 
 proximal tubule 
 loop of Henle 
 distal tubule 
 collecting duct system 
 Glomeruli produce 125 ml of filtrate/minute 
o Where 124 ml is reabsorbed by the tubules 
o Resulting in approximately 1.5 L of urine/day 
 Histology: 
o Cortex contains:  
 Glomerulus, proximal, distal tubules 
o Medulla contains:  
 Loop of Henle and collecting duct 
   Cortex Image 
o Proximal Tubules contain pink eosinophilic cytoplasm with a narrow lumen  
o Distal Tubules contain cuboidal epithelium with a wider lumen 

 Medulla Image 
o Showing tubules in cross sections associated with the Loop of Henle and Collecting ducts 
o No glomeruli 
o No proximal tubules 
o No distal tubules 

o Zoomed in Image 

Proximal convoluted tubule   prominent brush border (*) which is facing the tubular lumen 
(PCT)  o increases the luminal surface area of the cells facilitating absorption 

The proximity of the distal   Macula densa of the distal tubule: 
tubule and the glomerulus   o Is a dense accumulation of cells along the border of the distal tubule 
  that is adjacent to the cells within afferent arteriole 
 The cells within this region of the afferent arteriole are called 
Juxtaglomerular cells 
 Which are modified smooth muscle cells 

Juxtaglomerular apparatus:    specialized structure formed by the distal convoluted tubule and the glomerular afferent arteriole 
   located near the vascular pole of the glomerulus 
 main function: 
o to regulate blood pressure and the filtration rate of the glomerulus 
 where regulation of the glomerular filtration rate depends on the tubular luminal concentration of 
sodium and chloride associated with the “tubuloglomerular feedback” 

 
 
 
Urinary Histology 3 
Ureter   Main function of the ureter is to provide a conduit to drain 
  urine at low pressures from the kidney to the bladder 
o Urine flows down to the bladder when a person is 
standing, sitting, and laying down 

 Histology cross section of the ureter 
o Labels:  
 L‐lumen 
 E‐epithelium also called urothelium 
 M‐muscularis 
 Surrounded by adipose tissue 
 
 Urinary Bladder 
o Inside of the bladder are ridges called rugae 
 At the base of the bladder:  
 There is an area that is smooth and triangular in shape 
called the Trigone 
 At the base of the Trigone: There are orifices for the 
ureters to allow urine to enter the bladder  
 
 At the tip of the Trigone: There is the opening of the 
Urethra passing through the urogenital diaphragm 
 Cross section of the urinary bladder wall  
o From the Top 
 Lumen 
 Transitional layer of epithelium 
 Lamina propria – loose connective tissue 
 Muscular layer – Detrusor muscle 
 Adventitia with fat cells 
 
 Serosa or peritoneum layers is covering the upper 
portion and sides of the urinary bladder 
 Histology Section 
o From the Top 
 Lumen 
 Lamina propria boundary is the line the arrow is pointing 
to 

Urothelium   The urothelium (aka transitional epithelium):  
  o Which covers the: 
 renal pelvis, ureters, bladder, parts of urethra 
o composed of 3‐5 cell layers, which can contract and expand   
 The term transitional epithelium does not imply that this 
epithelium is in actual transition from one type to another, 
but rather refers to the appearance of the cells changes as the 
organs with which they are associated become distended 
versus not distended: 
o cuboidal when bladder is not distended or relaxed 
o flat when bladder is distended 
 Umbrella cells cover the top of the urothelium and become very stretched out when the bladder is distended 
The urothelium (aka   Accommodate the fluctuation in volume 
transitional epithelium):   protect against the caustic effects of urine – by providing a powerful barrier to 
  urine  
 Umbrella cells: 
o apical exocytosis of specialized fusiform vesicles (aka discoid vesicles) 
during distension of the bladder provides additional fragments (reserve) of 
cell membrane, which are incorporated into the cell membranes, allowing 
them to stretch in a full bladder 
o the apical plasma membrane of umbrella cells, facing the urine, is covered with rigid‐looking plaques, which, together with 
tight junctions, form a specialized membrane compartment that represents one of the tightest and most impermeable barriers 
in the body 
 Urothelial carcinoma: bladder, pelvis, ureter, urethra 
Female versus Male Urethra   Male urethra is connected with the reproductive system 
   Both female and male urethra are composed of epithelium 
o Transitional epithelium near bladder 
o Squamous epithelium near external orifice 
o Mucus urethral glands 
 Pathology: 
o Urine cytology is a test to look for abnormal cells in your urine.  
 is used along with other tests and procedures to diagnose urinary tract cancers. 
 
Glossary  capsule  a structure enclosing an organ, usually composed of dense 
connective tissue 
cortex  the outer portion of an organ, distinguished from its inner, 
medullary portion 
septum/septa  a wall, dividing a cavity or structure into smaller ones 
calyx; plural noun: calyces  a cuplike cavity or structure 
minor  lesser, small 
major  greater 
hilus or hilum  a depression or pit at that part of an organ where the vessels 
and nerves enter 
medulla  the inner portion of an organ, usually in the center 
Afferent, from afferre  [Latin] = to bring toward 
Efferent, from efferre  [Latin] to bring out
juxta  [Latin] close to
 
vasa recta  [Latin] straight vessels 
 
Renal Pathology: Introduction to Glomerular Diseases 
 
Kidney Diseases 
 
Valves   Kidney Diseases –Bird’s Eye View: 
 Each year in the US >100,000 people are diagnosed with end stage renal disease. 
 CDC estimates that >10% of adultsin the US (>20 million people) may have chronic kidney disease of varying levels and 
seriousness 
 Causes of kidney failure: 
o Prerenal 
o Intrarenal 
o Postrenal 
 Glomerular disease fit into a much bigger category of intra renal diseases and are actually much less common than diabetes and 
hypertension. However, their evaluation via biopsy plays a major role in the selection of therapies. 
New cases of kidney failure   Glomerular disease fit into a much bigger category of intra renal diseases and are actually much less 
by primary cause  common than diabetes and hypertension   Other, 
19.70%

 However, their evaluation by biopsy plays a major role in the selection of therapies 
Diabetes, 
GN, 7.90% 44%

HTN, 
28.40%

 
Glomerular Histology 
 
Review   Glomerular tuft = network of capillaries 
o endothelium 
o mesangium: cells & matrix 
o basement membrane 
o visceral epithelium (aka “podocytes”) 
 Bowman’s capsule: basement membrane + parietal epithelium 
 Cellular function/response:  
o phagocytosis, control of blood flow/intracapillary pressure: mesangial 
o proliferation: mesangial, endothelial, parietal epithelial; NOT podocytes  
 Glomerular filtration barrier:  
o loss of structural integrity (hematuria) 
o loss of selective filtering (proteinuria) 
Light microscopy: H&E stain 


Electron microscopy: black 
& white 


 
Pathogenesis of glomerular diseases: immune complex versus other 
 
Pathogenesis of glomerular   Two major categories: 
diseases  o Immune mechanism‐mediated glomerular injury 
 Circulating immune complexes deposition 
 In situ binding of antibodies with immune complex or without immune complex formations 
 Anti‐glomerular basement membrane antibody 
 Antibody against antigen on podocytes 
 Abnormal activation of complement 
o Other mechanisms 
 No immune complexes and no antibodies detectable by current methods 
 Nephron loss 
 Genetics 
 Monogenic diseases 
 Polygenic diseases – “risk alleles” 
To study glomerular   Immuno stains: type of antibody, etc Frozen section immunofluorescence 
diseases caused by immune   
mechanisms 

 Electron microscopy: precise localization, etc 
 

Immunofluorescence   Granular: Mesangium + basement membrane, basement 
patterns  membrane alone 
 

 Linear: along basement membrane 
 

 
 
Circulating immune‐  Complexes may be formed with:  
complex‐mediated diseases:  o endogeneous antigens (systemic lupus erythematosus) 
o exogeneous antigens (post‐infectious glomerulonephritis) 
o unknown antigens 
Pathology of circulating   Phase I: formation of antigen‐antibody complexes 
immune complexes   
 

 Phase II: deposition of circulating immune complexes in the 
glomerulus initiates complement and Fc receptor mediated 
leukocyte activation 
 

 
 Phase III: inflammatory reaction & tissue injury at the site of 
deposition antibody has no specificity to glomerular 
components !!! 
 Complement activation & recruitment of leukocytes = 
major pathway of antibody‐initiated glomerular injury 
 
 
 
Formation of immune   The outcome of immune complex formation depends on several factors including factors impacting the  
complexes does NOT  o pathophysiology of immune complexes (size) 
ALWAYS lead to disease  o duration of antigen exposure 
  o host response  
o localization of immune complexes in the glomerulus 
Localization of immune   Size: 
complexes in the  o large in subendothelial, small in subepithelial  
glomerulus:  o glomerular hemodynamics,  
  o mesangial phagocytic function 
 

 
 Molecular charge: 
o highly cationic in subepithelial 
o highly anionic in subendothelial 
o neutral charge in mesangium  glomerular filtration 
barrier has negative charge 

Glomerular filtration   The basement membrane can be divided into two filtration zones: proximal zone and 
barrier: proximal versus  the distal zone 
distal zones   The proximal zone is exposed to the contents in blood including:  
o Antigen 
o Antibody 
o Complement 
o Neutrophil 
o Macrophages 
o Where the proximal zone is surveyed by immune process but the distal zone is not 
   Immune Complex Deposition in the Sub‐endothelial space:  
o Complexes deposited in the proximal zone of the glomerular basement membrane (sub‐
/endothelial space): 
o elicit inflammatory reaction and proliferation in the glomerulus 
o Causing infiltration of leukocytes and structural injury of the filtration barrier with hematuria 

   The complexing of the antigen and antibody in the distal zone (sub‐/epithelial space) of the 
glomerular basement membrane: 
o are non‐inflammatory  
o Because the inflammatory cells that are circulating in the blood do not see the complexing of 
the antigen 
o affecting the podocytes (epithelial cells) 
o Causing the foot processes to become aphased with alteration of the filtration barrier (slite 
membrane) 
o resulting in proteinuria 
Immune complexing in   the complexing of the antigen and antibody in the sub‐epithelial space is unique because the binding occurs on the urinary side 
distal zone of the  of the glomerular basement membrane 
glomerular filtration:   the subsequent activation of complement and cytokine factors is modified (reduced) because the site of the deposit is remote 
from the activators that are normally present in the circulation (“non‐inflammatory”) 
 affect podocytes (epithelial cells) with alteration of the filtration barrier resulting in proteinuria 
 
Circulating and in‐situ immune complex mediated disease: postinfectious and membranous glomerulonephritis 
 
Circulating Immune   Serum sickness model 
complexes ‐ experimental   Antigen‐antibody complexes form in the circulation: disease 
occurs when complexes are formed with antigen in slight 
excess (complexes escape phagocytosis and deposit in 
tissues/surface of blood vessels) 
 

 
 Deposited antibody (IgG) can be seen in the kidney biopsy 
(green fluorescent granules) 
 

 
 
Circulating Immune   Example of human disease = postinfectious glomerulonephritis: 
complexes ‐ human disease  o infection is followed by development of antibodies 
o immune complexes are formed in the circulation 
o immune complexes are deposited in the proximal zone of the glomerular capillary wall 
o inflammatory reaction with leukocytic infiltration, mesangial & endothelial proliferation and structural damage (“Swiss 
cheese”) at the site of immune complex deposition antibody has no specificity to glomerular components 
 Glomerulus is affected by circulating immune complexes 
where the tuft is hypercellular with proliferation of 
epithelial and mesangial cells 
o there is also presence of tri‐lobed nuclei which are PMN 
(polymorphonuclear leukocytes) 
 
 
 Glomerulus with antibody (IgG) seen as granular fluorescent 
deposits on immuno stain 
 

 
 
Clinical syndrome:   hallmark of the NEPHRITIC SYNDROME and also mild 
hematuria  proteinuria, edema, renal failure, hypertension (HTN). 
 What can be the outcome of the inflammatory response?   
 Degradation of immune complexes by neutrophils, 
monocytes/macrophages and mesangial cells leads to 
healing phase with complete resolution in most patients, in 
particular in children 
In‐situ immune complex   Reaction of antibody with an antigen on basal surface of epithelial cells with in‐situ formation of immune complexes (seen as 
formation:  electron dense deposits – red star) under epithelial cells (sub‐epithelial) leads to loss of slit diaphragms and effacement 
(“fusion”) of the epithelial cell foot processes resulting in increased permeability with PROTEINURIA 
 Electron microscopy – normal 
 

 
 Electron microscopy ‐ disease: subepithelial electron dense 
deposits 
 

 
 Immunoglobulin IgG (antibody) deposits are seen as small 
granules along glomerular capillary wall (white arrow) on 
immuno stain. Stain for complement is also positive 
 

 
 
What is the clinical picture?   Sub‐epithelial immune complexes = distal zone: 
o no inflammatory response, no “Swiss cheese” injury 
o effacement of foot processes and loss of slit diaphragms leads to increased permeability 
 “gauze‐like” or fine mesh effect → PROTEINURIA 
 since larger particles are retained – NO hematuria 
 

 
 Clinical: proteinuria (hallmark of NEPHROTIC syndrome: 
heavy proteinuria, hypoalbuminemia, edema, ….) 
 Human disease: membranous nephropathy 
 

 
 
Membranous nephropathy   RATS injected with antigen (proximal tubular brush border) antibodies develop against proximal tubular 
– experimental model  brush border which CROSS‐REACT with basal surface of epithelial cells leading to formation of sub‐
(Heymann nephritis)  epithelial immune complex deposits  
 Q: what happens in humans? 

Pathogenesis of   several families with neonatal nephrotic syndrome and membranous nephropathy  
membranous nephropathy:   mothers had mutations in neutral endopeptidase (NEP) normal podocyte antigen 
from rats to humans   women who genetically lack NEP develop antibodies during pregnancy when 
exposed to NEP (blue dots) expressed by placental cells and by fetal cells entering 
the mother’s blood 
 From about the 18th week of gestation, maternal antibodies of the IgG class are actively transported across the placenta to the 
fetus, where they bind (in‐situ) to the NEP antigen expressed on podocytes 
 These observations validated the in situ paradigm in human membranous nephropathy subsequent proteomic studies identified 
phospholipase A2 receptor (PLA2R) and thrombospondin type‐1 domain containing 7A (THSD7A) as target antigens in 80% of 
patients with primary membranous nephropathy the remaining 20% still? 
Humans: membranous   autoimmune process with antibodies reacting with intrinsic renal antigens or planted antigens: nucleosomal complexes 
nephropathy  (systemic lupus erythematosus) viral, bacterial products, drugs… 
 The podocyte is at the center of the pathogenesis of membranous nephropathy either by  
o providing a source of endogenous antigens or by  
o creating an environment favorable to deposition and accumulation of immune complexes containing exogenous (non‐
podocyte) antigens  
 The podocyte is also a victim of complement activation and antibody activity, and hence there is a subsequent podocyte 
effacement with proteinuria 
 As our understanding of membranous nephropathy evolves, it is apparent that this nephropathy does not fit well into the 
classification scheme of hypersensitivity diseases i.e. it shows some overlap between type II and type III 
 Different mechanisms of glomerular injury are not mutually exclusive and in humans  
o where >1 mechanism may contribute to injury 
post‐infectious   infection elicits antibody response (immunoglobulin G, IgG)  
glomerulonephritis   immune complexes form in circulation (antigen + IgG + complement) 
 deposition of immune complexes in the capillary wall elicits inflammatory reaction leading to structural damage (“Swiss cheese”) 
with hematuria and proliferation  
 however, immune complexes are also formed in‐situ 
leading to formation of big sub‐epithelial deposits 
“humps” (white arrow) which are unique to postinfectious 
glomerulonephritis and therefore diagnostically useful 
o The humps contain SpeB (streptococcal exotoxin B) and 
streptococcal glyceraldehyde‐3‐phosphate 
dehydrogenase (GAPDH), which reaches the sub‐
epithelial aspect of the glomerular basement 
membrane owing to its cationic charge – here the antigen is not intrinsic but “planted antigen” 
   What have we learned from this? 
o experimental models are not perfect but offer some 
insight  
o in disease >1 mechanism is likely to be responsible.. 
o various classifications have a limited “fit” 
 In fact, it is not clear if in postinfectious glomerulonephritis 
immune complexes are formed mainly in the circulation or 
in situ by binding of antibodies to bacterial antigens 
“planted” in the glomerular basement membrane 
 
Anti‐ glomerular basement membrane‐mediated glomerulonephritis, crescent 
 
Antibody‐mediated   antibodies bound to tissue antigens activate the 
glomerular injury (type II  complement by the “classical” pathway 
hypersensitivity)   products of complement activation recruit neutrophils 
and monocytes triggering inflammation in tissues 
 leukocytes may also be activated by engagement of Fc 
receptors, which recognize bound antibodies  

antibodies against antigens   IgG (antibody) seen as a linear stain along the entire length of the glomerular basement 
within glomerular  membrane 
basement membrane 

   antibodies bind diffusely along the glomerular basement membrane: 
o linear stain for IgG (antibody) = damage along the entire length 
o severe damage to the GBM with multiple areas of necrosis ‐ “sieve‐like” effect with 
big holes big leaking large number of RBCs with GROSS hematuria 
 Human disease: anti‐glomerular basement membrane antibody disease 

How to stop hematuria ?   Glomerular crescent = “glomerular stopper”  
GLOMERULAR CRESCENT  o stops bleeding in cases with severe damage to glomerular capillary 
wall (of various etiologies), but in this process also compresses the 
glomerular tuft, reduces filtration and leads to → rapidly progressing 
renal failure 
 
Anti‐glomerular basement   Experimental evidence – nephrotoxic serum (Masugi) nephrits in rats: inject anti‐rat 
membrane antibody‐ kidney antibodies (prepared in rabbits) linear IgG deposition  
induced glomerulonephritis   Human antigen = noncollagenous domain  (NC1) of the α3 chain of collagen type IV 
(normally encrypted and does not elicit antibody response) 
 Cross‐reactivity with pulmonary alveolar basement membrane = Goodpasture 
syndrome (gross hematuria + pulmonary hemorrhage) 

 
Abnormal activation of complement‐mediated glomerulonephritis 
 
Glomerular diseases caused   Unregulated/excessive activation of the alternative complement pathway leading to complement‐mediated injury – 
by complement activation  transformation from low‐grade physiologic activity (“tick‐over”) to unrestrained hyperactivity 
in the absence of antibody:   Triggers: excessive complement activation after minor vascular injuries 
  o acquired autoantibodies against complement components 
o inherited abnormalities of complement regulatory proteins 
 Human diseases  
o Glomerular:  
 dense deposit disease/C3 glomerulonephritis 
o Systemic (with significant renal manifestations):  
 thrombotic microangiopathies 
Complement system   Components (numbered C1‐C9) present in plasma in inactive forms; each activated by proteolysis to acquire own proteolytic 
  activity, thus setting up enzymatic cascade.   
 3 initiating pathways: 
o Classical 
 trigger: Ab+Ag 
 adaptive immunity 
o Alternative, begins @ C3 
 constitutively active 
 innate immunity 
o MBL (mannose‐binding Lectin)  
 trigger: lectin to mannose of bacteria innate immunity 
 both classical & lectin pathways begin with engagement of early complement components C1/C2, MASP (Mannose‐binding 
lectin‐Associated Serine Protease), very similar to C1 molecules of the classical complement pathway 
Alternative Complement   C3 convertase activity must be 
pathway  tightly controlled 
o in order to prevent excessive 
activation of complement 
 Alternative pathway 
o low‐grade physiologic activity 
(“tick‐over”) 
 C3NeF(C3 nephritic factor) 
o an autoantibody against C3 
convertase, binds to C3 convertase & prevents its degradation (stabilizes it) causing sustained complement activation 
 H factor 
o mutations in gene encoding factor H 
o autoantibodies to factor H 
o factor H deficiency 
 
Other mechanisms: podocyte injury – minimal change and focal and segmental  glomerular sclerosis 
 
Other mechanisms of   NOT able to detect immune complexes/antibodies by current techniques (immunstains, electron microscopy) 
glomerular injury:    less well known 
 
Podocyte injury non   circulating “permeability factor” not as yet identified – recurrence in transplants 
immune complex/antibody   viruses 
mediated:   drugs 
“podocytopathies”   adaptation to elevated glomerular capillary pressures & flow rates (glomerular hypertension) 
   genetic defects 
 experimental models of podocyte injury: toxins (puromycin) 
Pathology of podocyte   podocyte foot process effacement and loss of slit diaphragms can be reversible 
injury   or not reversible  
 Irreversible podocyte injury leads to podocyte detachment and loss 
 mature podocytes have limited capacity to replicate, hence podocyte depletion 
leads to scarring (sclerosis) 
 loss of slit diaphragms/foot process effacement is most highly associated with 
proteinuria 
 clinically: heavy proteinuria with NEPHROTIC SYNDROME 
 reversible nephrotic syndrome: minimal change disease 
 irreversible nephrotic syndrome: FSGS [Focal and Segmental Glomerular Sclerosis] 
Podocyte injury as seen by   NO immune complexes that we can see 
electron microscopy   NO inflammatory response 
 Reversible disease = minimal change disease 
 Irreversible disease = focal and segmental glomerular sclerosis (FSGS) 
 BOTH diseases begin with podocyte effacement with loss of slit diaphragms 
 Normal: podocyte foot processes and slit diaphragms 
preserved 
 

 
 Podocyte foot processes effacement with loss of slit 
diaphragms & PROTEINURIA 
 

 
 
   Minimal change disease and FSGS: one disease at opposite ends of a spectrum OR two 
different diseases?  

 Nephron loss   Once renal disease, glomerular or otherwise, destroys sufficient nephrons to reduce the glomerular filtration rate to 30‐50% of 
  normal, progression to end stage renal disease proceeds at varying rates via scarring, called glomerulosclerosis 
 Adaptive changes in response to the loss of nephrons at this stage are ultimately maladaptive and exacerbate progressive 
sclerosis 
 
Genetics and glomerular diseases:  Alport syndrome and  congenital nephrotic syndrome, “risk alleles” 
 
Genetic defects: monogenic   germline mutations in genes encoding:  
  o slit diaphragm proteins with nephrotic syndrome 
o type IV collagen with hematuria (Alport syndrome) 
 NPHS1 encoding nephrin, NPHS2 encoding podocin rare 
hereditary forms of the nephrotic syndrome 
 

 
 Normal 
 

 
 Alport syndrome – hematuria 
 

Genetic variants in the   APOL1 risk variants have large effects on several different types of kidney disease 
APOL1 gene account for a  previously thought to be distinct entities, often previously labelled as 
large fraction of the high  “hypertensive nephropathy in African Americans”  
rates of nondiabetic kidney   These variants, found only in individuals with recent African ancestry, (<10,000 
disease in African  years) confer enhanced innate immunity against African trypanosomes. These 
Americans    alleles are nearly absent in populations of European and Asian ancestry 
 APOL1 risk variants arose approximately 4,000 years ago in Africa and rose quickly 
to high frequency.  In Nigeria, approximately 46% of chromosomes contain either 
the G1 or G2 allele. The ancestors of modern Europeans left Africa many millennia before the origin of these risk alleles, so the 
risk alleles are not found in Europeans. Today, approximately 36% of all African Americans carry the G1 or G2 alleles 
 
 APOL1 Nephropathy   People who have at least 1 copy of either the G1 or G2 APOL1 variant (allele) are resistant to infection by trypanosomes 
  (protozoa), but people who have 2 copies of either variant are at an increased risk of developing a non‐diabetic kidney disease 
 Sickle cell trait confers protection against malaria caused by Plasmodium 
falciparum (a protozoan) 
 The presence of the alleles is not enough to have the phenotype  
o These are risk alleles rather than a single‐gene disorders and additional “hits” 
are necessary, which may be genetic, environmental, or both 
o Development of preventive measures for those at risk 

   Lessons learned: 
o genetic differences substantially influence an individual’s lifetime risk for kidney disease  
o evolution of genes related to host defense against pathogens may limit kidney longevity  
o expanding our understanding of renal development and function 
o the design of novel therapeutics for kidney disease as well as preventive measures for those at risk 
 The variants have proven to be useful for genetic screening in African Americans and in the selection of kidney donors 
IgA nephropathy   geographic and racial differences in IgA nephropathy prevalence have long been recognized until 
recently it was still debated to what degree these were due to differences in disease diagnosis 
(e.g., due to diverse local biopsy practices) rather than biology  
 it is now clear that a substantial portion of disease risk is conferred genetically.  Recent series of 
genome‐wide association studies [GWAS] have identified several susceptibility loci   
 the genetic loci identified thus far comprise genes associated with innate and adaptive immunity, 
and the complement system 
 the complement locus involve genes encoding proteins which regulate the alternative 
complement pathway  
 IMAGE: World‐wide genetic risk for immunoglobulin A nephropathy.  Genome‐wide association studies indicate different 
worldwide risks for IgA nephropathy  
 
Summary and Vocabulary 
 
NEPHRITIC SYNDROME ‐   circulating immune complexes – postinfectious glomerulonephritis (serum sickness model) 
HEMATURIA    anti‐glomerular basement membrane antibody mediated ‐ anti‐glomerular basement membrane disease (nephrotoxic serum 
  [Masugi] nephritis) 
NEPHROTIC SYNDROME –   in‐situ immune complex formation – membranous glomerulonephritis (Heymann nephritis model) 
PROTEINURIA   podocyte injury non‐immune complex mediated 
 reversible = minimal change disease 
 irreversible = focal and segmental glomerular sclerosis (FSGS) 
Genetics   monogenic diseases 
 polygenic and “risk alleles” 
   Different pathways of glomerular injury are not mutually exclusive and in humans more than one may contribute to injury 
 Host factors, which are usually also not static and include genetic diversity, are critical to determine who does and who does not 
develop nephritis 
 Thus, the disease is a dynamic process, more akin to a movie rather than a snap‐shot 
Vocabulary   Glomerular diseases usually have the “glomerulo” prefix 
o see postinfectious glomerulonephritis 
 Glomerulonephritis is used preferentially in reference to glomerular diseases with an inflammatory/proliferative response  
 Glomerular pathologies, without an inflammatory response may be referred to as “nephropathy”or “glomerulopathy” 
o see membranous nephropathy (glomerulopathy) 
 Nephrosis is meant to indicate a non‐inflammatory nephropathy, which is associated with nephrotic syndrome 
 “nephritis” can also be attached/used in connection with other kidney diseases, such as “pyelonephritis” 
 nephros [Greek] = kidney, nephrologist = MD specializing in medical kidney diseases 
 ren [Latin] = kidney, renal pathology 
 urologist takes care of “surgical ”kidney diseases such as tumors, reflux, etc. 
 ūrīna [Latin] 
 Sclerosis: 
o Glomerular sclerosis: increased collagenous extracellular matrix that is expanding the mesangium, and subsequently 
obliterating the capillary lumen, or forming adhesions with the Bowman’s capsule 
 Vascular sclerosis: 
o Hyaline arteriolosclerosis: hyaline (proteinaceous) deposits with thickening of the wall and narrowing of the lumen of small 
arteries, i.e. “arterioles” 
 Hyaline from Greek: crystal, glass. A hyaline substance appears glassy and pink in H&E stain  
 Arteriosclerosis: “hardening of the arteries”, wall thickening and loss of elasticity 
 Nephrosclerosis: “hardening” of the kidney due to vascular disease 
 H&E stain (hematoxylin & eosin stain) = routine pathology stain cytoplasm is pink (staining with eosin) and nuclei are dark blue 
(staining with hematoxylin) 
 
 
 
Renal Physiology Review 
 
Measurement of Renal   There are 4 types of structures in the kidney 
Function  o Glomeruli 
o Tubules 
o Interstitium 
o Blood vessels 
 Renal function usually means glomerular function (glomerular filtration) or tubular function (urinary excretion) 
Glomerular filtration   The glomeruli filter the plasma, allowing passage of solutes according to size and charge.  
 Normally, only small amounts of albumin and other larger proteins (globulins) are filtered, and 
then most of filtered proteins are reabsorbed and catabolized by renal tubules. 
 Therefore, large amounts of albumin in the urine suggest glomerular disease. 

Calculating glomerular   Net filtration pressure = Hydrostatic Pressure (blood pressure) – Capillary osmotic 
filtration pressure  pressure – Hydrostatic pressure (Bowman’s capsule) 
o Hydrostatic Pressure (Glomerular capillary) = PGC 
o Net Filtration Pressure = PGC – PI – Pfluid 
o                                          = 55 – 30 – 15 = 10 mmHg 

Glomerular dynamics   GFR = Kf [(PGC – Pfluid) – PI] 
o Kf = ultrafiltration coefficient 
o PGC = Hydrostatic Pressure (blood pressure) 
o Pfluid = Hydrostatic pressure (Bowman’s capsule) 
o PI = Capillary osmotic pressure 
o In the absence of urinary obstruction, Pfluid is fairly constant.   
 where an increase in PGC or decrease in PI will increase GFR 
 Or a decrease in PGC or increase in PI will decrease GFR 
Renal plasma flow   Renal plasma flow (RPF) can be estimated using para‐
aminohippuric acid (PAH) clearance since nearly 100% of 
PAH entering the renal circulation is excreted 
o RPF = CPAH (Clearance of PAH) 
 Conversion of RPF to Renal Blood Flow (RBF) 
o RBF = RPF/(1‐Hct) 
 where Hct = Hematocrit 

Filtration fraction   Filtration fraction (FF) = GFR/RPF 
 Normally  
o GFR is ~ 120 mL/min 
o RPF is ~ 600mL/min 
o therefore FF ~ 120/600 = 20% 
 A decrease in RPF will stimulate the intrarenal renin‐angiotensin (RAS) system 
o resulting in constriction of the efferent arteriole (EE) 
 increasing back pressure on the glomerular capillary and maintaining PGC, thus preserving GFR 
 FF is thus increased. 
 An increase in RPF will increase PGC and thus increase GFR  FF remains unchanged 
Effects of Drugs  At the Afferent arteriole: 
 Normally Prostaglandins will preferentially dilate the afferent arteriole 
o Which increase in RPF 
 Causes an increase in GFR 
 So that FF remains constant 
 NSAIDS will inhibit Prostaglandins 
o Cause a decrease in GFR 
 Dihydropyridines (Calcium Channel Blocker) 
o dilate the afferent arteriole 
 Causing an increase in GFR 
At the Efferent arteriole: 
 Normally Angiotensin II will preferentially constricts the efferent arteriole 
o Causing a decrease in RFP 
 Resulting in an increase in GFR 
 So that FF increases 
 RAAS Inhibitor (ACE Inhibitors or Angiotensin receptor Inhibitor) will inhibit Angiotensin II 
o Causing a decrease in GFR 
 Glomerular   Filtration fraction (FF) = GFR/RPF 
Hemodynamics    
 

 Systemic hypertension would increase glomerular capillary 
pressure in absence of autoregulation 
o Filtration fraction (FF) = GFR/RPF 
o In order to keep the FF constant, GFR would have to increase 
o High blood pressure will increase the flowrate in the afferent 
which will cause the GFR to increase 
 
 BUT Renal autoregulation prevents transmission of systemic 
blood pressure to glomerular capillaries 
o When the blood pressure goes up, this leads to a 
autoregulation affect to prevent an increase in the renal 
plasma flow 
o Autoregulation prevents the transmission of the high blood 
pressure to limit the injury that could occur to the glomerular 
 
capillary 
 Intrarenal RAS 
o In the afferent arteriole, the juxtaglomerular apparatus makes 
Renin due to decreased blood flow 
o Renin converts angiotensinogen to Angiotensin 1 
o Angiotensin 1 is converted to Angiotensin 2 by ACE in the lungs 
o Angiotensin 2 will then constrict the efferent arteriol 
 
 Effect of lowering BP (or blood volume) on renal circulation and 
intrarenal RAS 
o If the blood pressure decreases  causes a decrease in RPF 
and GFR  which will be detected by the juxtaglomerular 
apparatus to secrete Renin  Lead to angiotensin 2 to 
constrict the efferent arteriole   to increase GFR 
 
 Inhibition of RAS dilates efferent arterioles and lowers 
glomerular capillary pressure 
o By blocking ACE  causes a decrease in Angiotensin II  
causes the efferent arteriole to relax (dilate)  leads to a 
decrease in GFR 
 

 Inhibition of RAS normalizes glomerular capillary pressure in the 
presence of hypertension 
o Kidney disease will inhibit the RAS 
o By blocking ACE with a patient who has high blood pressure  
causes a decrease in Angiotensin II  causes the efferent 
arteriole to relax (dilate)  leads to a normal GFR 
o Because High blood pressure causes an increase in GFR 
 
 
 Dihydropyridine calcium antagonists abolish renal 
autoregulation and prevent normalization of glomerular capillary 
pressure 
o Normally when the blood pressure goes up, this leads to a 
autoregulation affect to prevent an increase in the renal 
plasma flow 
o But Dihydropyridine calcium channel blocker will inhibit 
autoregulation and cause the GFR to increase 
 
 
 
   Increasing plasma protein concentration causes an increase in oncotic pressure which is causing fluid to flow back into the 
glomerular capillary 
Effect  GFR  RPF  FF = GFR/RPF 
Afferent arteriole constriction  Decrease  Decrease  No change 
Efferent arteriole constriction  Increase  Decrease  Increase 
Increase plasma protein concentration Decrease No change Decrease
Decrease plasma protein concentration Increase No change Increase
 
Constriction of Ureter  Decrease No change Decrease
Angiotensin 2   Vasoconstriction both afferent and efferent arterioles —> Therefore increases the FF 
o But has a greater effect on the efferent arterioles 
 As a consequence: 
o RFP decreases 
o GFR increases 
o PPC decreases 
o PGC increases 
o FF increases 
o Oncotic capillary pressure increases 
During a Stress Response   Increase in sympathetic input and high levels of Ang 2 
 As a consequence 
o Vasoconstriction both afferent and efferent arterioles 
o Because both constrict, there is a large drop in RPF and only a small drop in GFR 
o Net effect is an increase in FF 
Sympathetic Nervous   Causes vasoconstriction of the afferent and efferent arterioles 
System  o Has a greater effect on afferent arterioles 
 Which causes a decrease in GFR and a decrease in RPF resulting in an increase in FF 
 Causes an increase in oncotic pressure  
 Resulting in an increase in proximal tubule reabsorption of fluid and solutes 
 As a consequence: 
o RPF decreases 
o GFR decreases 
o PPC decreases 
o PGC decreases 
o FF increases 
o Oncotic capillary pressure increases 
 Increase in renin secretion by the JG cells in the afferent arterioles  
 Beta blockers decrease renin secretion by blocking beta‐1 receptors in the kidney 
o Decrease afterload on the heart by decreasing renin secretion 
 Because angiotensin 2 causes vasoconstriction the peripheral arterioles which increases afterload 
Renin Release   Decrease flow in the afferent arterioles will stimulate the release of Renin 
o EX – hemorrhage, dehydration, CHF, renal artery stenosis 
 Increase in sympathetic drive to the JG cells 
 Low luminal NaCl concentration at the macula densa 
Clinical Correlate: ACE   Used for diabetic nephropathy 
Inhibitors and ARBs  o Leads to reduction in glomerular capillary pressure 
o Vasodilator efferent arterioles 
o Decreases GFR 
o Reduces damage and fibrosis of glomeruli —> delays the need for dialysis 
 Treat hyperfiltration pressures caused by hyaline arteriolosclerosis of the efferent arterioles 
 Guidelines for ACE Inhibitors and ARBs 
o Give to patients with nephrotic syndrome and stable chronic renal failure 
o Avoid in patients with severely compromised GFR (risk of hyperkalemia) and with Acute Renal Failure 
o May cause a type 4 renal tubular acidosis due to blocking aldosterone 
 Leading to helper kale is 
o Switch from ACE Inhibitor to ARB in cases with drug‐induced cough or angioedema 
o Both are contraindicated in bilateral renal artery stenosis 
 Which causes a rapid Decreases GFR 
 
 
 
 
 
Measurement of glomerular   Inulin clearance:   
filtration rate (GFR)  o Inulin is a 5000 kD polysaccharide from the Jerusalem artichoke 
o It is freely filtered by the glomeruli and neither reabsorbed nor secreted and thus inulin clearance can be used to measure 
GFR. 
o Although inulin clearance is the “gold standard”, in clinical practice, GFR is generally measured by creatinine clearance 
 Measured creatinine clearance:   
o When a substance is cleared from the blood by glomerular filtration, the excretion rate of the substance (assuming no 
secretion or reabsorption by the tubules) will equal the volume of plasma that is totally cleared of the substance.  Therefore: 
o Plasma concentration x Clearance = Urine concentration x Urinary flow rate, or: 
o Clearance = (Urine concentration x Urinary flow rate)/ Plasma concentration [(UV/P)] 
Creatinine clearance   Creatinine excretion depends on creatinine generation by muscle as a byproduct of creatine metabolism.   
 In the steady state creatinine production = creatinine excretion 
 Creatinine is freely filtered and not reabsorbed by the kidney (there is minimal secretion which can generally be ignored) 
o Clearance = (Urine concentration x Urinary flow rate)/ Plasma concentration [(UV/P)] 
 For example:  
o If a patient excretes 1.0 L of urine in 24 hours (1440 min), and the plasma and urine creatinine concentrations are 1.0 mg/dL 
and 144 mg/dL, respectively 
o the creatinine clearance = (144 mg/dL x 1000 mL/1440 min) / 1.0 mg/dL = 100 mL/min 
Estimation of creatinine   Inverse creatinine 
clearance   Since creatinine clearance (CCr) is inversely proportional to plasma creatinine (Cr), i.e. CCr ~ 1/Cr 
o a rough approximation of renal function can be obtained in this manner. 
 For example 
o if CCr is 100 mL/min when Cr is 1.0 mg/dL, then CCr would be 50 mL/min when Cr is 2 
mg/dL, 25 mL/min when Cr is 4 mg/dL, and 12.5 mg/dL when Cr is 8.   
 Inverse relationship between creatinine clearance and plasma creatinine 

Estimation of creatinine   Cockroft‐Gault equation 
clearance (2)  o Creatinine clearance (mL/min) = [(140‐age) x body wt (kg) / 72 x plasma Cr] x 0.85 (if female) 
 Still used to estimate GFR in special populations (e.g., spinal cord injured patients).  In addition, the FDA still uses this equation 
when evaluating drug dosing. 
Estimation of glomerular   Creatinine‐based formulae:  
filtration rate (eGFR)  o Note:  all of these formulae require a steady‐state (stable) level of plasma creatinine and cannot be used in patients whose 
renal function is rapidly changing 
 MDRD formulae:   
o The most commonly used 4‐variable formula utilize serum creatinine, age, gender, and race. 
 MDRD formula:  eGFR (mL/min/1.73m2) = 186 x SCr‐1.154 x Age‐0.203 x [0.742 if female] x [1.21 if black] 
 CKD‐EPI (Chronic Kidney Disease Epidemiology Collaboration) equation: 
o CKD‐EPI formula:  eGFR (mL/min/1.73m2) =  141 x min(SCr/k,1)a x max(SCr/k,1)‐1.209 x 0.993Age x [1.018 if female] x [1.159 if 
black] 
Chronic Kidney Disease   In CKD 
(CKD)  o GFR will generally decline linearly with time at a rate dependent on the type and severity 
of kidney disease 
o the typical linear decline in GFR will be evident when GFR is plotted vs. time (see following 
Figure) 

Tubular Function   With CKD 
o loss of nephrons  
 leads to loss of tubular as well as glomerular function 
 However, it is possible to have isolated defects in tubular function in the face of normal or near‐normal GFR 
Fractional excretion   Fractional excretion of a substance is the amount of that substance that is excreted into the urine relative to the amount of 
that substance that is filtered by the kidney.   
o It thus depends on both glomerular and tubular function. 
 Fractional excretion = Amount excreted / Amount filtered 
 Note: Since fractional excretion is generally expressed as a percent and not a fraction, it can be defined simply as the percent of 
the filtered substance that is excreted into the urine. 
Fractional excretion of   Filteres load = GFR * Px 
sodium (FENa)   Excretion rate = V * Ux 
 FENa = (Na excreted)/(Na filtered) 
o FENa = (UNa x V)/ (PNa x GFR) 
 Substituting creatinine clearance (UCr x V/PCr) for GFR results in: 
o FENa = (UNa x V) / (PNa x (UCr x V/PCr) ) 
 Simplifying 
o FENa = (PCr * UNa) / (UCr * PNa) x 100% 
 
 
 
 
What is the FENa in healthy   Most healthy people on standard American diets have a FENa of ≈ 1%.  Let us see why. 
people?  o A typical American diet contains about 6 grams of sodium daily.   
o Since 1 mmol of sodium = 23 mg, this is 261 mmol.   
 In the steady state, what is ingested must be excreted (mostly in the urine, with small amounts in stool and sweat).  
 How much sodium is excreted?  
o Let us assume that 250 mmol/day of sodium appears in the urine.   
 How much sodium is filtered?   
o Plasma sodium x GFR, i.e., 140 mmol/L x 180 L/day = 25200 mmol/day. 
 Fractional excretion of sodium (FENa) = (250 mmol/25200 mmol) x 100 ≈ 1% 
Patient 1   A 50‐year‐old man with moderately severe CKD (estimated GFR by creatinine‐based formula of 25 mL/min/1.73m2) is seen in 
clinic.  His physical examination is normal and he has no peripheral edema.    
 What is the expected FENa? 
o Greater than 1% 
 How much sodium is excreted?  
 Still 250 mmol/day (unless the patient has restricted dietary sodium intake) 
 How much sodium is filtered? 
 In this instance, if the GFR is 25% of normal, i.e. 45 L/day rather than 180 L/day, the amount of sodium filtered will be 45 
L/day  x 140 mmol/L = 6300 mmol/day. 
 Fractional excretion of sodium = (250 mmol/63000 mmol) x 100 ≈ 4% 
 Important Points 
o FENa should increase as GFR decreases. 
o A “normal” FENa in someone with severe CKD means that either the patient is not ingesting sodium or there is a stimulus 
(such as decreased renal blood flow) for the kidney to reabsorb sodium. 
Acute Renal Failure   CKD ‐‐ generally a linear decline in GFR with time. 
 Acute renal failure ‐‐ linear increase in plasma creatinine concentration (“delta creatinine”).   
o Do NOT use eGFR to assess renal function.  
 Slowing of “delta creatinine” indicates renal function is improving.   
o Continued improvement in kidney function will lead to fall in plasma creatinine 
concentration (a negative “delta creatinine”).   
 The arrow indicates an acute renal insult (such as ischemia or acute toxic exposure).  
Triangles depict serum creatinine concentration which continues to rise until creatinine 
excretion again equals creatinine generation and then falls as excretion exceeds generation. 
Glomerular and Peritubular   A decrease in renal plasma flow (RPF) stimulates the intrarenal renin‐angiotensin system (RAS), leading to 
Capillaries   constriction of the efferent arteriole.   
o This serves to maintain hydrostatic pressure in glomerular capillaries (PGC) and GFR, increasing filtration 
fraction (GFR/RPF).   
o A resulting decrease in PPC and increase in πPC leads to increased reabsorption of solute and water by the 
tubules.  
 In this diagram PGC is the same as PH in the previous diagram. 
 A decrease in RPF causes the efferent arteriole to constrict to increase GFR 
 But at the same time, the efferent plasma flow has decreased  causes a decrease in the efferent pressure  causes an 
increase in oncotic pressure  results in an increase in the reabsorption of solute and water by the tubules 
 This is why in peritubular reabsorbs salt and water due to renal failure 
Causes of Acute Renal   Acute renal failure (ARF) is now referred to by the acronym “AKI” which can mean “acute kidney injury” or “acute kidney 
Failure  impairment” 
   Differential diagnosis of AKI   Pre‐renal: 
o Pre‐renal (decreased renal  o Decrease in effective circulating volume  Decreases GFR 
blood flow)  o Example: Heart Failure 
o Intra‐renal (kidney injury)   Intra‐renal: 
 Tubular (most common)  o Immune complex deposition in glomerulus  Decrease in GFR 
 Glomerular (acute  o Example: 
glomerulonephritis)   Glomerulonephritis: acute inflammation of the kidney, typically caused by an 
 Interstitial (acute interstitial  immune response 
nephritis)   Proteinuria: the presence of abnormal quantities of protein in the urine, which 
 Vascular (acute  may indicate damage to the kidneys 
vasculopathy/vasculitis) (least   Post‐renal: 
common)  o Obstruction of the ureter (kidney stone) 
o Post‐renal (urinary obstruction)   Increase in Bowman’s Space hydrostatic pressure  Causes a decrease in GFR 
o Example:  
 Hydronephrosis: refers to distension and dilation of the renal pelvis and calyces, 
 
usually caused by obstruction of the free flow of urine from the kidney 
 
 
 
 
 
 
 
 
Urinary indices   FENa 
 FEurea (fractional excretion of urea) 
 Urine specific gravity and osmolality 
 Urinalysis 
Differential Diagnosis of AKI    Pre‐Renal  Renal  Post‐Renal 
Urine Sodium  <20 mmol/L >20 mmol/L Variable
Fractional excretion of  <1% >1% Variable
Sodium (FeNa) 
Fractional excretion of Urea  <35%  >35%  Variable 
(FeUrea) 
Urine specific gravity  >1.015  ~1.010  Variable 
Urine Osmolality  >350 mmol/kg  <350 mmol/kg  Variable 
Urinalysis  Hyaline casts Granular casts; may have  Hyaline casts: may have 
hematuria, proteinuria, and  hematuria, pyuria 
or pyuria depending on 
etiology 
 Pre‐renal = decrease renal blood flow (decrease in RPF) 
o Urine sodium < 20 mmol/L 
 Because there will be an increase in the reabsorption of sodium and water 
o FeUa < 35% 
 Urea is reabsorbed by the tubules 
 In pre‐renal failure, there is an increase in the reabsorption of urea   causes the FeUa to decrease 
o Specific gravity > 1.015 
 Because the tubules are able to concentrate urine which causes the specific gravity to increase 
 Renal = kidney injury 
o Because in tubular injury, the tubules are not functioning properly and not reabsorbing sodium which causes the fraction of 
sodium to increase 
o Specific gravity to be normal 
 because the kidney is not able to concentrate or dilute 
Patient 1   A 22‐year‐old medical student is taking renal physiology, and as part of the course, he submits a plasma sample and a 24‐hour 
  urine for determination of creatinine clearance.  His plasma creatinine is 1.0 mg/dL and his 24‐hour urine contains 1440 mg of 
24 hrs = 1440 min  creatinine in a volume of 1.0 L.   
1 dL = 100 mL   What is his creatinine clearance? 
o His creatinine clearance (CCr) can be determined by the formula: 
o Clearance = (Urine concentration x Urinary flow rate)/ Plasma concentration [(UV/P)] 
o CCr = (1440 mg/L x 1.0 L / 1440 min)  / 1.0 mg/dL   
o        = 1 mg/min / 0.01 mg/mL  
o        = 100 mL/min 
 Ten years later, he is a nephrology attending, and decides to recheck his plasma creatinine. He is dismayed to find that it is now 
2.0 mg/dL. He repeats the test several times over the next several days and gets the same result each time (indicating he has 
CKD).  
 What is his 24‐hour urine creatinine now? 
o Answer = 1440 mg 
o It is the same as it was 10 years earlier, providing he did not lose or gain muscle mass in the interim. Since plasma creatinine 
is stable, he is in a steady state and thus excreting all of the creatinine produced. 
 What is his creatinine clearance now? 
o Answer = 50 mL/min 
o CCr = (1440 mg/L x 1.0 L / 1440 min) / 2.0 mg/dL = 1 mg/min / 0.02 mg/mL = 50 mL/min 
Patient 2   A 65‐year‐old nephrologist undergoes coronary artery bypass surgery (CABG) after which he develops AKI. His plasma 
creatinine was 1.0 mg/dLpre‐operatively. On the first post‐operative day his plasma creatinine is 2.0 mg/dL. On the second post‐
operative day it is 2.8 mg/dL, and on the third post‐operative day it is 3.2 mg/dL. He is non‐oliguric (making normal urine) and 
not fluid overloaded and has no electrolyte abnormalities. 
 GFR = 1/PCr 
o 1.0 mg/dL delta = 0 
o 2.0 mg/dL delta = 1 
o 2.8 mg/dL delta = 0.8 
o 3.2 mg/dL delta = 0.4 
 Since the delta is getting smaller then his kidneys are improving even though his creatinine is increasing 
 Is his renal function worsening or improving on post‐operative day 3? 
o Answer = Improving because the rate is slowing 
 Do you predict that he will need dialysis? 
o Answer = No because his renal function is expected to continue to improve providing there are no additional renal insults. His 
plasma creatinine will probably peak at about 3.5 mg/dL and then begin to decline. 
Tubular Physiology   Proximal convoluted tubule (PCT) ‐‐ bulk solute and water reabsorption  
 Loop of Henle ‐‐ reabsorbs electrolytes (primarily sodium, potassium, and chloride).  
 Distal convoluted tubule (DCT) and collecting duct (CD) ‐‐ fine tune sodium excretion and regulate potassium and 
acid‐base balance.  
 In the absence of antidiuretic hormone (ADH), the thick ascending limb (TAL) of the loop of Henle, the DCT, and 
the CD are impermeable to water, which allows formation of dilute urine. 
 ADH increases water permeability in the CD, allowing urine concentration.  
Proximal convoluted tubule   Contains brush border 
(PCT)   Reabsorbs all glucose and amino acids and most bicarbonate, sodium, chloride, phosphate, 
potassium, water, and uric acid. 
 Isotonic absorption  Generates and secretes ammonia which enables the kidney to secrete more 
protons 
 PTH inhibits sodium/phosphate cotransport 
o Which causes phosphate excretion 
 Angiotensin 2 stimulates sodium/proton exchange 
o Which causes an increase in sodium, water and bicarbonate reabsorption (permitting contraction alkalosis 
 65‐80% of Sodium is reabsorbed 
 Drugs 
o Acetazolamide inhibits the carbonic anhydrase causes an increase of bicarbonate in the lumen  
Loop of Henle – thin  Thin descending loop of Henle 
descending limb (TDL) and   Passively reabsorbs water via medullary hypertonicity  due to impermeable to sodium 
thick ascending limb (TAL)   Concentrating segment  makes urine hypertonic 
Thick ascending loop of Henle 
 Reabsorbs sodium, potassium, and chloride 
 Indirectly induces paracellular reabsorption of magnesium and calcium through positive lumen 
potential generated by potassium back leak 
 Impermeable to water 
 Makes urine less concentrates as it ascends 
 10‐20% of sodium is reabsorbed 
 Drugs 
o Loop diuretics block the sodium‐potassium‐chloride transporter 
Distal convoluted tubule   Reabsorbs sodium, chloride 
(DCT)   Makes urine fully dilute  hypotonic 
 PTH increases the calcium/sodium exchange 
o Which causes calcium reabsorption 
 5‐10% of sodium is reabsorbed 
 Drugs  
o Thiazide diuretics inhibit the sodium‐chloride co‐transporter 
collecting duct (CD)   Collecting tubule contains two types of cells:  
o Principal cells and Intercalated cells 
 In the Principal cells: 
o Chloride is reabsorbed paracellularly 
o Sodium transporter (Epithelial Sodium Channel = ENAC) which can be inhibited by Amiloride and 
Triamterene which are potassium sparing drugs 
o Aldosterone stimulates the recruitment of sodium channels (ENAC) to increase sodium reabsorption 
into the cell 
 But by bringing sodium into the cell, Aldosterone causes potassium to be loss in the urine 
o ADH will activate V2 receptors on the basolateral side of the cells to stimulate the insertion of aquaporins on the urine side of 
the cell to facilitate the reabsorption of water 
 There are two types of intercalated cells: alpha and beta 
o In alpha‐intercalated cells:  
 protons are secreted into the urine to facilitate the reabsorption of bicarbonate into the blood 
o In beta‐intercalated cells:  
 protons are reabsorbed into the blood to facilitate the secretion of bicarbonate into the urine 
Tubular Pathophysiology   PCT dysfunction leads to a characteristic disorder called Fanconi syndrome.  
o Associated with an increase excretion of nearly all amino acids, glucose, bicarbonate, and phosphate. 
 May result in metabolic acidosis  Proximal renal tubular acidosis 
o Causes include: 
 Hereditary defects – Wilson disease, tyrosinemia, glycogen storage disease, cystinosis 
 Ischemia 
 Multiple myeloma 
 Nephrotoxins/drugs – ifosfamide, cisplatin, tenofovir, expired tetracyclines 
 Lead poisoning 
 TAL dysfunction leads to Bartter’s syndrome,  
o Affects sodium/potassium/chloride cotransporter 
o Results in hypokalemia and metabolic alkalosis with hypercalciuria 
o Presents similarly to chronic loop diuretic use (e.g. furosemide (Lasix)). 
o Autosomal recessive 
 DCT dysfunction results in Gitelman’ssyndrome,  
o Reabsorptive defect of NaCl which leads to hypokalemia, hypomagnesemia, metabolic alkalosis, hypocalciuria 
o Similar to using life‐long thiazide diuretics 
o Autosomal recessive 
o Less severe than Bartter syndrome 
 CT dysfunction results in Liddle syndrome 
o Gain of function mutation causes an increase in the sodium reabsorption in the collecting tubules 
o Results in hypertension, hypokalemia, metabolic alkalosis, decreased aldosterone 
o Presents like hyperaldosteronism, but aldosterone is nearly undetectable 
o Autosomal dominant 
o Treatment: Amiloride 
 Syndrome of Apparent Mineralocorticoid Excess  Cortisol tries to be the SAME as Aldosterone 
o Hereditary deficiency of 11‐beta‐hydroxysteroid dehydrogenase, which normally converts cortisol to cortisone in cells 
containing mineralocorticoid receptors. 
o Excess cortisol in these cells from enzyme deficiency 
 Causes and increase in mineralocorticoid receptor activity 
 Resulting in hypertension, hypokalemia, metabolic alkalosis 
o Low serum aldosterone levels  
o Can acquire disorder from glycyrrhetinic acid (present in licorice) which blocks activity of 11‐beta‐hydroxysteroid 
dehydrogenase 
o Treatment: Corticosteroids 
 Exogenous corticosteroids decrease endogenous cortisol production which causes a decrease in mineralocorticoid receptor 
activation 
Solute  Glucose  Phosphate  Bicarbonate  Potassium  Uric acid  Amino Acids  Low MW 
Proteins 
Disorder  Glycosuria  Phosphaturia  Metabolic acidosis (proximal renal  Hypokalemia  Hypouricemia  Aminoaciduria  Tubular 
Hypophospha‐ tubular acidosis)  proteinuria 
 
temia 
RAAS   Liver produces Angiotensinogen 
 Kidneys produce Renin due to stimulation by 
o Decrease in blood pressure (JG cells) 
o Decrease in Sodium delivery (macula densa cells) 
o Increase in sympathetic tone (beta‐1 receptors) 
 Renin converts Angiotensinogen to Angiotensin 1 
 Angiotensin 1 travels to the lungs where ACE converts Angiotensin 1 to Angiotensin 2 
o ACE also breaks down Bradykinin 
 Angiotensin 2 then acts on.. 
o Acts at angiotensin 2 receptors type 1 (AT1) on vascular smooth muscle 
 Causes vasoconstriction 
 Results in increase in blood pressure 
o Constricts efferent arterioles of the glomerulus 
 Causes an increase in filter fraction to preserve renal function (GFR) in low‐volume states (i.e. when RBF is decreased) 
o Stimulates the release of Aldosterone from the adrenal gland 
 Causes an increase in sodium channel and sodium/potassium pump insertion in principal cells 
 Enhances potassium and proton excretion by way of principal cells potassium channels and by alpha‐intercalated cell proton 
ATPases 
 Creates favorable sodium gradient for sodium and water reabsorption 
o Stimulates the release of ADH from the posterior pituitary 
 Increases aquaporin insertion in principal cells 
 Causes water reabsorption 
o Increase PCT sodium/proton activity 
 Causes sodium, bicarbonate, and water reabsorption (can permit contraction alkalosis) 
o Stimulates the hypothalamus 
 Causes the thirst response 
Erythropoietin   Released by interstitial cells in peritubular capillary bed in response to hypoxia 
 Stimulates RBC proliferation in bone marrow 
 Often given as a supplement in chronic kidney disease 
1,25‐(OH)2D3   Within PCT cells, PTH stimulates 1alpha‐hydroxylase to  convert 25‐OHD3 (inactive vitamin D) to 1,25‐(OH)2D3 (active vitamin D) 
Prostaglandins   Paracrine secretion vasodilators (PGI2 and PGE2) the afferent arterioles to increase RBF 
o NSAIDs block renal‐protective prostaglandin synthesis 
 Causes constriction of afferent arteriole and decreases GFR 
 Results in acute renal failure 
Dopamine   Secreted by PCT cells to promote natriuresis (excretion of sodium in the urine) 
 At low doses 
o Dopamine dilates the interlobular arteries, afferent arterioles, and efferent arterioles 
 Which causes an increase in RBF with little or no change in GFR 
 At high doses 
o Acts as a vasoconstrictor 
Site of action of hormones   Angiotensin 2 (AT2) 
acting on kidney  o Synthesized in response to decrease in blood pressure. 
o Causes efferent arteriole constriction  increase in GFR and an increase in FF but with 
compensatory sodium reabsorption in the proximal and distal nephron. 
o Net effect: preservation of renal function (increased FF) in low‐volume states with 
simultaneous sodium reabsorption (both proximal and distal) to maintain circulating 
volume  
 Parathyroid hormone (PTH) 
o Secreted in response to… 
 decrease in plasma calcium concentration 
 increase in plasma phosphate concentration 
 decrease in plasma 1,25‐(OH)2D3 
o Causes an increase calcium concentration reabsorption in the DCT, a decrease in phosphate reabsorption in the PCT, and an 
increase in 1,25‐(OH)2D3 production 
 The increase in 1,25‐(OH)2D3 production causes an increase in calcium and phosphate absorption from the gut 
 
 Atrial Natriuretic Peptide (ANP) 
o Secreted in response to an increase in atrial pressure 
 Causes an increase in GFR and an increase in sodium filtration with NO COMPENSATORY SODIUM REABSORPTION in the 
distal nephron 
 Net effect: sodium loss and volume loss 
 Aldosterone 
o Secreted in response to decrease blood volume (via AT2) and an increase in the plasma potassium concentration 
 Causes an increase sodium reabsorption, an increase in potassium secretion, and an increase in proton secretion 
 ADH (Vasopressin) 
o Secreted in response to an increase in plasma osmolarity and a decrease in blood volume 
o ADH binds to receptors on principal cells, causing an increase in the number of aquaporins and thus an increase in water 
reabsorption 
Patient 3   A 70‐year‐old woman is admitted to the intensive care unit (ICU) with abdominal pain and sepsis.  Examination reveals 
hypotension (BP 90/50 mmHg), obtundation, and cool extremities.  The serum creatinine is 3.0 mg/dL which one month 
previously was 1.0 mg/dL.  The urinalysis reveals a specific gravity of 1.010 (same as plasma = not diluting or concentrating the 
urine) and is negative for blood and protein.  On microscopic exam, there are many granular but no cellular casts.  The FENa is 
4% and the FEurea is 50%. 
 What is the most likely diagnosis? 
o Acute tubular injury 
o This patient has sepsis with evidence of acute tubular injury (specific gravity 1.010, many granular casts in the urine) and the 
FENa and Feurea are elevated.  
 If this was Pre‐Renal Injury? 
o concentrated urine (specific gravity > 1.015) 
o hyaline casts 
o low FENa and low Feurea would be expected. 
o No Hematuria and No proteinuria would be expected in glomerulonephritis 
o No pyuria would be expected in interstitial nephritis. 
 
Urinalysis 
 
Urinalysis (UA)   The UA is to Nephrology what the electrocardiogram (EKG) is to Cardiology! 
 There are three portions of a complete UA 
o Appearance of the urine  
o Dipstick evaluation 
o Microscopic examination 
 A negative dipstick usually obviates the need for a microscopic examination. 
pH   The normal urine pH range is 4.5 to 8 (usually 5–7).  
o On a typical U.S. diet, the urine is usually acidic (pH 5‐6) is healthy individuals.  
urine pH < 5  in the presence of metabolic acidosis
urine pH > 5.3  in the presence of metabolic acidosis raises the possibility of distal renal tubular acidosis (RTA). 
very high urine pH (> 8)  suggests the presence of urea‐splitting organisms (e.g., Proteus), in which case production of 
 
ammonia (NH3) will raise the urine pH 
Specific gravity   Specific gravity:  weight of urine relative to distilled water;  
o reflects the number and size (weight) of solute particles in urine 
 The normal range of urine specific gravity is 1.001 (very dilute) to 1.030 (very concentrated).  
o Since the specific gravity of plasma is normally 1.010 
 a urine specific gravity of ~1.010 indicates that the urine is neither concentrated nor dilute (isosthenuria).  
 In a patient with an acute decline in renal function: 
o specific gravity >1.020  
 suggests normal ability to concentrate urine 
 thus prerenal failure (decreased renal blood flow) 
o specific gravity of about 1.010  
 suggests loss of tubular function  
 Thus acute tubular necrosis/acute kidney injury (AKI).  
 In a hyponatremic patient: 
o With an inappropriately high specific gravity (>1.010) suggests antidiuretic hormone (ADH) secretion 
 In a hypernatremic patient: 
o With an inappropriately low specific gravity (<1.010) suggests diabetes insipidus (central or nephrogenic) 
Protein   The dipstick for protein detects primarily albumin.  
 Normal urine usually has no protein by dipstick, but occasionally very concentrated urine will be trace or even 1+ positive for 
protein in healthy individuals.  
 If dipstick proteinuria is detected, it should be quantitated by a random urine albumin‐to‐creatinine ratio (UACR) and/or urine 
protein‐to‐creatinine ratio (UPCR). 
 Simultaneous measurement of both UACR and UPCR is a good screening test for the presence of paraproteinuria, as in 
myeloma, in which case there will be a much more marked increase in urine total protein relative to urine albumin. 
Blood   A positive test indicates heme is present, which can be due to  
o red blood cells (RBCs) 
o myoglobinuria – heme pigment 
o hemoglobinuria – heme pigment 
 Microscopic hematuria is hematuria in the absence of a visual change in color of the urine.  
o As few as 2 to 3 RBCs/high‐power field (hpf) may make the dipstick positive. 
Glucose   Normal urine does not contain glucose  
o because of the reabsorption of filtered glucose by the proximal tubule. 
 Glycosuria with elevated blood glucose indicates diabetes mellitus.  
 Glycosuria with normal blood glucose indicates renal glycosuria 
o which may be isolated or associated with other evidence of proximal tubular dysfunction (phosphaturia, aminoaciduria, 
bicarbonaturia) (Fanconi syndrome). 
Ketones   Normally, there are no ketones in the urine. 
o Ketoacidosis  the body fails to adequately regulate ketone production  
 causing a severe accumulation of keto acids  
 which causes the blood pH to substantially decreased 
 Ketonuria without ketoacidosis suggests starvation 
o low carbohydrate (such as Atkins) diet 
o (rarely) isopropyl alcohol ingestion 
 Ketonuria with ketoacidosis suggests diabetic or alcoholic ketoacidosis.  
o Note that in some patients with ketoacidosis, the dipstick may be negative due to the reduction of acetoacetate to beta‐
hydroxybutyrate (the latter is not detected by the dipstick). 
Bilirubin   Normally, there is no bilirubin in the urine.  
 If present, this suggests hepatobiliary disease  
o failure to conjugate and/or excrete bilirubin into the gut 
Urobilinogen   Bilirubin is secreted in bile into the gut 
o where it is metabolized by microorganisms into urobilinogen.  
 Urobilinogen is then absorbed and partially excreted into the urine.  
 In the presence of liver disease 
o urobilinogen can accumulate in plasma and appear in the urine.  
 Elevated Bilirubin in the blood and urine without urobilinogen in the urine suggests biliary obstruction. 
 
 
Leukocyte esterase   This is an enzyme found in white blood cells (WBCs) and indicates the presence of pyuria,  
o Pyuria is the presence of pus or WBC in the urine 
 This can be due to either urinary tract infection (UTI) or inflammation (such as interstitial nephritis). 
Nitrite   Enterobacteria convert urinary nitrate to nitrite 
o therefore a positive nitrite test suggests UTI.  
 Note that not all organisms make nitrite, so UTI may be present with a negative nitrite. 
 
Microscopic Exam   This is necessary if the dipstick is positive 
   Sample contains 
o Calcium oxalate (hypocitraturia) – crystal is shaped like an envelope or dumbbell 
o Cystine (low pH) – crystal is shaped hexagonal  

   Under polarized light, the sample contains 
o Uric acid (low pH) – crystal shape is rhomboid or rosettes 

   Sample contains: 
o Ammonium magnesium phosphate or struvite (high pH) – coffin lid 

   RBC casts due to… 
o Glomerulonephritis 
o Malignant hypertension 

   WBC casts due to… 
o Tubulointerstitial inflammation 
o Acute pyelonephritis 
o Transplant rejection 
   Granular (Muddy Brown) casts due to… 
o Acute tubular necrosis 

 
 
 
 
 
 
 
 
 
 
 
 
 
 
 
 
 
 
 
 
 
 
 
 
 
 
 
 
 
Patient 1   A 70‐year‐old man with a long history of tobacco use presents to the ER with weakness.  Physical examination reveals mild 
supine hypertension without orthostatic changes.  There is an epigastric bruit.  There is no edema.  The serum creatinine is 3.0 
mg/dL which one month previously was 1.0 mg/dL.   
 The UA reveals: 
 Color ‐ yellow   Urobilinogen ‐ negative 
 pH ‐ 5.5   Leukocyte esterase ‐ negative 
 Specific gravity ‐ 1.025 (high)   Nitrite ‐ negative 
 Protein ‐ negative   WBC ‐ 2/hpf (normal) 
 Blood ‐ negative   RBC ‐ 3/hpf (upper end of normal) 
 Glucose ‐ negative   Bacteria ‐ negative 
 Ketones ‐ negative   Many hyaline casts – nonspecific and can be a normal finding often seen in concentrated urine samples 
 Bilirubin ‐ negative   Few granular casts 
 What is the most likely diagnosis? 
o Renovascular disease 
 The most likely diagnosis in this elderly male smoker with an epigastric bruit and acute azotemia is bilateral renal artery 
stenosis (D).   
 Acute glomerulonephritis is excluded by the absence of proteinuria and hematuria and interstitial nephritis by the absence 
of significant pyuria.   
 Acute tubular injury would lead to isosthenuria (specific gravity same as plasma, i.e., 1.010), whereas this patient has 
concentrated urine, as is expected in any form of pre‐renal failure, including bilateral renal artery stenosis.  
 The patient underwent renal angiography which revealed total occlusion of the right renal artery and 95% stenosis of the 
left renal artery.   
 Angioplasty and stenting of the left renal artery resulted in improvement of renal function. 
Patient 2   A 22‐year‐old medical student who was previously healthy sees a physician because of symptoms of an upper respiratory 
infection (rhinorrhea, sneezing) and red urine.  Physical examination is normal. The serum creatinine is 3.0 mg/dL which one 
month previously was 1.0 mg/dL.    
 The UA reveals: 
 Color ‐ red   Urobilinogen ‐ negative 
 pH ‐ 5.5   Leukocyte esterase ‐ negative 
 Specific gravity ‐ 1.020 (high)   Nitrite ‐ negative 
 Protein ‐ 3+   WBC ‐ 2/hpf 
 Blood ‐ 3+   RBC ‐ >100/hpf (high) 
 Glucose ‐ negative   Bacteria ‐ negative 
 Ketones ‐ negative   Few hyaline casts 
 Bilirubin ‐ negative   Many granular casts 
 Mod RBC casts 
 What is the most likely diagnosis? 
o Acute glomerulonephritis 
 The most likely diagnosis in this medical student with proteinuria and hematuria and RBC casts in the urine is acute 
glomerulonephritis (GN).   
 In acute GN the urine specific gravity is usually elevated and there is often edema due to renal sodium retention.   
 This is because glomerular inflammation leads to decreased glomerular blood flow 
 resulting in decreased post‐glomerular perfusion and stimulation of sodium and water reabsorption by the kidneys. 
 Interstitial nephritis is characterized by pyuria and WBC casts.   
 Acute tubular injury would be expected to cause isosthenuria and pigmented (“muddy brown”) granular casts and not 
hematuria.   
 A renal biopsy confirmed the expected diagnosis of Immunoglobulin A (IgA) nephritis.   
 This is the most common cause of glomerulonephritis in the U.S.   
 The history of a URI accompanied by gross hematuria is characteristic of this disorder. 
Patient 3   A 60‐year‐old man presents to the ER with fatigue.  He saw a physician for urinary frequency and urgency one week ago and was 
told he has a urine infection and was given an antibiotic (he does not remember the name).  Examination reveals hypertension 
(BP 160/100 mmHg), a generalized skin rash, and 1+ lower extremity edema.  The serum creatinine is 3.0 mg/dL which one 
month previously was 1.0 mg/dL.   
 The UA shows: 
 Color ‐ yellow   Urobilinogen ‐ negative 
 pH ‐ 5.5   Leukocyte esterase – positive  Pyuria 
 Specific gravity ‐ 1.010  Isosthenuria   Nitrite ‐ negative 
 Protein ‐ 1+   WBC ‐ 50/hpf 
 Blood ‐ negative   RBC ‐ 3/hpf 
 Glucose ‐ negative   Bacteria ‐ negative 
 Ketones ‐ negative   Few hyaline casts 
 Bilirubin ‐ negative   Many granular casts 
 Occasional WBC casts 
 What is the most likely diagnosis? 
o Acute interstitial nephritis 
 The most likely diagnosis in this middle‐aged man with pyuria and mild proteinuria, hypertension, acute azotemia, and 
recent antibiotic exposure is acute interstitial nephritis (AIN) 
 In AIN, the urine specific gravity often about 1.010 
 indicating inability to concentrate or dilute urine (isosthenuria).   
 Acute tubular injury is also characterized by isosthenuria but pyuria would not be expected in this case.   
 Renal biopsy confirmed the diagnosis.   
 
Patient 4   A 70‐year‐old woman is admitted to the intensive care unit (ICU) with abdominal pain and sepsis.  Examination reveals 
hypotension (BP 90/50 mmHg), obtundation, and cool extremities.  The serum creatinine is 3.0 mg/dL which one month 
previously was 1.0 mg/dL.   
 The UA shows: 
 Color ‐ yellow   Urobilinogen ‐ negative 
 pH ‐ 5.5   Leukocyte esterase ‐ negative 
 Specific gravity ‐ 1.010  isosthenuria, thus renal failure    Nitrite ‐ negative 
 Protein ‐ trace   WBC ‐ 2/hpf 
 Blood ‐ negative   RBC ‐ 3/hpf 
 Glucose ‐ negative   Bacteria ‐ negative 
 Ketones ‐ negative   Few hyaline casts 
 Bilirubin ‐ negative   Many pigmented granular casts 
 No cellular casts 
 What is the most likely diagnosis? 
o Acute tubular injury 
 The most likely diagnosis in this elderly woman with sepsis and isosthenuria with pigmented (“muddy brown”) granular 
casts in the urine is acute tubular injury (formerly called acute tubular necrosis, or “ATN”).    
 There is no hematuria or pyuria to suggest GN or AIN, respectively.   
 Pre‐renal failure due to hypotension could be entertained, but one would in that case expect the urine to be concentrated 
(specific gravity > 1.010) and contain hyaline and not pigmented granular casts.   
 The patient was diagnosed with acute cholecystitis and bacteremia.   
 She responded well to antibiotics with eventual improvement of her renal function.   
 
Nephritic Syndrome 
 Nephritic Syndrome = glomerular injury is primarily due to neutrophils 
 Primarily nephritic types of glomerular diseases 
o Postinfectious Glomerulonephritis 
o IgA Nephropathy (Berger Disease)  
o Hereditary nephritis 
o Rapidly Progressive Glomerulonephritis (Type 1, 2, 3) 
Clinical and laboratory findings 
Sign/Symptom  Description 
Hypertension   Due to salt retention 
Periorbital edema   Due to salt retention 
Oliguria   Approximately 400 mL urine/day 
 Due to decrease GFR from inflamed glomeruli 
 Tubular function is intact 
Hematuria   Dysmorphic RBCs are present with irregular membranes 
 Glomeruli becomes inflamed from Immune Complex deposition causing damage to RBC membranes 
Neutrophils   Present in the urine sediment 
 Particularly in Immune Complex types of nephritic glomerulonephritis 
RBC casts   Key finding 
 Occasionally, WBC casts are also present 
Proteinuria   >150 mg/day but <3.5 gm/day 
Azotemia   BUN/Cr ratio >15 
 Tubular function is intact in acute glomerulonephritis 
 
Postinfectious Glomerulonephritis 
(Acute Proliferative/ Poststreptococcal Glomerulonephritis/ Serum Sickness) 
Clinical   Typical Preentation: 
o Children: (6‐10 years old)  
 Presents as acute nephritic syndrome including 
 Hematuria 
 Edema 
 Hypertension – associated with headache 
 renal failure (95% recovery).  
o Adults: (rare)  
 Presents as acute nephritic syndrome is less common 
 slow progression to chronic glomerulonephritis with 15‐50% of adults developing end stage kidney disease.  
 Pathogenesis 
o develops 1‐4 weeks after recovery from infection… 
 a prior pharyngeal infection by streptococci (or other bacteria or even viruses) 
 following impetigo.  
o Group A beta‐hemolytic streptococcal infection (GABHS, nephritogenic, M protein virulence factor ) 
 Diagnosis: 
o tea‐color (smoky or coca‐cola‐like) urine 
o hematuria with RBC casts 
o mild proteinuria.  
o ASO (Anti‐streptolysin O) may be elevated  
o Complement (C3) is typically low.  
o Humps: (in‐situ) ab binds bacterial antigens planted in the glomerular basement membrane.  
o Cationic nephritogenic antigens (SpeB: streptococcal exotoxin B, streptococcal GAPDH) traverse anionic glomerular basement membrane to 
subepithelial location. 
 Prognosis: 
o children total recovery with resolution of pathology in >95% 
o adults slow progression to chronic glomerulonephritis 
o 15‐50% of adults develop end stage kidney disease 
o a small subset of children and adults may develop very severe acute illness with gross hematuria and rapidly progressive renal failure 
 Treatment: 
o supportive 
Mechanism   glomerulonephritis follows prior infection 
 immune complexes form in the circulation (antigen + IgG antibody) 
 deposition of immune complexes in the capillary wall  
o activates complement  
 which attracts neutrophils  
 which mediate damage with endocapillary proliferation and structural damage (“swiss cheese”) with hematuria  
 immune complexes are also formed in‐situ  
o leading to formation of big subepithelial deposits “humps” (white arrow)  
 which are unique to postinfectious glomerulonephritis and therefore diagnostically useful 
 It is not clear if immune complexes are formed mainly in the circulation or in situ by binding of antibodies to bacterial antigens “planted” in the 
glomerular basement membrane  
 
Microscopy  Light   Normal – glomerular capillaries patent 

 
   Diffuse endocapillary proliferation  no longer see the capillary lumen 
 leukocytic infiltration 
 Capillaries obliterated by endocapillary proliferation & influx of PMNs (3‐lobed nuclei) 

   
Fluorescence   Granular “lumpy bumpy” IgG and C3 in GBM and mesangium; Granular IgA in some cases.  

 
Electron   Primarily subepithelial humps; subendothelial electron dense deposits in early disease stages. 

 
   
 
An 8‐year‐old boy presents with headaches, dizziness, and malaise.  He was seen for a severe sore throat 2 weeks ago.  Physical 
examination reveals facial edema.  The blood pressure is 180/110 mm Hg (Hypertension).  A 24‐hour urine collection demonstrates 
oliguria, and urinalysis shows hematuria. What test(s) may be helpful in the differential diagnosis in this child? 
A. ASO (antistreptolysin O antibody) 
B. Complement level 
C. both 
D. neither 
 
IgA Nephropathy (Berger Disease)   
“IgA nephropathy” refers to disease with deposits of IgA limited to kidney. 
Clinical   Typical Presentation: 
o Affects children and young adults (mostly males) 
o Recurrent gross hematuria within 1‐2 days of a nonspecific upper respiratory tract infection;  
 Alternatively, after GI or UTIs. 
 “painless hematuria following infection” = suggestive of IgA nephropathy 
 Epidemiology: 
o IgA nephropathy  = most common glomerular disease worldwide 
o children and young adults 
 Pathogenesis 
o mucosal infection leads to production of IgA & formation of IgA containing immune complexes which deposit in the mesangium 
o activation of the complement via alternative pathway 
o abnormalities in clearance of IgA (liver disease – decreased hepatobiliary clearance, secondary IgA nephropathy) 
o genetic or acquired abnormality of immune regulation (celiac disease) 
 Pathology: 
o mesangial proliferation of variable intensity 
o IgA‐containing immune complexes in the mesangium = diagnostic 
o electron dense deposits by electron microscopy 
 Diagnosis: 
o hematuria, mild proteinuria 
o complement levels are replenished by the liver and usually not decreased 
 Prognosis: 
o variable, dependent on glomerular pathology 
o many relatively indolent/prolonged, ultimately leads to renal failure  
o rare patients with clinical course of rapidly progressive glomerulonephritis (discussed later) 
o Systemic IgA vasculitis in children– Henoch‐Schӧnlein purpura (HSP) – excellent prognosis 
 Treatment: 
o Supportive 
Mechanism   The multi‐hit pathogenesis model of IgA nephropathy  
o Hit 1 = increased production of galactose‐deficient IgA1 (after respiratory/gastrointestinal exposure to viruses, bacteria, 
food products)   
o Hit 2 = formation of autoantibodies that recognize galactose‐deficient IgA1 
o Hit 3 = subsequent formation of pathogenic immune complexes 
o Hit 4 = deposition of immune complexes in the mesangium, activation of mesangial cells, induction of glomerular injury 
complement activation via alternative pathway 
 
 
 
 
Microscopy  Light   Focal mesangial (between the capillaries) proliferative glomerulonephritis; mesangial widening 

 
Fluorescence   IgA ± IgG, IgM, and C3 in mesangium 

 
Electron   Mesangial and para‐mesangial dense deposits 

 
 
 
A 32‐year‐old man complains of recurrent hematuria since his youth.  The hematuria typically occurs following upper 
respiratory tract infections.  Vital signs are normal.  Urinalysis shows hematuria and mild proteinuria.  Laboratory 
studies disclose normal levels of BUN and creatinine (NL=0.7‐1.5).   Which of the following is the most likely diagnosis? 
A. Alport syndrome 
B. IgA nephropathy 
C. Hereditary nephritis 
D. pneumonia 
E. Postinfectious glomerulonephritis 
 
 
Henoch‐Schönlein Purpura (HPS) 
(IgA Nephropathy plus systemic IgA deposition) 
Clinical   Typical Presentation:  
o Abdominal distention/pain 
o GI bleeding 
o Arthralgia 
o Palpable purpura on legs and buttocks 
 Epidemiology: 
o Systemic childhood disorder (3‐8 yo) 
o Onset follows Upper respiratory infection  
o Associated with IgA nephropathy  Hematuria 
 Pathogenesis 
o IgA nephropathy has a predilection for young adults while HSP is more predominant among children 
 Pathology: 
o The main findings are increased cells and Ig deposition in the mesangium, white blood cells, and the 
development of crescents.  
o The changes are indistinguishable from those observed in IgA nephropathy. 
 Diagnosis: 
o Blood tests show: 
 elevated creatinine and urea levels with kidney involvement 
 raised IgA levels 
 raised CRP or ESR 
 Prognosis: 
o Good in most patients with recovery occurring 94% of children and 89% of adults 
o In adults, kidney involvement progresses to end stage renal disease 
 Treatment: 
o Supportive 
Mechanism   Deposits of IgA may be also systemic and involve kidneys with hematuria as well as skin with purpuric skin lesions, gastrointestinal tract with 
bleeding and abdominal pain and arthralgia. 
Microscopy  Light   hypersensitivity vasculitis, and immunofluorescence demonstrates IgA and C3 (a protein of the complement system) in the 
blood vessel wall 

  
Fluorescence   IgA deposits are found in the walls of small superficial capillaries (yellow arrows). 

  
 
Electron   
 
Alport Syndrome  
(Principle example of Hereditary Nephritis) 
Clinical   Typical Presentation: 
o Case example: 
 A 20 year old male was found to have hematuria via pre‐employment testing; scr (serum creatinine): 2.5 MG/DL [n=0.7‐1.5]; Family history: 
several members of his family, mostly males, with chronic renal failure by age 50 years; affected individuals with hearing problems, various 
eye disorders, including lens dislocation.   
o Characterized by  
 hereditary nephritis (isolated hematuria without infection) 
 hearing loss 
 ocular abnormalities (early cataracts) 
 Epidemiology: 
o age 5‐20 years at presentation 
o 20‐50 years with overt renal failure 
 Pathogenesis 
o X‐linked inheritance most commonly 
 Caused by a defect in type 4 collagen  
 due to a mutation in the COL4A5 gene coding for the alpha‐3, alpha‐4, and alpha‐5 chains of type 4 collagen 
o (+) family history 
 Male patients – full spectrum 
 Females – carriers, rare with disease (X‐chromosome inactivation) 
 Pathology: 
o paraffin sections: normal, non‐diagnostic  
o NO immune complexes 
o electron microscopy diagnostic 
 Diagnosis: 
o hematuria 
o genetic testing 
 Prognosis: 
o overt renal failure between 20‐50 years of age 
 Treatment: 
o supportive, transplantation, counseling, family testing 
Mechanism   Gross or microscopic hematuria begins in childhood. Hearing loss (leading to sensorineural deafness) and various ocular abnormalities of the lens 
and cornea can occur. Alport is a progressive disease that ultimately results in renal failure. 
Microscopy  Light   
Fluorescence   Negative   
Electron   Alternating thickening and thinning of basement membrane with splitting of the lamina densa, causing a basket weave 
appearance 

  
 “basket weave” 

 
  
 
A 20 year old male was found to have hematuria via pre‐employment testing. Also his scr (serum creatinine) was 
elevated at 2.5 MG/DL [n=0.7‐1.5].  He has been otherwise healthy.  Several members of his family, mostly males, were 
diagnosed with chronic renal failure by age 50 years. Which best applies to this patient: 
A. recommend repeated testing at age of 50 
B. eye testing may be needed, no hearing testing needed 
C. only hearing testing needed 
D. he has IgA nephropathy 
E. eye and hearing testing may be needed       
 
 
 
 
 
 
 
 
 
Rapidly progressive glomerulonephritis (RPGN) Type I (Goodpasture Syndrome) 
(Linear – anti‐basement membrane antibody) 
Clinical   Typical Presentation: 
o Case Example 
 Young male (college student); hemoptysis; blood in urine, gross; drop in urinary output 
 History: smoker; working in a car garage during summer; other industrial exposure 
 UA: abundant red blood cells with casts; sCr elevated to 5.0 MG/DL [n=0.7‐1.5] 
 Chest X‐ray bilateral pulmonary opacities 
 Characterized by: 
o hemoptysis and pulmonary hemorrhage *pulmonary involvement) 
o gross hematuria 
o Oliguria ‐ drop in urinary output (acute renal failure) 
o NO sinusitis  
 Epidemiology: 
o young MEN (20‐40 yo) 
o rare, 12% of crescentic glomerulonephritis 
 Pathogenesis 
o Type 2 Hypersensitivity Reaction  Goodpasture Syndrome 
o anti‐glomerular basement membrane antibodies  
o exposure: viruses, smoking, solvents (paints, dyes), drugs 
o genetic predisposition to autoimmunity 
 Pathology: 
o Crescents 
o Linear stain for IgG by immunofluorescence, not seen by electron microscopy 
o in glomeruli/pulmonary alveoli 
 Diagnosis: 
o anti‐glomerular basement membrane antibodies in serum levels may be low in rapid binding on the kidney 
 Prognosis is Poor: 
o renal failure 
o pulmonary failure 
 Treatment: 
o Plasmapheresis, (removal of pathogenic antibodies from the circulation) 
Mechanism   Goodpasture Disease/Syndrome: Antibody cross‐reactivity with pulmonary alveolar basement membrane 
o Linear IgG deposits along glomerular and alveolar basement membranes to a novel antigen (non‐collagenous protein NC1) 
 NC1 is normally hidden and does not elicit an antibody response 
o Clinically: RPGN with hematuria + pulmonary hemorrhage (hemoptysis) = pulmonary‐renal syndrome 
Microscopy  Light   Hypercellularity proliferation, crescents, and fibrin deposition in glomeruli 

  
Fluorescence   Smooth and linear pattern of IgG and C3 along the glomerular basement membrane 

    
Electron   There are no deposits, but there is glomerular basement membrane disruption 

 
  
 
 
 
 
 
 
 
 
 
 
 
 
 
 
 
 
 
Rapidly progressive glomerulonephritis (RPGN) type II 
(Granular – immune complex deposition) 
Clinical   Typical Presentation: 
o Characterized by: 
 gross hematuria 
 drop in urinary output (acute renal failure) 
 Epidemiology: 
o rare ‐1% of postinfectious,  
o small subset of IgA,  
o systemic lupus erythematosus [SLE] 
o older children, young adults (10‐40 yo) 
 Pathogenesis 
o severe immune complex formation with necrosis and 
o breaks in glomerular basement membrane 
 Pathology: 
o Crescents with immune complexes: as compared to other forms of glomerulonephritis that are not complicated by crescents 
 IgG+C3 granular deposits containing immune complexes also seen in Postinfectious Glomerulonephritis and Lupus Nephritis 
 IgA+C3  deposits containing immune complexes in the mesangium also seen in IgA Nephropathy 
o electron dense deposits by electron microscopy 
 Diagnosis: 
o depending on etiology could be: 
 postinfectious, IgA nephropathy, diffuse proliferative lupus 
 Prognosis: 
o chronic renal failure 
 Treatment:  
o Immunosuppression 
Mechanism   except for the severity and the presence crescents, the other diagnostic pathology remains the same as in the corresponding forms of 
glomerulonephritis not complicated by crescents (the prototype): 
o granular deposits containing immune complexes of IgG + Complement in postinfectious glomerulonephritis and lupus nephritis 
o IgA + Complement immune complexes in IgA nephropathy 
Microscopy  Light   
Fluorescence   Shows granular pattern 

    
 
Electron   Shows discrete deposits 
 
Rapidly progressive glomerulonephritis (RPGN) type III 
(Negative IF – pauci‐immune) 
Clinical   Typical Presentation: 
o Characterized by: 
 drop in urinary output (acute renal failure) 
 gross hematuria  
 hemoptysis (cough with blood tinged sputum) 
 shortness of breath 
 recurrent “sinusitis” 
 Palpable purpura  associated with vasculitis  
 Epidemiology: 
o Older patients 
 Pathogenesis 
o antineutrophil cytoplasmic autoantibodies (ANCA) 
 Pathology: 
o Crescents 
o NO immune complex deposits 
o NO anti‐glomerular basement membrane autoantibodies 
 Diagnosis: 
o ANCA: antineutrophil cytoplasmic autoantibodies 
 Prognosis: 
o renal failure 
o pulmonary 
 Treatment: 
o immunosuppression 
Mechanism   ANCA = autoantibody  
o heterogeneous group of autoantibodies 
o formation induced by drug, cross reactive microbial antigen, other… 
o react with neutrophil antigens causing premature degranulation/activation, release of lytic enzymes leading to vascular damage 
o present in serum 
o do not form circulating immune complexes 
o ANCA = antibody‐mediated disease (type II hypersensitivity)  direct cause of pauci‐immune crescentic glomerulonephritis/systemic vasculitis 
o highly sensitive diagnostic marker of pauci‐immune glomerulonephritis/systemic vasculitis 
 RPGN type III may be a component of systemic vasculitis such as:  
o microscopic polyangiitis 
o Granulomatosis with polyangiitis [GPA] (aka pulmonary angiitis and granulomatosis, Wegener granulomatosis)  
 C‐ANCA (PR3‐ANCA) 
 Vasculitis may involve upper respiratory tract (Ear/nose/throat), Lung and Kidney (ELK) synchronously or in turns. 
o Eosinophilic Granulomatosis with Polyangiitis [EGPA] (aka allergic granulomatosis and angiitis, Churg‐Strauss 
syndrome) 
 P‐ANCA (MPO‐ANCA) 
 allergic granulomatosis and angiitis:  
 small vessel vasculitis associated with peripheral eosinophila 
 eosinophilia associated symptoms (asthma, allergic rhinitis, lung infiltrates) 
 Immunofluorescence on neutrophils:  
o cytoplasmic (c‐ANCA) or  
o perinuclear (p‐ANCA) pattern 
o c‐ANCA antigen: proteinase 3 (PR3)  
o p‐ANCA antigen: myeloperoxidase (MPO) 
o Currently ELISA assay; see vasculitis lecture… 
Microscopy  Light   crescents (in response to glomerular capillaries vasculitis) 

  
Fluorescence   Negative (“pauci immune”)  

  
 
Electron   Negative (“pauci immune) 
 
 
A 35‐year‐old man with a history of smoking presents with hematuria and bloody sputum.  Over the next 2 days, he develops oliguria 
and renal failure, after which he is placed on dialysis.  A renal biopsy is stained with antihuman IgG, and the results are shown.  
Which of the following best described the pattern of direct immunofluorescence observed on this photomicrograph?  

 
A. Discontinuous and mesangial 
B. Finely granular along the perimesangial reflections 
C. Linear along the glomerular basement membrane 
D. Mesangial  
E. Peripheral granular humps 
 
A 68‐year‐old man complains of nasal obstruction, bloody nose, cough and bloody sputum.  A chest x‐ray displays cavitated lesions 
and multiple nodules within both lung fields.  Urinalysis reveals 3+ hematuria and red blood cells casts.  Laboratory studies show 
anemia and elevated serum levels of C‐ANCA (antineutrophil cytoplasmic antibody).  Peripheral eosinophils are not increased.  A 
renal biopsy exhibits focal glomerular necrosis with crescents.  What is the appropriate diagnosis? 
A. eosinophilic granulomatosis with polyangiitis (Churg‐Strauss syndrome) 
B. Goodpasture syndrome 
C. Hypersensitivity vasculitis 
D. postinfectious glomerulonephritis 
E. granulomatosis with polyangiitis (Wegener granulomatosis) 
 
A 30‐year‐old man with a history of smoking suddenly develops oliguria, hematuria and hemoptysis.  Serologic studies reveal 
antibodies to the glomerular basement membrane.  Which of the following pathologic changes is visible by light microscopy in this 
biopsy specimen? 
A. Crescents in the urinary space 
B. Leukocytic infiltrates in the glomeruli 
C. Mesangial cell proliferation 
D. Thickening of the glomerular basement membrane 
E. Thrombi in glomerular capillaries 
 
A 16‐year‐old boy comes to the physician with a 1‐year history of intermittent, painless hematuria without dysuria on increased 
frequency of micturition.  He says he has also had several respiratory infections and adds that the hematuria increased within 
several days of the infections.  Which of the following is most likely? 
A. Increased antistreptolysin O titer 
B. mutation in basement membrane protein gene  
C. IgA mesangial deposits 
D. Proteinuria exceeding 3.5 gm/24 h 
E. subepithelial humps deposits 
 
Nephrotic Syndrome 
 Nephrotic Syndrome = glomerular injury due to cytokines and NOT neutrophils 
o Cytokines damage podocytes, causing them to fuse together 
o Cytokines destroy the negative charge of the glomerular basement membrane 
 Primarily nephrotic types of glomerular diseases 
o Membranous nephropathy 
o Minimal Change Disease 
o Focal Segmental Glomerulosclerosis (FSGS) 
Clinical and laboratory findings 
Sign/Symptom  Description 
Proteinuria   >3.5 gm in 24 hrs  key finding 
Hypoalbuminemia   Albumin level < 3 gm/dL 
 Generalized pitting edema and ascites due to hypoalbuminemia cause a decrease in plasma oncotic pressure 
o Pitting edema in Nephritic Syndrome 
 due to sodium retention causing an increase in plasma hydrostatic pressure 
o Increased risk for developing spontaneous peritonitis (inflammation of the peritoneum) 
 due to Streptococcus pneumoniae or Escherichia coli 
Hypertension   due to sodium retention 
Hypercoagulable state   due to the loss of Antithrombin 3  potential for renal vein thrombosis 
Hypercholesterolemia   Hypoalbuminemia increases synthesis of cholesterol  leading to Hyperlipidemia and Lipiduria (lipid droplets in urine) 
Hypogammaglobulinemia   Due to the loss of gamma‐globulins in the urine 
Fatty casts   With maltese crosses and oval fat bodies  key finding of nephrotic syndrome 
 
The nephrotic syndrome is characterized by severe  A. A combination of small pore size and negatively charged pore‐forming molecules 
proteinuria, decreased serum albumin level, and edema.   prevents albumin filtration 
This result from damage to one or more components of the  B. A combination of small pore size and positively charged pore‐forming molecules 
glomerular capillary wall.  In particular, the glomerular  prevents albumin filtration 
basement membrane is essential for maintaining serum  C. Albumin is freely filtered across the basement membrane but is readily reabsorbed 
oncotic pressure.  In nonpathologic states, which of the  along the nephron 
following properties of the glomerular basement membrane  D. The positive charge of proteoglycans in the basement membrane repels albumin 
prevent albumin from being freely filtered into the urine?  E. The small size of the glomerular basement membrane pores excludes albumin 
molecules 
 
Membranous Glomerulonephritis 
(Membranous Nephropathy) 
Clinical   Typical Presentation:   Example Case: 
o Male, young (20‐40 yo); deep vein thrombosis (after air trip, pulmonary embolus due to loss of antithrombin 3); 
edema, eye puffiness, shortness of breath, “tight shoes”; Urine “foamy”; UA: proteinuria 3.5‐12 gm/24hrs; 
Cholesterol 320 MG/DL; Serum creatinine: 1.5 mg/DL [n=0.7‐1.5]; Clinical diagnosis = nephrotic syndrome 
 Characterized by: 
o edema, in elderly can masquerade as cardiac failure (puffy eyes) 
o thrombosis – due to loss of anti‐thrombin III 
o increased risk of infections 
o associated with primary diseases versus secondary diseases as in 10% SLE patients 
o Proteinuria (non‐selective) – with urinary loss of globulins and albumin 
 Epidemiology:   young/middle age adults between 30 and 60 years of age 
 30% adults, second most common cause nephrotic syndrome (children 5%) 
 85% autoimmune (“idiopathic”), 15% secondary 
 Pathogenesis   Characterized by the presence of subepithelial granular deposits (in situ immune complex formation) 
 85% of cases is caused by autoimmune cross‐react with renal antigen (intrinsic or planted), SLE 
 Secondary membranous nephropathy occurs to other disorders, including: 
o Cancer:  
 carcinomas (lung, colon, melanoma), leukemia, non‐Hodgkin’s lymphoma 
o infections:  
 malaria, hepatitis B, syphilis, schistosomiasis 
o Autoimmune conditions: 
 Systemic lupus erythematous 
o Toxins 
 Inorganic salts:  
 gold, mercury 
 Drugs:  
 Penicillamine, Captopril, Nonsteroidal anti‐inflammatory agents 
 Pathology:   no inflammation (due to subepithelial deposits), no proliferation, capillary wall thickening  glomerulus looks 
normal 
 IF = gG + Complement (C3) granular deposits 
 EM = subepithelial electron dense deposits  
 Effacement loss of foot processes 
 
 Diagnosis:   nephrotic syndrome (hypoalbuminemia, hyperlipidemia and lipiduria) 
 secondary hyperlipidemia, increase total cholesterol, increase low density lipoproteins (LDL) cholesterol, 
accelerated atherogenesis 
 NO complement drop ‐ chronic, relatively slowly progressing 
 antibody testing for PLA2R, THSD7A (for idiopathic membranous) 
 Prognosis:   One third of patients have spontaneous remission 
 One third progress to require dialysis 
 One third continue to have proteinuria, without progression of renal failure 
 Treatment:   difficult, immunosuppressive drugs (Prednisone) DOES NOT RESPOND TO STEROIDS (USUALLY) 
 non‐specific anti‐proteinuric 
 secondary: treatment of the underlying disease 
 
 recurrence after transplantation 
Mechanism   Membranous nephropathy: review 
o The experimental model of membranous nephropathy was described by Heymann in 1959 in rats (see recorded session “introduction to 
glomerular diseases”).  However, it was not until recently that this model could be validated in humans… 
 several families with neonatal membranous nephropathy 
 mutations in neutral endopeptidase (NEP, podocyte antigen) in mothers with normal fetus 
 with pregnancy, mother formed antibodies to NEP which circulated across placenta to the NEP antigen of the fetus and led to the in‐situ 
formation of sub‐epithelial immune complexes in the glomeruli of the fetus, and clinically with nephrotic syndrome in the newborn. 
 these observations validated the in situ paradigm in human membranous nephropathy 
 in “idiopathic” cases of human membranous nephropathy – proteomic analysis of the target antigen identified it as phospholipase A2 receptor 
(PLA2R) 
 another target: thrombospondin type‐1 domain containing 7A (THSD7A) 
 PLA2R and THSD7A in 80% 
 20% ? 
Microscopy  Light   “spike and dome” or “holey”on silver stain 
o immune complex deposits = silver negative 
o basement membrane = silver positive (black) 
     
 NO inflammation, no proliferation, diffuse thickening of 
the basement membrane” 
 
 

Fluorescence   granular deposits of IgG and C3 along basement 
membrane 
 
 
 
Electron   subepithelial deposits 

 
 
 
 
A 50‐year‐old man with a history of large bowel obstruction is diagnosed with colon cancer  A. A spike‐and‐dome pattern of deposition on silver stain 
and undergoes resection of his colon.  He returns to his physician for his regular checkup  B. proliferative glomerulonephritis 
and complains that in the past 3 weeks he has not been feeling well and has noticed  C. hump‐like subepithelial deposits on light microscopy 
significant swelling of his legs.  On physical examination, the physician notes 2+ pitting  D. Nonlinear mesangial staining with IgA 
edema and a blood pressure of 155/94 mm Hg.  Urinalysis shows 4+ protein with no RBCs or  immunofluorescence 
casts.  The patient has otherwise been healthy.  Which of the following would most likely be  E. “Splitting” of the lamina densa 
present on a kidney biopsy from this patient? 
A 44‐year‐old Caucasian man complains of swelling of his legs and puffiness around his  A. anti‐glomerular basement membrane 
eyes.  His abdomen has become protuberant and he feels shorts of breath.  Physical  glomerulonephritis (linear along basement membrane) 
examination reveals generalized edema and ascites.  Total serum protein is 5.2 g/dl  B. IgA nephropathy (granular in mesangium) 
(reference = 5.5‐8.0 g/dl), and albumin is 1.9 g/dl (reference = 3.5‐5.5 g/dl).  Serum  C. hereditary nephritis (IF = negative, EM = basket weave) 
cholesterol is elevated at 530 mg/dL.  There are 5 g of protein in a 24‐hour urine collection.   D. Membranous glomerulopathy (granular) 
The urinary sediment contains many hyaline casts but no RBCs or inflammatory cells.  A  E. postinfectious glomerulonephritis 
renal biopsy stained by direct immunofluorescence for IgG is shown.  Which of the 
following is the most likely diagnosis? 
 
 
 
 
 
 
 
 
 
 
 
 
Minimal Change Disease 
(Lipoid nephrosis, Nll Disease) 
Clinical   Typical Presentation:   Example Case:  
o 3 yo boy with periorbital and generalized edema; UA: proteinuria, 2.5 gm/24 hr; Clinical diagnosis: 
nephrotic syndrome 
 Characterized by: 
o edema (periorbital, generalized) 
 Epidemiology:   children 1‐7 yo, most common cause of nephrotic syndrome 
o no biopsy if uncomplicated clinical course, presumed minimal change disease 
o 95% of nephrotic syndrome in children 1‐4 years is Minimal Change Disease  very prevalent  
o 75% of nephrotic syndrome in children 7‐8 years is Minimal Change Disease 
 adults only 10% do need a kidney biopsy 
 Pathogenesis   reversible podocyte injury 
 proteinuria has been attributed to  
o circulating (T cell derived) cytokines that cause podocyte damage and effacement of foot processes   
o depression of immunity (viral infections, Hodgkin disease)  
o NSAIDs (non‐steroidal anti‐inflammatory drugs) 
 Pathology:   normal glomeruli in paraffin sections 
 no immune complex deposits 
 foot processes effacement by electron microscopy 
 Diagnosis:   nephrotic syndrome – protein loss is selective (smaller plasma proteins  albumin) 
 Prognosis:   Very good for children 
o several episodes of nephrotic syndrome (relapses), normal renal function (!) 
o resolution @ puberty 
 
 Treatment:   RESPONDS TO STEROIDS 
Mechanism   Minimal change disease – REVERSIBLE podocyte injury 
o Podocyte injury: 
 podocyte foot processes effacement, loss of slit diaphragm, reversal upon treatment 
 no‐immune complex deposition, “non‐inflammatory”  
o Clinical: NEPHROTIC syndrome 
o Relapses, but NO KIDNEY FAILURE 
Microscopy  Light   no inflammation/no cellular proliferation 
 no immune complex deposits 
 
 
 Uniform and diffuse effacement (fusion) of the epithelial 
cell foot processes (red arrows) 
 
 

Fluorescence   Negative 
Electron   effacement (fusion) of the epithelial cell foot processes 
(red arrows) 
 
 
 
 
 
 
 
 
 
 
 
 
 
 
 
 
 
 
 
 
 
 
 
 
 
 
Focal Segmental Glomerulosclerosis (FSGS) 
Clinical   Typical   Example Case 
Presentation:  o 10 yo boy with nephrotic syndrome; 5 year history of relapsing (recurrent) nephrotic syndrome, initially responding to 
steroids (similar to Minimal Change), subsequently became steroid‐dependent and steroid‐resistant; rising serum 
creatinine (shows progression of sclerosis); clinical diagnosis: nephrotic syndrome, clinically recurrent with renal failure 
 Characterized by: 
o nephrotic syndrome 
o higher incidence of hematuria 
o reduced GFR [glomerular filtration rate] 
o hypertension  
o proteinuria (non‐selective) 
 Epidemiology:   adults 35%, most frequent diagnosis in human kidney biopsy 
 children 10%, older children 
 African American, Hispanic patients more affected 
 Etiology: 
o Primary – Idiopathic 
o Secondary to one of the following conditions 
 associated with loss of renal tissue’ superimposed on other glomerular diseases (IgAN) 
 Sickle cell anemia 
 IV drug use (heroin) 
 AIDS 
 morbid obesity 
 Pathogenesis   irreversible injury to podocytes 
 Pathology:   initially only rare glomeruli involved (i.e. focal)      
 With progression lesions occur in some tufts while sparing others  
 Affected glomeruli exhibit  
o increased mesangial matrix 
o obliterated capillary lumina 
o deposition of hyaline masses (hyalinosis) 
o lipid droplets 
 Diagnosis:   nephrotic syndrome 
 genetic testing 
 Prognosis:   progression to renal failure, at least 50% end stage kidney disease within 10 years 
 Treatment:   initially may be steroid‐responsive (mimicking minimal change disease) 
 progressively steroid dependent/resistant  DOES NOT RESPOND TO STEROIDS 
 progressing to end stage kidney disease 
 
 recurrence in transplants 
Mechanism   FSGS: disease progression 
o initially only rare glomeruli involved (i.e. focal) 
o preferentially juxta‐medullary glomeruli involved (sampling!) 
o with disease progression more glomeruli become sclerosed 
o segmental sclerosis gradually becomes more advanced & progressing to global sclerosis 
o secondary tubular atrophy, interstitial fibrosis, end stage kidney 
 FSGS is a common phenotype with a diverse etiology 
o 3 most common forms are:  
Primary FSGS:    “idiopathic” or may be due to a circulating factor cytokine 
Adaptive FSGS:   The adaptive response to nephron is loss associated with… 
o (1) obesity, renal ablation, reflux nephropathy; (2) hypertension, body‐building; (3) low birth weight/premature birth; (4) sickle cell disease; (5) scarring of previously 
active lesions 
APOL1 FSGS:   APOL1 FSGS “risk alleles”‐ genetic risk alleles in apolipoprotein L1 (APOL1) in African Americans; 70% of idiopathic FSGS cases in AA relate to APOL1 
 
o 3 less common forms are:  
FSGS (high‐penetrance):   High‐penetrance mutations in genes encoding slit diaphragm proteins, other  
Medication‐associated FSGS:   IFN, bisphosphonates, anthracycline, heroin 
Viral FSGS:   HIV, parvovirus B19 
 APOL1 FSGS 
o Genetic variants in the APOL1 gene account for a large fraction of the high rates of nondiabetic kidney disease in African Americans   
 APOL1 risk variants have large effects on several different types of kidney disease previously thought to be distinct entities: FSGS, HIV‐
associated nephropathy, severe lupus nephritis, sickle cell nephropathy and unspecified CKD (Chronic Kidney Disease), often previously 
labelled as “hypertensive nephropathy in African Americans”  
 These variants, found only in individuals with recent African ancestry, (<10,000 years) confer enhanced innate immunity against African 
trypanosomes.  
 Where these alleles are nearly absent in populations of European and Asian ancestry 
 The presence of the alleles is not enough to have the phenotype  risk alleles rather than a single‐gene disorder and additional “hits” are 
necessary, which may be due genetic, environmental, or both 
o APOL1 Nephropathy  
 People who have at least 1 copy of either the G1 or G2 APOL1 variant (allele) are resistant to 
infection by trypanosomes (protozoa), but people who have 2 copies of either variant are at an 
increased risk of developing a non‐diabetic kidney disease 
 Genotype may be G1/G1, G2/G2, or the compound heterozygous state of G1/G2 
Microscopy  Light   increased collagenous extracellular matrix that is expanding the mesangium, and subsequently obliterating the capillary 
lumen, or forming adhesions with the Bowman’s capsule 
 Segmental Sclerosis and halinization of the glomeruli 

 
 
 Global Sclerosis 
 

Fluorescence   Often negative, but may be positive for nonspecific focal deposits of IgM, C3, or C1 in the areas of hyalinosis 
 
Electron   Effacement of foot processes similar to minimal change disease 
 
Highly Selective Proteinuria 
 C(IgG)*C(Albumin) or C(IgG)*C(Transferin) is < 0.1 
 Indicates proteinuria is primarily due to defective charge selectivity with intact size selectivity 
 Characteristic of childhood minimal change disease 
Non‐Selective Proteinuria 
 C(IgG)*C(Albumin) or C(IgG)*C(Transferin) is > 0.2 
 Indicates a significant component of abnormal size selectivity 
 Mixed sized and charge selectivity defect with non‐selective proteinuria is usual with most diseases causing glomerular 
proteinuria other than minimal change disease 
 
A 30‐year‐old man with a history of drug  A. Acute glomerulonephritis 
Normal Glomeruli 
addiction presents with a 6‐month history  B. Amyloidosis 
of progressive swelling in his ankles and  C. Crescentic glomerulonephritis 
abdomen.  Urinalysis shows heavy  D. Diffuse proliferative glomerulonephritis 
proteinuria (>4g /24 hours) but no evidence  E. Focal and segmental glomerulosclerosis 
of inflammatory cells or RBCs .  Laboratory 
studies reveal hyperlipidemia and 
hypoalbuminemia.  Serum creatinine level is 
normal.  The blood test for ANCA is negative.  The patient responds well to 
treatment with corticosteroids, but edema and proteinuria recur the following 
year.  The steroid treatment is repeated with the same results.  Upon the third 
recurrence of edema and proteinuria, the patient becomes steroid resistant.  A 
renal biopsy is shown.  Which of the following is the most likely diagnosis for 
this patient’s glomerulopathy?  
 
Nephrotic Syndrome with Hematuria 
 
Membranoproliferative Glomerulonephritis (MPGN) 
(formally called MPGN Type I) 
Clinical   Typical Presentation:   Characterized by: 
o Some patients present with: 
 Hematuria or Proteinuria in the non‐nephrotic range 
 Combined nephrotic‐nephritic presentation 
 Epidemiology:   older children 10% 
 adults 10% 
 Pathogenesis   Primary immune complex formation with in situ immune complex formation 
 Occurs in association with  
o hepatitis B and C antigenemia 
o systemic lupus erythematosus 
o infected atrioventricular shunts 
o extrarenal infections with persistent or episodic antigenemia  
 Pathology:   lobular tufts 
 thick glomerular basement with double contour 
 IgG + complement 
 Diagnosis:   low complement C3  due to significant immune complex formation 
 Prognosis:   progression to renal failure 
 
 Treatment:   treatment of underlying disease 
Mechanism   
Microscopy  Light   Thick glomerular basement membrane  “double contour” 
or “tram track” on silver stain or PAS 
 
 
 
Fluorescence   Granular as a result of immune complex deposition of C3 with IgG, C1q, or C4 
 Nephrotic presentation of SLE 
Electron   electron dense deposits, subendothelial and mesangial 
o mesangial cells are extending/growing into the 
capillary wall chasing the electron dense deposits due 
to it’s phagocytic properties   
 
 

 
Dense‐Deposit Disease (DDD) 
(formally called MPGN Type II or C3 Glomerulonephritis) 
Clinical   Typical   Characterized by: 
Presentation:  o nephrotic syndrome with hematuria 
 Epidemiology:   Rare 
 older children 
 Pathogenesis   sustained activation of complement via alternative pathway (non‐antibody mediated), no antigen‐antibody formation 
 despite strong associations between C3 dysregulation and DDD, the evidence in humans is predominantly 
circumstantial and it is still not clear how the complement abnormality induces the glomerular changes 
 Pathology:   Excessive complement activation – C3 nephritic factor (IgG antibody that stabilizes C3 convertase 
o causes persistent complement activation 
 resulting in a decrease in C3 levels 
 NO immunoglobulin by immunofluorescence 
 dense deposits within lamina densa by electron microscopy 
 Diagnosis:   complement serum C3 levels 
 molecular studies 
 Prognosis:   poor, progression to renal failure 
 recurs more frequently in renal transplants 
 Treatment:   new therapies controlling complement activation: 
 
o Eculizumab, humanized monoclonal antibody functioning as a terminal complement inhibitor 
Mechanism   Note: in patients who show C3 only by immunofluorescence but NO dense deposits by electron microscopy – “C3 glomerulopathy” 
Microscopy  Light 

  
Fluorescence   Granular and linear pattern of C3 
 NO Immunoglobulins 
Electron   dense deposits in lamina densa (red triangle) and the subendothelial space of the GBM  transformed into irregular, “ribbon 
like”, extremely electron‐dense structure 

 
  
 
Systemic Diseases 
 
Diabetic Nepropathy 
Clinical   Typical Presentation:   Characterized by: 
o proteinuria,  
o progressive decline in GFR (glomerular filtration rate) 
o hypertension 
 Epidemiology:   Diabetic nephropathy is the most common cause for ESRD (End Stage Renal Disease) in the USA 
 Pathogenesis   Nonenzymatic glycosylation of the GBM and tubular basement membrane 
o Which increases vessel and tubular cell basement membrane permeability to proteins 
o Produces hyaline arteriolosclerosis and involves the efferent arterioles before the afferent arterioles 
 Pathology:   Diabetic glomerulosclerosis = progressive thickening of glomerular basement membrane and increase in 
mesangial matrix eventually forming KW (Kimmelstiel‐Wilson) nodules 
 Pathologic‐based staging of DKD: 
o Class I = glomerular basement membrane thickening  
o Class II = > 25% expansion of the mesangial space, class III = nodular sclerosis, class IV = global sclerosis 
 Diabetic glomerulosclerosis underlies the clinical syndrome of diabetic nephropathy: 
o Proteinuria 
o Progressive decline in GFR 
o Hypertension  
 overt nephropathy correlates with retinopathy 
o Kidney biopsy NOT done if course typical (only after > 5 years since onset of 
diabetes) 
o Presents with cotton wool spots, flame hemorrhages  
 similar lesions are also caused by 
 alpha‐toxin of Clostridium novyi 
 dot‐blot hemorrhages 
 Diagnosis:   Microalbuminuria – first sign of diabetic glomerulopathy 
o Albumin levels = 1.5 to 8 mg/dL 
o Usually begins 5 to 10 years after poor glycemic control 
 Prognosis:   Diabetic nephropathy in type 2 diabetes can be more difficult to predict because the onset of diabetes is not 
usually well established. Without intervention, 20‐40 percent of patients with type 2 diabetes/microalbuminuria, 
will evolve to macroalbuminuria 
 
 Treatment:   ACE inhibitor 
 thickening of the basement membrane is due to non‐enzymatic glycosylation of the vascular basement 
afferent arteriole efferent arteriole
Mechanism  DIABETES
membrane   nonenzymatic glycosylation of vascular
basement membrane
o resulting in hyaline arteriosclerosis of the efferent arterioles  > efferent arteriole       afferent afferent

 which increases GFR leading to mesangial expansion  high glomerular filtration pressure

hyperfiltration

microalbuminuria

nephrotic syndrome

ACE (angiotensin-converting enzyme) inhibitors – to slow progression

Microscopy  Light   Afferent and efferent hyaline arteriolosclerosis 


o When the afferent arteriole becomes hyalinized, the GFR decreases 

o  
 Nodular masses (Kimmelstiel‐Wilson) develop in the mesangial matrix 
o Nodules due to increased type 4 collagen synthesis and trapped proteins 

o
Fluorescence   Nonspecific 
Electron   The lamina dense shows thickening of the basement membrane Shows fusion of podocytes 

 
  
 
 
 
 
 
 
 
 
 
 
 
Systemic Amyloidosis 
Clinical   Typical Presentation:  o Characterized by:  multisystem 
o nephrotic syndrome, cardiac, peripheral nerve 
o beware of external signs ‐ relatively rare but can be very helpful:  
 macroglossia 
 periorbital purpura 
 submandibular swelling 
 shoulder pad 
 nail lesions 
 Epidemiology:   rare, underdiagnosed 
 Pathogenesis   Underlying mechanism is protein folding disorders ‐ beta pleated sheet 
 But what leads to the disease can be variable  
o plasma cell dyscrasia  AL light chain derived 
o underlying chronic inflammation  AA amyloidosis 
o genetic predisposition  mutation in the amyloidotic protein 
o other/unknown… 
 Pathology:   kidney involvement in 70%, ‐ cardiac, peripheral nerves 
 Diagnosis:   tissue diagnosis: biopsy of an affected organ or a “surrogate” site 
 Prognosis:   amyloid type and stage dependent, delay in diagnosis 
 Treatment:   amyloid protein type dependent 
 aggressive chemotherapy with stem cell rescue 
 treatment of underlying inflammation,  
 
 liver transplantation, pharmacologic clinical trials 
Mechanism   AL – amyloid Light chain (derived from immunoglobulin light chain) 
o Clonal proliferation of plasma cells synthesizing abnormal immunoglobulin 
 plasma cell dyscrasia (small clone, low tumor burden [“small dangerous clones”]) aka “primary”, ca 85% of AL 
 multiple myeloma (overt plasma cell malignancy) 5‐15% 
 derived from immunoglobulin lambda or kappa light chain 
 2/3 lamda > 1/3 kappa 
 Normal is 1/3 lambda and 2/3 kappa 
 intrinsic properties of the light chain 
Microscopy  Light   Routine stain: extracellular “amorphous” deposits, not‐
specific for amyloid 
 

 Need Congo red stain with green birefringence under 
polarized light [“apple green” birefringence] due to 
amyloid deposition in the mesangium 
   
 Amyloid can be detected in subcutaneous fat 
o Fat biopsy typically from periumbilical abdomen for 
diagnosis and screening of patients at risk (i.e. known 
plasma cell dyscrasia)   
 
 
Fluorescence   
Electron   Amyloid is fibrillary (appears as fibers) only by electron 
microscopy 
 
 
 

 
Amyloidosis is a consequence of the deposition of abnormally folded protein, which acquires a beta‐pleated sheet conformation. 
Although many (>32) different proteins have been shown to form deposits of amyloid, all deposits of amyloid share the same 
staining pattern, with an affinity to Congo red dye and a fibrillar ultrastructural appearance. 
 
Among the various amyloidoses, some are localized while other may be either systemic or systemic and/or localized. While only a 
few amyloidosis types are seen in the majority of patients, some amyloidoses are rare or exceedingly rare. In medicine, the most 
important are those which are treatable versus not‐treatable; hereditary amyloidoses also require genetic counselling. 
 
The most common amyloidoses include: AL (amyloid light chain), AA (amyloid A protein) and AB (amyloid Beta protein). The latter, 
associated with Alzheimer’s disease is an example of a localized amyloidosis limited to the central nervous system. It will be 
discussed in some detail during the neuropathology lectures. 
 
Among the various amyloid types, 85% of patients develop amyloidosis as a result of the deposition of monoclonal immunoglobulin 
light chains (AL); AL is associated with an underlying clonal proliferation of plasma cells. These plasma cells produce an abnormal 
immunoglobulin light chain, which circulates in the blood and can form deposits in various organs, typically the kidney, the heart, 
and peripheral nerves. 
 
AA amyloidosis, also referred to as a reactive systemic amyloidosis, is derived from SAA protein (serum amyloid A protein). This 
disease is secondary to an associated, long‐standing, inflammatory condition leading to a sustained elevation of SAA levels. In the 
US, and other developed countries, such diseases include: rheumatoid arthritis and inflammatory bowel disease (Crohn diseases); 
heroin abusers may also be affected. Deposits of amyloid are systemic but target primarily the kidneys, liver and spleen. 
 
AA amyloidosis is a serious complication in a subset of patients affected by autoinflammatory diseases that are secondary to 
mutations in genes (MEFV gene) encoding proteins (pyrin) that are involved in the regulation of innate immunity. The best know 
example of these diseases is Familial Mediterranean fever (FMF). Presents with episodic fever, arthritis, pleuritic, peritonitis. Usually 
occurs in people of Mediterranean origin: Sephardic Jews, Armenians, Arabs, Greeks, Turks, and Italians. Treatment = Colchicine for 
prevention of attacks of fever 
 
Hereditary amyloidoses are individually rare but collectively contribute almost 10% of systemic amyloidoses. In these diseases, the 
amyloid protein itself is affected by a mutation. The most prevalent is Amyloidosis Derived from Transthyretin (ATTR). Interestingly, 
even a normal transthyretin (wild type), is prone to undergo fibrillogenesis in older individuals, mainly males. ATTR affects primarily 
the heart and peripheral nerves.  
 
Amyloid derived from Leukocyte Chemotactic factor 2 (ALect2) has recently emerged as a systemic amyloidosis with predominantly 
renal involvement, affecting predominantly Mexican Americans. Currently, no treatment is available for this disease and it is 
important to avoid its misdiagnosis as AL. 
 
Hemodialysis‐associated amyloidosis: long term hemodialysis; derived from beta2‐macroglobulin (not effectively removed by 
dialysis); periarthritis, carpal tunnel syndrome, tenosynovitis; incidence decreased with newer dialysis membranes; Treatment is 
with kidney transplantation 
 
Localized amyloidosis: forming nodules in the lung, larynx, skin, urinary bladder, mostly AL 
 
Endocrine amyloid 
 
Diagnosis of amyloidosis is based on pathologic examination of tissue. In routinely stained sections, more advanced deposits appear 
amorphous, “hyaline‐like”, and Congo red stain is required for diagnosis. While the kidney is frequently involved, fat biopsy may be 
used for screening patients. 
 
Differential diagnosis of proteinuria/nephrotic syndrome in adults: 
 Focal and Segmental Glomerular Sclerosis/Minimal Change Disease 
 Membranous nephropathy 
 Diabetes 
 Amyloidosis 
 
Other types of amyloidosis include: 
 Cardiac amyloidosis – heart failure, arrhythmia 
 Polyneuropathy – sensory and autonomic disturbances 
 Amyloid deposits are unevenly distributed in tissues 
 
Correct identification of the amyloid protein type is critical since treatments are based on the protein type.  
 For patients with light chain amyloidosis treatment strategies involve aggressive chemotherapy with bone marrow 
transplant.  
 Patients with AA amyloidosis are treated with various anti‐inflammatory drugs.  
 Patients with hereditary amyloidoses have been treated with liver transplantation as a form of a “surgical gene therapy”.  
 However, recently, pharmacologic treatments have emerged and are currently available in clinical trials. 
 
 
 
 
 
 
 
 
 
Lupus Nephritis 
(Systemic Lupus Erythematous [SLE]) 
Clinical   Typical Presentation:   Characterized by: 
o Multisystem – can cause wide variety of patterns of damage to kidney 
o hematuria 
o nephritic syndrome 
o a nephrotic syndrome 
o hypertension 
o renal failure 
 Epidemiology:   primarily young women 
 Pathogenesis   autoimmune: anti DNA, erythrocytes, platelets 
 genetic predisposition: familial, HLA association, other genes 
 environmental triggers 
 Pathology:   kidney involvement in 60‐70% by light microscopy 
 different patterns (class I‐V) 
o Class 4 is most common associated with MPGN which causes the nephritic syndrome 
o 10% of patients develop lupus membranous nephropathy (Class 5) with nephrotic syndrome 
 abundant immune complex deposits 
 Diagnosis:   Kidney biopsy 
 autoantibodies: ANA’s, anti‐Sm, anti‐ dsDNA 
 Prognosis:   kidney involvement significant 
 
 Treatment:   variable level of immunosuppression largely dependent on activity of kidney involvement 
Mechanism   
Microscopy   
Light   Wire loop – thickening of the basement membrane 
 
This disease will be covered more extensively later and will involve separate lectures as well as small groups. Here the discussion is 
limited to the basics of pathologic evaluation. 
1. Typical presentation:  
a. very diverse, multisystem but renal involvement with edema is common 
2. Epidemiology:  
a. primarily young women 
3. Etiology/pathogenesis:  
a. immune DNS‐anti DNA complexes 
4. Pathology:  
a. kidney involvement is seen in 60‐70% of patients by light microscopy and involves different patterns (class I – V) 
most of which are associated with abundant immune complexes and a variable proliferative activity. 
5. Laboratory tests  
a. involve testing for various autoantibodies including ANAs, anti‐Sm, anti‐dsDNA, etc 
6. Prognosis  
a. is largely dependent on the severity of kidney involvement and the response to treatment 
7. Treatments  
a. involve variable levels of immunosuppression, largely dependent on the activity of kidney involvement. Kidney 
biopsy is, therefore, frequently performed to assess the activity and the potential for reversibility of renal 
involvement and, thereby, to guide the intensity of treatment 
 
   
Vascular Diseases 
 
Thrombotic Microangiopathies (TMA) 
(Umbrella term for clinical syndromes with thrombosis, thrombocytopenia, or microangiopathic hemolytic anemia) 
 Typical Presentation:   Characterized by: 
o Typical HUS: 
 Sudden onset after gastrointestinal or flulike prodromal episode including: 
 Bleeding manifestations (hematemesis and melena) 
 Diarrhea 
 Severe oliguria 
 Hematuria 
 Microangiopathic hemolytic anemia (MAHA) 
 Atypical HUS: 
 No diarrhea 
o HUS and TTP have some clinical overlapping features 
 Microangiopathic hemolytic anemia 
 Thrombocytopenia 
o TTP has a dominant involvement of the central nervous system where the kidneys are less commonly involved 
 Epidemiology:   Children 
 Adults 
 Pathogenesis   endothelial injury with platelet activation & aggregation 
o Primary TMA:  
 typical hemolytic‐uremic syndrome (HUS) 
 75% of cases follow intestinal infection with  
o Shiga toxin E. coli (O157:H7) due to ingestion of infected ground meat 
o Shigella dysenteriae type 1 due to the Shiga toxin carried by neutrophils in the circulation 
 Renal glomerular endothelial cells are toxin targets due to expression of the membrane toxin receptor leading to 
o increased adhesion of leukocytes 
o increased endothelin production and loss of endothelial nitric oxide  favoring vasoconstriction 
o endothelial damage  presence of tumor necrosis factor (TNF) 
 Resulting cell death leads to 
o Thrombosis in the glomerular capillaries, afferent arterioles, and interlobular arteries 
o Vasoconstriction resulting in the characteristic thrombotic microangiopathy  
 Clinical studies:  
o recent diarrhea, melena (blood in stool) 
o renal failure,  
o thrombocytopenia,  
o schistocytes (fragmented red blood cells) in peripheral blood smears 
 atypical HUS (aHUS) = complement mediated TMA 
 Unregulated/excessive activation of the alternative complement pathway leading to complement‐mediated injury: 
transformation from low‐grade physiologic activity (“tick‐over”) to unrestrained hyperactivity 
 Triggers: excessive complement activation after minor vascular injuries 
o acquired autoantibodies against complement components 
o inherited abnormalities of complement regulatory proteins 
 Human diseases: glomerular = dense deposit disease/C3 glomerulonephritis 
 Systemic: aHUS 
 thrombotic thrombocytopenic purpura (TTP) 
 platelet aggregation due to von Willebrand factor‐cleaving protease (ADAMTS13) deficiency leading to formation of 
unusually large von Willebrand factor multimers leading to excessive thrombosis 
o absence/decrease of ADAMTS13, the von Willebrand factor cleaving metalloprotease 
o acquired due to autoimmune antibody to ADAMTS13, or  
o hereditary 
o massive platelet thrombi 
o Secondary TMA: 
 Drug‐induced TMA [DITMA] 
 2 mechanisms 
o immune‐mediated reactions and  
o dose‐ or duration‐related toxic reactions 
 Typical History: 
o Patient on chemotherapy, who suddenly develops renal failure +/‐ mental status changes (confusion) 
 Malignant Hypertension and TMA 
 arteries and arterioles with fibrinoid necrosis narrowing the lumen 
 microangiopathic hemolytic anemia due to mechanical injury (shearing) 
 renal function and MAHA usually recover with management of blood pressure  
 Pathology:   widespread thrombosis in small vessels with fibrin thrombi predominantly involving glomeruli 
 similar morphology despite differences in pathogenesis 
 Diagnosis:   thrombocytopenia 
 schistocytes (fragmented red blood cells) in peripheral blood smears 
 ADAMTS13 (TTP) 
 Prognosis:   Serious 
 Treatment:   supportive 
 underlying disease 
 Eculizumab antibodies against complement (C5) in atypical HUS 
 
 plasma exchange to replace ADAMTS13 
Light   Fibrin thrombi in glomeruli and small vessels; endothelial 
injury 
 Schistocytes n peripheral blood smears 
     
 
 

 
 Diseases of blood vessels: several diseases, including benign nephrosclerosis, malignant hypertension, atherosclerosis, renal 
artery stenosis, thromboembolic diseases and vasculitides are covered by separate lectures. In this lecture I will discuss 
thrombotic microangiopathies. 
 Thrombotic microangiopathy (TMA) is an umbrella term for various clinical syndromes with: 
‐ thrombosis (widespread platelet‐rich thrombi) in the microcirculation 
‐ thrombocytopenia (consumption of platelets) 
‐ microangiopathic hemolytic anemia (narrowing of blood vessels by thrombi) 
 Thus the clinical picture consists of: 
‐ microangiopathic hemolytic anemia (MAHA) 
‐ thrombocytopenia 
‐ renal failure 
 Typical clinical presentation of TMA: 
‐ microangiopathic hemolytic anemia, thrombocytopenia, renal failure (some) various clinical syndromes with 
clinical overlap (HUS, TTP) diarrhea in typical HUS [hemolytic uremic syndrome) no diarrhea in atypical HUS, TTP 
(thrombotic thrombocytopenic purpura) 
 Epidemiology:  
‐ children, adults 
 Etiology/pathogenesis: endothelial injury with platelet activation and aggregation 
‐ primary TMA: 
• typical hemolytic‐uremic syndrome (HUS), 75% of cases, infection with E. coli producing Shiga toxin E. coli 
(STEC‐ HUS) atypical HUS (aHUS)‐ uncontrolled complement activation thrombotic thrombocytopenic 
purpura (TTP), platelet aggregation due to von Willebrand factor‐cleaving protease (ADAMTS13) 
deficiency leading to formation of unusually large von Willebrand factor multimers 
‐ secondary TMA: 
• drug toxicities (chemotherapy)  
• malignant hypertension, scleroderma 
• antiphospholipid antibodies (SLE), pregnancy, contraceptives… 
 Pathology:  
‐ widespread thrombosis in small vessels and injury to endothelial cells; similar morphology despite differences in 
pathogenesis 
 Laboratory tests: 
‐ thrombocytopenia 
‐ schistocytes (fragmented red blood cells) in peripheral blood smears 
‐ ADAMTS13 (TTP) 
 Prognosis:  
‐ serious 
 Treatment: 
‐ supportive, 
‐ underlying disease, 
‐ eculizumab antibodies against complement (C5) in atypical HUS 
‐ plasma exchange in TTP to replace ADAMTS13 
 
 
 
 
 
 
 
 
 
Questions 
A 7 yo boy developed malaise followed by bloody diarrhea.  He was previously healthy with a good appetite and he really enjoyed 
his hamburger, which his father bought him from a street stand…His mother noticed that his urine turned red and was also 
diminished in volume…His kidney biopsy will show: 
A. subepithelial hump‐like deposits 
B. IgA deposits in glomeruli 
C. Crescents in >50% of glomeruli 
D. IgA deposits in glomeruli and skin capillaries  
E. Thrombi in glomerular capillaries 
 
Clinical studies will most likely show: 
A. renal failure,  
B. thrombocytopenia,  
C. schistocytes (fragmented red blood cells) in peripheral blood smears  
D. all of the above (a‐c) 
E. crescents in the glomeruli 
 
A 6‐year‐old boy presented with upper respiratory tract infection. After two days, he developed an erythematous, nonpruritic rash 
that progressed proximally from both feet to thighs and upper extremities and subsequently abdominal pain associated with 
melena. On physical examination, there was pharyngeal erythema, and nontender, nonblanching purpuric rash involving both upper 
and lower extremities with a mild pedal edema. 
Laboratory tests showed  
 mild leukocytosis (WBC: 10,900/microL) 
 macroscopic hematuria on urinalysis 
 Hb: 13.5 g/dL [N: 11.5‐15.0] 
 Hct: 41.2%; [N:35.0‐45.0] 
 serum Creatinine: 0.9 mg/dL[N: 0.5‐1.0mg/dL] 
 stool for occult blood: positive 
 
His kidney biopsy is likely to show: 
A. Thrombi in glomeruli 
B. Basket weave on electron microscopy 
C. Postinfecttious glomerulonephritis with “humps” 
D. Thrombi in glomeruli associated with positivity for IgA 
E. Mesangial IgA deposits 
Diseases Affecting Tubules and Interstitium 
 
Tubulointerstitial Nephritis (TIN) 
 
Tubulo‐interstitial nephritis   refers to a group of inflammatory diseases of the kidneys that primarily involve the interstitium AND tubules.  
o The glomeruli may be spared altogether or affected only late in the course 
 Two types of Tubulo‐interstitial nephritis 
o Infectious, the most common diseases of the kidney caused by bacterial infection where the renal pelvis is prominently 
involved – where the more descriptive term pyelonephritis (of the pelvis) is used. 
o Interstitial nephritis is used for TIN that are Non‐infectious (nonbacterial) in origin including 
 Drug and toxin‐induced (second most common) 
 tubular injury resulting 
 Metabolic disorders – hypokalemia, urate, oxalate 
 Initially only the tubules are affected associated with acute uric acid nephropathy and light chain cast nephropathy, where 
with progression of the disease there is also involvement of the interstitium 
 Physical injury – irradiation 
 Viral infections – polyomavirus nephritis 
 Immune reactions 
 Neoplasms – multiple myeloma 
 
Acute Pyelonephritis 
Overview   Acute pyelonephritis, a common suppurative inflammation of the kidney and the renal pelvis caused by bacterial infection.  
o An important manifestation of urinary tract infection (UTI) 
 which can involves  
 Lower urinary tract cystitis, prostatitis, urethritis 
 Upper urinary tract  pyelonephritis 
 or both 
o The majority of cases of pyelonephritis are associated with infection of the lower urinary tract. 
 Which may remain localized without extending to involving the kidney.  
 Which makes UTIs an extremely common clinical problem. 
Pathogenesis   The principal causative organism are the enteric gram‐negative rods 
o Escherichia coli (the most common) 
o Other important organisms that are associated with recurrent infections include: 
 Proteus, Klebsiella, Enterobacter, and Pseudomonas; 
o Rare organisms include: 
 Staphylococci and Streptococcus faecalis 
 Route of Progression: Bacteria reach the kidney from the: 
o lower urinary tract  ascending infection 
 this is the most important and most common route bacteria reach the kidney 
o bloodstream  hematogenous infection 
 Pyelonephritis may result from seeding of the kidneys by bacteria in the course of septicemia or infective endocarditis 
   Bacteria can adhere to mucosal surfaces  Followed by colonization of the distal urethra (introitus 
in females) OR use fimbriae to attach to the surface urothelial cells 
 Organisms then reach the bladder through colony growth and by moving against the flow of urine 
o Which may occur during urethral instrumentation  including catheterization and cystoscopy 
 In the absence of instrumentation, UTIs most commonly affect females 
 due to the close proximity of the female urethra to the rectum favoring the colonization by 
enteric bacteria 
o Infections can also be caused by: 
 Outflow obstruction or Bladder dysfunction  causes the natural defense mechanisms of the 
bladder to be overwhelmed and allows for developing a UTI 
 Stasis – allows the bacteria to be introduced into the bladder and multiply without being 
flushed out or destroyed by the bladder wall (bladder mucosa has antimicrobial properties) 
 Bacteria can then ascend along the ureters to infect the renal pelvis and parenchyma 
o UTIs are frequent with urinary tract obstruction, benign prostatic hyperplasia, and uterine 
prolapse 
 Diabetics are also at an increased risk for UTIs  
o due to the increased susceptibility to infection and neurogenic bladder dysfunction 
Vesicoureteral Reflux (VUR)   Define = associated with incompetence of the vesicoureteral orifice  
o Which is an important cause of ascending infections 
 The reflux allows bacteria to ascend the ureter into the pelvis 
 Etiology: 
o Present in 20‐40% of young children with a UTI 
 Congenital defect that results in incompetence of the ureterovesical valve  
o Acquired in patients with a flaccid bladder due to spinal cord injury or with neurogenic bladder dysfunction secondary to 
diabetes 
 Pathogenesis: 
o VUR results in residual urine within the urinary tract after voiding  favoring bacterial growth 
o Intrarenal Reflux  occurs when infected bladder urine can be propelled up to the renal pelvis and into the renal parenchyma 
through open ducts at the tips of the papillae  
Morphology   Gross Features: 
o One or both kidneys can be involved  
o The affected kidney may be normal in size or enlarged 
o Characterized by: 
 Discrete, yellowish, raised abscesses which are grossly apparent on the renal surface 
Acute Pyelonephritis is   In the early stages, pus formation (patchy interstitial 
characterized by  suppurative inflammation) is limited to the interstitial tissue 
liquefactive necrosis with  o but later abscesses rupture into tubules.  
abscess formation within   Where large masses of intratubular neutrophils (PMN), 
the renal parenchyma  giving rise to the characteristic white cell casts found in   
  the urine.  
 Typically, the glomeruli are not affected.  NO BIOPSY 
NEEDED! 
 When obstruction is prominent, the pus may not drain and 
then fills the renal pelvis, calyces, and ureter, producing 
pyonephrosis. 
Papillary Necrosis is a   With three predisposing conditions:    
second form of  o Diabetes, UTI, Analgesic abuse, sickle cell anemia, 
pyelonephritis where  obstruction, and tuberculosis 
necrosis affects the renal   This lesion consists of a combination of ischemic and 
papillae  suppurative necrosis of the tips of the renal pyramids (renal 
  papillae).  
   The pathognomonic gross feature of papillary necrosis is 
sharply defined gray‐white to yellow necrosis of the apical 
two thirds of the pyramids.  
 One papilla or several or all papillae may be affected.  
 Microscopically, the papillary tips show characteristic 
coagulative necrosis, with surrounding neutrophilic infiltrate 
Acute or Chronic Cystitis –   In long‐standing cases associated with obstruction,    
inflammation of the  o the bladder may be grossly hypertrophic with trabeculation 
bladder  of its walls 
  o The bladder may be thinned and markedly distended from 
retention of urine. 
Polyomavirus nephritis –   Polyoma virus (arrows) nephropathy 
viral pyelonephritis   Occurs in renal transplants (5%) 
 Can mimics rejection (blue arrows) 
   
 Diagnosis: immuno stain, brown nuclei 

 Electron microscopy – inclusions 
 

Predisposing Conditions  Urinary obstruction   either congenital or acquired 


Instrumentation   of the urinary tract, most commonly catheterization 
Vesicoureteral reflux   
Pregnancy   4% to 6% of pregnant women develop bacteriuria sometime during pregnancy, and 20% to 40% 
of these eventually develop symptomatic urinary infection if not treated. 
Female gender and   After the first year of life (an age by which congenital anomalies in males commonly become 
patient age  evident) and up to approximate age 40 years, infections are much more frequent in females. 
With increasing age, the incidence in males rises as a result of the development of prostatic 
hyperplasia, which causes urinary outflow obstruction. 
Preexisting renal lesions   causing intrarenal scarring and obstruction 
Diabetes mellitus   in which common predisposing factors are infection and bladder dysfunction 
Immunosuppression and   
 
immunodeficiency 
Clinical   The onset of uncomplicated acute pyelonephritis usually is sudden 
o with pain at the costovertebral angle and evidence of systemic infection 
 such as chills, fever, and malaise 
 localizing urinary tract signs of dysuria, frequency, and urgency 
 The urine appears turgid  
o due to the contained pus (pyuria).  
 Even without antibiotic treatment  the disease tends to be benign and self‐limited.  
 The symptomatic phase of the disease typically lasts no longer than a week 
o although bacteriuria may persist much longer.  
 The disease usually is unilateral 
o Where affected persons do not develop renal failure because they still have one unaffected kidney.  
 In cases in which predisposing factors are present: 
o the disease may become recurrent or chronic, particularly when involvement is bilateral.  
 The development of papillary necrosis is associated with a much poorer prognosis. 
Chronic Pyelonephritis and Reflux Nephropathy 
Overview   Defined = interstitial inflammation and scarring of the renal parenchyma is associated with 
grossly visible scarring and deformity of the pelvicalyceal system.  
 onset insidious, scarring (irreversible) 
 gradual renal insufficiency  
 routine u/a: pyuria/proteinuria 
 loss of concentrating ability:  
o polyuria 
o nocturia 
 X‐ray – contracted kidneys/deformed calyceal system 
 It can be divided into two forms:  
o chronic obstructive pyelonephritis 
o chronic reflux–associated pyelonephritis – hypertension in children 
Chronic Obstructive   Obstruction predisposes the kidney to infection.  
Pyelonephritis   Recurrent infections associated with diffuse or localized obstructive lesions  
o lead to recurrent renal inflammation and scarring 
 which eventually cause chronic pyelonephritis 
 Presentation: 
o Bilateral – associated with congenital anomalies of the urethra (posterior urethral valves) 
 resulting in fatal renal insufficiency  
o Unilateral – associated with calculi and unilateral obstructive lesions of the ureter.  
o Mimic malignancy  Xanthogranulomatous pyelonephritis 
 The underlying pathogenetic mechanism in XGP is renal outflow obstruction (caused by Staghorn calculi due 
to Proteus) in the setting of suppurative inflammation. Clinical presentation, radiographic features, and 
gross appearance of XGP may closely mimic a renal neoplasm making a correct pre‐operative diagnosis difficult. 
Chronic Reflux‐Associated   Most more common form of chronic pyelonephritic scarring 
Pyelonephritis (Reflux  o Due to a UTI in addition to congenital vesicoureteral reflux and intrarenal reflux 
Nephropathy)   Reflux may be unilateral or bilateral 
o Where the resulting renal damage may either cause  
 scarring and atrophy of one kidney or may involve both  which can lead to chronic renal insufficiency. 
Morphology   One or both kidneys may be involved, either diffusely or in patches.  
 Even when involvement is bilateral, the kidneys are not equally damaged and therefore are not equally contracted.  
o This uneven scarring is useful in differentiating chronic pyelonephritis from the more symmetrically 
contracted kidneys associated with vascular sclerosis (often referred to as “benign nephrosclerosis”) and 
chronic glomerulonephritis.  
 The hallmark of chronic pyelonephritis is scarring involving the pelvis or calyces, or both 
o leading to papillary blunting and marked calyceal deformities.  
 The microscopic changes are largely nonspecific, and similar alterations may be seen with other chronic 
tubulointerstitial disorders such as analgesic nephropathy.  
 The parenchyma shows the following features: 
o Uneven interstitial fibrosis and an inflammatory infiltrate of lymphocytes, plasma cells, and occasionally neutrophils 

  
o Dilation or contraction of tubules, with atrophy of the lining epithelium.  
 Many of the dilated tubules contain pink to blue, glassy‐appearing PAS‐positive casts, known as colloid casts 
 Which suggest the appearance of thyroid tissue  called thyroidization 

  
 Often, neutrophils are seen within tubules. 

  
o Chronic inflammatory cell infiltration and fibrosis involving the calyceal mucosa and wall 
o Arteriolosclerosis caused by the frequently associated hypertension 
o Glomerulosclerosis that usually develops as a secondary process caused by nephron loss (a maladaptation discussed earlier). 
Clinical   Many patients with chronic pyelonephritis seek medical attention relatively late in the course of the disease due to the  
o gradual onset of renal insufficiency 
o signs of kidney disease are noticed on routine laboratory tests.  
o With the development of hypertension.  
 The radiologic image is characteristic:  
o The affected kidney is asymmetrically contracted, with some degree of blunting and deformity of the calyceal system 
(caliectasis).  
o If the disease is bilateral and progressive, tubular dysfunction occurs with loss of concentrating ability, manifested by 
polyuria and nocturia. 
 Some patients ultimately develop secondary glomerulosclerosis, associated with proteinuria 
o eventually, these injuries all contribute to progressive chronic kidney disease. 
 
Drug‐Induced Interstitial Nephritis 
Non‐Infectious   Acute drug‐induced tubulointerstitial nephritis (TIN) occurs as an adverse reaction to any of an increasing number of drugs.  
Tubulointerstitial Nephritis   Acute drug‐induced TIN is associated most frequently with  
o synthetic penicillins (methicillin, ampicillin) 
o synthetic antibiotics (rifampin) 
o diuretics (thiazides) 
o nonsteroidal anti‐inflammatory agents 
o numerous other drugs (phenindione, cimetidine). 
Pathogenesis   Many features of the disease suggest an immune mechanism.  
 Clinical evidence of hypersensitivity includes  
o latent period 
o eosinophilia and rash 
o the idiosyncratic nature of the drug reaction (i.e., the lack of dose dependency) 
o recurrence of hypersensitivity after re‐exposure to the same drug or others that are similar in structure.  
 Serum IgE levels are increased  type I hypersensitivity 
 Presence of positive skin tests to drugs  a T cell–mediated (type IV) hypersensitivity reaction. 
 The most likely sequence of pathogenic events is as follows:  
o The drugs act as haptens that, during secretion by tubules, covalently bind to some cytoplasmic or extracellular component of 
tubular cells and become immunogenic.  
o The resultant tubulointerstitial injury is then caused by IgE‐ and cell mediated immune reactions to tubular cells or their 
basement membranes. 
Morphology   The abnormalities in acute drug‐induced nephritis are in the interstitium 
o which shows pronounced edema and infiltration by mononuclear cells, principally lymphocytes and 
macrophages  
o Eosinophils and neutrophils may be present, often in large numbers.  
 With some drugs (e.g., methicillin, thiazides, rifampin), interstitial non‐necrotizing granulomas with 
giant cells may be seen.  
 The glomeruli are normal except in some cases caused by nonsteroidal anti‐inflammatory agents, in 
which the hypersensitivity reaction also leads to podocyte foot process effacement and the nephrotic 
syndrome. 
Clinical   The disease begins about 15 days (range, 2 to 40 days) after exposure to the drug 
 characterized by  
o fever 
o eosinophilia (which may be transient or in the urine) 
o a rash (in about 25% of persons) 
o renal abnormalities/failure 
 Urinary findings include  
o Hematuria 
o minimal or no proteinuria 
o leukocyturia (sometimes including eosinophils) 
o Acute renal failure – associated with an elevated serum creatinine with oliguria develops in about 50% of cases, particularly in 
older patients.  
 Clinical recognition of drug‐induced kidney injury is imperative 
o because withdrawal of the offending drug is followed by recovery 
Typical Presentation   A 29 yo man has developed a fever and skin rash over the past 3 days 
 Five days later he had increasing malaise and sought medical attention. 
 PE: maculopapular erythematous rash on his trunk was nearly faded away. 
 Vitals: 
o Temp = 37.1 °C 
o BP = 135/85 mm Hg 
o Serum creatinine: 2.8 mg/dL 
o UA: 2+ proteinuria, 1+ hematuria, glucose (‐), ketones (‐), nitrite (‐) 
o Urine sediment showed RBCs and WBCs, some of which were eosinophils… 
 Hx:  
o recent treatment with antibiotic (ampicillin) for a sore throat… 
 Clinical diagnosis:  
o suspect acute drug‐induced interstitial nephritis 
o Immune mechanism: Latent period, eosinophilia & rash 
o Idiosyncratic nature of the drug reaction (i.e. the lack of dose dependency) 
o Recurrence of hypersensitivity after re‐exposure to the same/similar drug 
Over the Counter  NSAIDS: 
Medications   Acute hemodynamic (inhibition of prostaglandin synthesis)  
 Acute hypersensitivity interstitial nephritis 
 Clinical: renal failure + nephrotic syndrome  where renal failure is due to interstitial nephritis 
 Pathology: 
o interstitial nephritis 
o minimal change disease‐like 
Chinese herb nephropathy:  
 Aristolochic acid = potent nephrotoxin 
o rapidly progressive interstitial fibrosis and end‐stage renal disease [ESRD] 
o Presentation of ESRD is a young women using a Chinese herb as part of a slimming regimen 
Metabolic‐Induced Interstitial Nephritis 
Urate   acute, predominantly tubular component, chronic – also interstitial 
 Acute uric acid nephropathy: 
o leukemia/lymphoma on chemotherapy (tumor lysis syndrome) 
 Chronic urate (gouty) nephropathy  
o tophi: urate crystals + inflammatory reaction 
Frozen section – polarized light 

Nephrolithiasis ‐ stones 

*Outline of urate crystal 

Oxalate   Acute oxalate nephropathy 
o primary (hereditary) hyperoxaluria  
o ethylene glycol (antifreeze) intoxication (acute)  
o enteric hyperoxaluria  
o exposure to the anesthetic agent methoxyflurane 
o pyridoxine (vitamin B6) deficiency 
o excessive ingestion of vitamin C, diet rich in oxalic acid (rhubarb, cocoa, parsley, nuts) 
 Chronic oxalate nephropathy may occur after bariatric surgery (slimming surgery causing malabsorption) 
o Associated with primary hyperoxaluria 
calcium oxalate crystals: translucent crystals of different 
shapes, predominantly intraluminal, under polarized light 
strongly birefringent , fan‐like, sheaf‐like, or irregular shapes 
 
 
Hypercalcemia   Causes bone resorption 
 
Neoplasm ‐Induced Interstitial Nephritis 
Multiple Myeloma   Uric acid (therapy) 
 Hypercalcemia –  bone resorption  
 Light chain casts: monoclonal  
o Free light chains produced by malignant plasma cells are circulating in blood and filtered into urine, 
subsequently precipitate in distal tubules as intratubular light chain casts which causes RENAL FAILURE!!! 
 
 
A 32 yo computer programmer presents with generalized edema and unintentional  A. His biopsy will show crescents 
weight gain.  In general he has been healthy.  Only recently he complained of a lower  B. His biopsy will look normal by light microscopy 
back pain during a period requiring long hours at the computer…  However, the pain was  C. His biopsy will show subepithelial deposits 
relieved with some over the counter meds. His urinalysis shows 3+ protein, his serum  D. He most likely has interstitial nephritis and minimal 
creatinine is 5.0 MG/DL [n=0.7‐1.5].  You are suspecting that …  change disease 
E. e. He has postinfectious glomerulonephritis   
A 26 yo female is discovered to have elevated serum creatinine level. She has been  A. You suspect cancer as a cause of her renal failure
healthy, well nourished. In fact she has been trying to shed some weight but with no  B. You ask her about all her prescription medications 
great success…  C. She probably has reflux nephropathy 
D. She has acute pyelonephritis 
E. You ask her about her about herbal preparations 
 
 
 
 
 
 
 
 
 
 
 
 
 
 
 
 
Acute Tubular Injury (ATI) 
Clinical   Typical   Characterized by:  
Presentation:  o Oliguria with elevation of BUN and Creatinine 
o Metabolic acidosis 
o Hyperkalemia 
o UA: Dirty brown granular casts and epithelial 
casts 
 Two Major Types of ATI 
o Ischemic Acute Tubular Necrosis (ATN): 
 Most common cause of ATI 
 Due to a decrease blood flow caused by  
 severe hemorrhage 
 severe renal vasoconstriction 
 hypotension 
 dehydration 
 shock 
 acute pancreatitis 
o Nephrotoxic Acute Tubular Necrosis (ATN): 
 Large number of causes including: 
 Drugs – Polymyxin, methicillin, gentamicin, sulfonamides 
 Radiographic contrast dye agents 
 Heavy metals – mercury, lead, and gold 
 Organic solvents – carbon tetrachloride, Chloroform, Methyl alcohol 
 Ethylene glycol – antifreeze poisoning causes calcium oxalate crystals 
to precipitate in the tubules (seen on polarized light)  
o Ethylene glycol is a colorless, odorless, sweet liquid, found in 
antifreeze.  It may be drunk accidentally or purposefully. When 
broken down by the body to glycolic and oxalic acid. Calcium 
oxalate crystals may be seen in the urine acidosis or an increased 
osmol gap in the blood 
 Mushroom poisoning 
 Phenol 
 Pesticides 
 Myoglobin 
 Epidemiology:   most common cause of acute renal failure 
 Pathogenesis   ischemic tubular injury and toxic injury  
o necrosis of segments of the tubules (typically proximal tubules) 
o FOCAL 
o proteinaceous casts in distal tubules 
o interstitial edema 
o disturbances in blood flow (intra‐renal vasoconstriction) 
o reduced GFR (glomerular filtration rate) 
o diminished delivery of oxygen/nutrients to tubular epithelial cells 
 Pathology:   tubular necrosis, exfoliation…regeneration 
Normal tubules (left) versus ischemic tubules 
(right) 

 
 
 Diagnosis:   renal failure 
 diagnosis is usually based on clinical grounds 
 Prognosis:   reversible 
 Excellent prognosis if the patient survives the underling disease and if the patient had no preexisting 
kidney disease 
 
 Treatment:   Supportive 
ATI Typical Presentation   A 20 yo college student was involved in a motorcycle accident with acute loss of blood upon arrival of paramedics, his BP was 
low and after stabilization of acute bleeding he was transported to a hospital, where he received a transfusion of 3 units of 
packed RBCs.  Over the next week his serum creatinine increased to 4 mg/dL and his urinary output decreased   
 Clinical diagnosis: 
o acute kidney injury (AKI) due to acute tubular injury (ATI), primarily ischemic 
o He underwent hemodialysis for the next 2 weeks and subsequently developed marked polyuria with urinary output close to 3 
L/day.   
o His renal function gradually returned to normal. 
 
 
 
 
 
 
 
 
Combined Acute Tubular Injury (ATI) 
ATI Combined   Combined (Ischemic and Nephrotoxic) 
o mismatched blood transfusion  
o hemolytic crises (hemoglobinuria) 
o skeletal muscle injury (myoglobinuria) 
o intratubular casts, crystals 
o frequently also interstitial component as well 
ATI Combined Typical   College student involved in a motorcycle accident severe blunt trauma to the abdomen and extremities oliguria and dark brown 
Presentation  urine over 3 days…Urine dipstick analysis positive for blood. Urine microscopic urinalysis negative for RBCs. BUN increased to 38 
mg/dL… 
 Clinical diagnosis: 
o acute tubular injury (ATI) toxic and ischemic 
o dialysis for the next 3 weeks improvement 
o Urinary output >3 L/24 hrs for 1 week before BUN normalized 
 Explanation: 
o This patient sustained muscle crash injury that resulted in myoglobinemia and myoglobinuria. 
o The large amount of excreted myoglobin was toxic to tubules and this patient developed acute tubular injury (ATI) 
o With supportive care, the tubular epithelium can regenerate, and renal function can be restored.   
o During the recovery phase there is polyuria because the glomerular filtrate cannot be adequately reabsorbed by the damaged 
tubular epithelium 
 
Urolithiasis (Urinary Tract Obstruction) 
Renal Calculi   Occurs in up to 6% of the population 
 Men are affected more often than women 
Stone Composition   Most stones (75%) are calcium oxalate stones 
 Magnesium ammonium phosphate (struvite) stones  
o associated with infection by urea‐splitting bacteria (Proteus) 
o often form large staghorn calculi 

o  
 Uric acid stones are seen in gout, leukemia, acidic urine 
 Cystine stones are uncommon 
Pathology   Most stones are unilateral stones that are formed in the calyx, pelvis, and urinary bladder 
Clinical features   Calcium stones are radiopaque and can be seen on x‐ray 
 Renal colic may occur if small stones pass into the ureters 
 Stones may cause: 
o Hematuria 
o Urinary obstruction 
o Predispose to infection 
Treatment of Stones   Is with lithotripsy or endoscopic removal 
 
Content  Precipitates with  Frequency  Age  Radiology  Shape  Causes  Treatment 
Calcium  Calcium oxalate =  Type 1:  Adults and  Opaque  Envelope or dumbbell Hypocitraturia often associated with a decrease in  Thiazides, citrate, low‐
hypocitraturia   80%  children  urine pH  sodium diet 
Can result from ethylene glycol ingestion, vitamin C 
abuse, hypocitraturia, Malabsorption (Crohn’s) 

Calcium phosphate  Wedge‐shaped prism  Low‐sodium diet, thiazides 


= High pH (Basic) 
Magnesium,  High pH (Basic)  Type 2:  Adults  Opaque  Coffin lid Caused by infection with urease positive bacteria  Treat underlying infection, 
Ammonium,  10%  (Proteus mirabilis, Staphylococcus saprophyticus,  surgery to remove stone 
Phosphate   Klebsiella) that hydrolyze urea to ammonia  causes 
(struvite)  urine alkalinization. 
Commonly form staghorn calculi 

Uric acid  Low pH (Acidic)  Type 3: 6‐ Adults  Radio  Rhomboid or rosettes Risk factors: decrease in urine volume, arid climates,  Alkalinization of urine, 


9%  lucent  acidic pH  allopurinol 
Strong association with hyperuricemia (gout). 
Often seen in diseases with increased cell turn over 
(leukemia) 

Cystine  Low pH (Acidic)  Rare: 1‐2%  Children  Midly  Hexagonal  Hereditary (autosomal recessive) condition in which  Low‐sodium diet, 


opaque  Cystine‐reabsorbing PCT transporter loses function,  alkalinization of urine, 
causing cystinuria.  chelating agents if 
Transporter defect also results in poor reabsorption of  refractory 
Ornithine, Lysine, and Arginine (COLA). 
  Cystine is poorly soluble  stones form in urine. 
Usually begins in childhood. 
Can form staghorn calculi. 
Sodium cyanide nitroprusside test is positive 

 
 
 
Presentation of Renal Calculi 
Population  Increase incidence in males >40 yo 
Signs and Symptoms  Nausea and vomiting; Pain radiates to flank area; Sudden onset of severe flank 
pain that radiates to the back; the pain maybe intermittent depending on stone 
movement 
Risk Factors  Infection; Urinary stasis; Immobility; Hypercalcemia; increased uric acid; 
increased urinary oxalate level 
Diagnosis:  Urinalysis (10–15 red blood cells per high power field); cystoscopy; X‐ray; stone 
analysis; Blood test (calcium, oxalate, uric acid) 
 
A retired dentist developed a sudden onset of a severe flank pain radiating to  A. Nephrolithiasis 
the back with nausea and vomiting. Urinalysis showed 10–15 red blood cells  B. Acute tubulointerstitial nephritis 
per high power field. He denied any history of alcohol or recreational drug use.  C. Acute tubular injury 
His past medical history includes type 2 diabetes and hypertension. What  D. Acute pyelonephritis   
should be considered in the differential diagnosis? 
Patient described in a previous case passed spontaneously a stone. What  A. Send stone to pathology for chemical analysis 
would be the best choice?  B. Send stone to pathology for photographic analysis 
C. Most likely atheromatous stone 
D. Give back stone to patient for safe keeping 
E. Discard the stone if patient is not interested in keeping it   
 
 
Acute Kidney Injury 
 
Technical Definition: AKI   The KDIGO guidelines define AKI as follows: 
o Increase in serum creatinine by ≥0.3 mg/dL within 48 hours 
OR 
o Increase in serum creatinine to ≥1.5 times baseline, which occurred within the prior seven days 
OR 
o Urine volume <0.5 mL/kg/hour for six hours 
 The diagnostic criteria should only be applied after volume status has been optimized. 
 Urinary tract obstruction needs to be excluded if urine volume is used as a sole criteria. 
Staging System of AKI   Based on Rifle Criteria 
Stage  Creatinine  Urine Output 
1. Risk   Increase in serum creatinine 1.5‐1.9 times baseline OR >0.3 mg/dL   Reduction in urine output to <0.5 
mL/kg/hr for 6‐12 hours 
2. Injury   Increase in serum creatinine 2.0‐2.9 times baseline   Reduction in urine output to <0.5 
mL/kg/hr for >12 hours 
3.   Increase in serum creatinine 3.0 times baseline OR >4.0 mg/dL   Reduction in urine output to <0.3 
Failure   The initiation of renal replacement therapy  mL/kg/hour for >24 hours 
 In patients < 18 years old with a decrease in the estimated   Anuria for >12 hours 
 
glomerular filtration rate to <35 mL/min/1.73 m^2 
Causes of AKI  Prerenal 
 Due to decrease in RBF (Hypotension) 
o Which causes a decrease in GFR 
 This allows sodium, water, and BUN are retained by the kidney in an attempt to conserve volume 
  Which increases the BUN/creatinine ratio (BUN is reabsorbed where creatinine is not) and decreases the FeNa 
Intrarenal 
 Due to acute tubular necrosis or ischemia/toxins 
 Less commonly due to Acute Glomerulonephritis (associated with Rapidly Progressive Glomerulonephritis, Hemolytic Uremic 
Syndrome) or Acute Interstitial Nephritis 
 In Acute Tubular Necrosis, patchy necrosis 
o Causes debris to obstruct the tubule and fluid to backflow across the necrotic tubule 
 Causing a decrease in GFR 
o Urine has epithelial/granular casts 
o BUN reabsorption is impaired 
 Which causes a decrease in the BUN/creatinine ratio and an increase in FeNa 
Postrenal 
 Due to an outflow obstruction (stones, BPH, neoplasia, congenital anomalies). 
 Develops only with bilateral obstruction 
  Prerenal  Intrarenal  Postrenal 
Urine Osmolality (mOsm/kg)  >500  <350  <350 
Urine Sodium (mEq/L)  <20  >40  >40 
FeNa  <1%  >2%  <1% (mild); >2% (severe) 
 
Serum BUN/Cr  >20 <15 Varies
Prerenal Causes   Absolute decrease in Effective Circulating Volume (ECV) 
o Due to volume depletion or hemorrhage 
 Relative decrease in Effective Circulating Volume 
o Due to heart failure or Cirrhosis 
 Impaired renal autoregulation with low Effective Circulating Volume 
o NSAIDS or ACE inhibitors or ARB 
 Vasoconstriction/Occlusion 
o Hypercalcemia, Calcineurin inhibitors, or Renal artery stenosis 
Normal Renal   The kidneys are very effective in autoregulating blood flow. 
Autoregulation  o Primarily due to changes in the resistance of the afferent arterioles associated with 2 mechanisms 
 Myogenic response: 
 The intrinsic property of smooth muscle to contract when stretched  
 Tubuloglomerular Feedback: 
 Increased Mean Arterial Pressure leads to an increase in RBF and an increase in GFR 
 High delivery of sodium ions to the macula densa  Cause the secretion of Adenosine and ATP  Causes vasoconstriction 
of the afferent arterioles  Which leads to a decrease in renal blood flow and a decrease in GFR 
 Low delivery of sodium ions to the macula densa  Causes the arterioles to dilate  Causes an increase in renal blood 
flow and increase in GFR 
Renal Homeostasis   Arteriolar Resistance 
Effect  GFR  RBF/RPF 
Afferent arteriole constriction  Decrease  Decrease 
Efferent arteriole constriction  Increase  Decrease 
Afferent arteriole dilates  Increase  Increase 
Efferent arteriole dilates  Decrease  Increase 
Increase plasma protein concentration  Decrease  No Change 
Decrease plasma protein concentration  Increase  No Change 
Constriction of the Ureter  Decrease  No Change 
 
Dehydration  Decrease  Big Decrease 
Effects of Prostaglandins   Prostaglandins mediate vasodilation of the afferent arterioles 
and Renin/Angiotensin  o Which modulates the vasoconstriction from the sympathetic nervous system 
 Renin/Angiotensin mediates vasoconstriction of the efferent arterioles 
Low Effective Circulating   NSAIDs inhibit the synthesis of prostaglandins and interferes with the protective effects that opposes vasoconstriction from the 
Volume  sympathetic nervous system 
o Leads to a decrease in RPF and in GFR which could cause renal failure 
 ACE Inhibitors and ARBs are used for diabetic nephropathy because they lead to a reduction in glomerular capillary pressure and 
reduced damage/fibrosis of the glomuli 
o  Leads to vasodilation of the efferent arterioles  Causes a decrease in glomerular capillary pressure and a decrease in GFR  
Causes an increase in RPF due to the decrease in resistance to flow 
Prerenal Azotemia   Appropriate physiological response to renal hypo‐perfusion. 
o Acute renal “success”, not failure. 
 Can complicate any clinical scenario with decreased effective circulating volume (true or relative). 
 Generally, reversible with treating the underlying etiology. 
 If unable to correct in a timely manner, can progress to ischemic ATN. 
 Prerenal azotemic and ischemic ATN thought to cause up to 75% of AKI in the hospitalized setting. 
Post‐renal Failure   Upper Urinary Tract Obstruction: 
o Causes: Kidney stones, retroperitoneal fibrosis, 
o Extrinsic tumor compression of the ureters bilaterally 
o Bilateral obstruction with superimposed infection is a medical emergency. 
 Lower Urinary Tract Obstruction: 
o Most common in‐hospital cause of obstruction.  
o Pain meds are common culprits. 
o Enlarged prostate most common cause in general. 
 Diagnosis:  
o History: decreased urine output (not always reliable) 
o Physical exam: suprapubic fullness 
o Ultrasound: hydronephrosis (upper urinary tract obstruction) or enlarged bladder (lower urinary tract obstruction) 
o Bladder scan to check for post‐void residual 
 Treatment:  
o Relieve the obstruction 
o Need to monitor for post‐obstructive diuresis 
o Urine output may be > 5‐6 liters 
o Need to monitor for hypernatremia 
 
Intrinsic Renal Failure 
Intrinsic Renal Failure   Can affect difference areas of the kidney: 
o Tubules: Acute Tubular Necrosis 
o Glomerulus: Acute Glomerulonephritis 
o Interstitium: Acute Interstitial Nephritis 
o Blood vessel: TTP, Eclampsia, Malignant HTN 
o Collecting System: Acute Pyelonephritis 
 
Tubules: Acute Tubular Necrosis (ATN) 
Acute Tubular Necrosis   Characterized as Toxic or Ischemic 
  o Ischemic: 
 Leads to a prolonged prerenal state 
o Toxic: 
 Are separated into two categories: 
 Exogenous – something that is given to the patient 
o EX – antibiotics (Vancomycin), cancer drugs, IV Iodinated contrast dye, and mannitol (high osmotic compounds)  
 Endogenous – something the body produces in excess  
o EX – Myoglobin (Rhabdomyolysis), Uric acid (Tumor Lysis Syndrome), Light Chains (Multiple Myeloma), Hemoglobin 
(Hemolysis) 
o Tumor Lysis Syndrome: Lysis of tumor cells post chemotherapy exposure; The higher the tumor burden, the greater 
the risk. 
Contrast Induced   Risk Factors: 
Nephropathy  o Advanced Age; Underlying CKD; Type II DM; Volume depletion; CHF; Anemia; Contrast load; Concomitant use of nephrotoxic 
agents 
o Incidence reported: 0.6 to 2.3% but higher if risk factors present 
 Clinical Features: 
o Rise in serum creatinine 48‐72 hours after contrast exposure 
o Generally non‐oliguric  still making urine 
o Initially pre‐renal urine indices with high urine specific gravity but ultimately leads to ATN 
 Treatment: 
o Prevention is the key. 
o Hydrate patients with IV fluids pre and post contrast exposure 
o Stop diuretics and ACEI/ARB where appropriate 
o Supportive medical management and in some cases, may need dialysis support. 
 
Pathogenesis   Key finding: granular (“muddy brown”) casts  

  
 Occurs in three stages 
o INITIATION PHASE: (hours to days) 
 Evolving tubular injury 
 Potentially reversible if diagnosed early 
o MAINTAINANCE PHASE: (typically 1‐2 weeks) 
 May be prolonged to 1‐3 months in some cases 
 Established renal injury 
 Generally oliguric (stop making urine) during this phase 
 Muddy brown urine with casts noted in sediment 
o RECOVERY PHASE:  
 Repair and regeneration of tubules. 
 Polyuric phase (making LOTS of urine 4‐8 L) and if patient is intubated/no access to free water, at risk for hypernatremia, 
replace 1/2‐1/3 of UOP with hypotonic fluid. 
 Even though serum creatinine may reach prior baseline, some residual scarring expected and can eventually lead to CKD. 
 
Glomerulus: Acute Glomerulonephritis 
   Unexplained renal failure 
 Can be hypertensive 
 Subnephrotic range proteinuria  <3 gm/day 
 Active urine sediment with positive blood, protein, WBC, dysmorphic RBC and RBC casts. 
 Kidney biopsy for definitive diagnosis 
Associated Diseases   Post Strep GN 
 Lupus Nephritis 
 ANCA Vasculitis 
 Anti GBM 
 Ig A/HSP 
 
Interstitium: Acute Interstitial Nephritis (AIN) 
Etiology   Medications – PPIs, Bactrim, Quinolones 
 Infections 
 Auto‐immune 
Clinical   Classic triad of fever, rash and eosinophilia seen in < 10% of cases. 
 Urine eosinophils neither sensitive nor specific. 
 Sterile pyuria  WBC in urine with NO BACTERIA 
 Definitive diagnosis by kidney biopsy 
Treatment   Generally reversible once the offending agent is stopped. 
 Treating underlying infection or auto‐immune disease. 
 
Blood Vessel: 
Most common causes   TTP/HUS 
o TTP – Thrombotic Thrombocytopenic Purpura 
o HUS: Hemolytic Uremic Syndrome 
 Malignant HTN 
o BP > 180/120 with end organ damage 
 Scleroderma renal crisis 
o Auto‐immune disease causing thickened skin and kidney problems 
 Preeclampsia 
o new onset HTN after 20 weeks, proteinuria can be present. Can cause AKI. 
 Thromboembolic disease 
 
Collecting System: Pyelonephritis 
   Clinical features:  
o fevers, flank pain, dysuria.  
 Positive urine/blood cultures with WBC casts in urine 
 Perinephric stranding on imaging 
 Treatment with fluids and ABX. 
 
 
 
 
 
Establishing a Diagnosis   History of inciting events – any nausea/vomiting/diarrhea/decreased oral intake, heavy exercise, fevers or rash, over the 
counter/herbal medication use, recent change in prescribed medications, recent hospitalizations, any contrast studies done in 
the recent past 
 Thorough chart review for in‐hospital AKI 
 Inputs/Outputs (often times urinary output (UOP) is not documented accurately and hence cannot take it at face value unless 
catheter in place), trends in BP/HR and weights, intra‐op notes if available, medication/contrast exposure 
 Physical exam in assessing volume status – JVD/crackles(fluid in the lungs)/edema/skin tenting but tough to assess volume status 
especially in cirrhotic patients whose total body volume is up but could be intravascularly dry. 
 Looking at the urine and urine sediment 
 Kidney biopsy for definitive diagnosis in Glomerulonephritis/Interstitial Nephritis/unexplained renal failure 
Pitfalls of Creatinine in AKI   AKI is not a steady state and hence cannot calculate estimated GFR using any of the available formulae 
 Serum creatinine lags behind the actual injury 
 Serum Creatinine levels depend on: 
o Clearance rate (changing in AKI) 
o Rate of production (changing in AKI) 
o Volume of distribution (changing in AKI) 
 Creatinine produced predominantly by the muscles and hence, muscular people can have high serum creatinine and emaciated 
people with very low serum creatinine, not corresponding with the actual GFR. 
 Drugs that block tubular secretion of creatinine can elevate serum creatinine without actual decrease in GFR 
o Ex: high dose bactrim, probenecid, high dose cimetidine 
Why was creatinine chosen   Creatinine is released from skeletal muscle at a constant rate proportional to muscle mass. 
to measure renal function?   Muscle mass decreases with age BUT GFR also normally decreases with age 
 Creatinine is freely filtered and NOT reabsorbed by the kidney; though a very small amount is secreted into the proximal tubule 
 Thus, if creatinine production remains constant, a decrease in GFR increases the plasma creatinine concentration, while an 
increase in GFR decreases plasma creatinine concentration 
Differentiating Pre‐Renal  Lab Value  Pre‐Renal  Intrinsic 
versus ATN (Intrinsic)  Urine Specific Gravity  > 1.020  <1.010 
Urine Osmolality (mOsm/kg)  >500  <350 
Urine Na (mEq/L)  <20  >40 
Fractional excretion of sodium (FeNa)  <35%  >35% 
Urine/Plasma Creatinine Ratio >40 <20
Serum BUN/Cr ratio  >20:1 <10:1
 
What do they reflect?  Intact tubular function  Impaired tubular function 
Serum BUN/Cr Ratio   High BUN 
o Prerenal state 
o High dose steroids 
o Hypercatabolic states – high fevers, burns 
o High protein diet – look for protein load in ICU patients on tube feeds 
o GI bleed 
 Low BUN or Creatinine 
o Low protein diet 
o Cachetic patients 
o Cirrhosis – high bilirubin interferes with creatinine measurement 
 High Creatinine 
o High muscle mass 
o Using creatine supplements 
o Rhabdomyolysis 
o Diet rich in animal protein 
o Drugs blocking tubular secretion 
o Interference with creatinine measurement – cephalosporins, ketosis 
Management of AKI   Treatment of underlying etiology  
o Volume repletion in volume depletion 
o Treatment of heart failure 
o Treatment of underlying infections 
o Stopping offending medications 
o Relieving obstruction. 
 Track daily weights, BP and Inputs/outputs. 
 Maintain mean arterial pressure (MAP > 60 mmHg). 
 Dose medications to renal function – can be tricky. 
 Avoid contrast studies and use least nephrotoxic medications when possible (should be the case in general). 
 Management of electrolyte disturbances – hyperkalemia, metabolic acidosis, hyperphosphatemia. 
Hemodialysis in AKI   Exact timing – controversial and varying results in literature. 
 Early dialysis: exposing the patient to risks of dialysis when there is a chance for renal function to recover. 
 Too late: may effect overall morbidity/mortality 
 Dialysis is an invasive procedure : requires line placement and complications associated with line placement (many sick ICU 
patients have bleeding diathesis), increased risk of hypotensive episodes and arrhythmias during dialysis (dialysis is the best 
available stress test for the heart!) 
 The bigger question is how do you determine the timing…. 
 
General indications for   Acidosis  
dialysis: AEIOU still stand  o metabolic acidosis refractory to medical management 
true to date.   Electrolytes 
o hyperkalemia refractory to treatment or rapidly rising levels in potassium 
 Intoxications with dialyzable drug, including  
o salicylates, lithium, isopropanol, methanol, and ethylene glycol (SLIME) 
 Overload  
o volume overload that does not respond to diuresis 
 especially with increased oxygen requirements 
 Uremia 
o elevated BUN with signs of uremia, such as uremic bleeding, encephalopathy, and pericarditis. 
Prognosis of AKI   Depends on severity of underlying illness. 
 Most patients recover but “complete” recovery is not always the case even if serum creatinine reaches baseline. 
 At risk for CKD with repeated AKI episodes. 
 Mortality of > 50% in patients with AKI and multi‐organ dysfunction. 
 Often times, patients die with renal failure than from renal failure due to widely available renal replacement therapy options. 
 
Congenital Abnormalities 
 
Development 
 
Kidney Development   Is characterized by three successive systems 
    o Pronephros 
o Mesonephros 
o Metanephros 
Week 4   Segmented nephrotomes appear in the cervical intermediate mesoderm of the embryo. These structures grow laterally and 
canalize to form nephric tubules. The first tubules formed regress before the last ones are formed. By the end of week 4, the 
pronephros disappears and does not function. 
Week 5   In week 5, the mesonephros appears as S‐shaped tubules in the intermediate mesoderm of the thoracic and lumbar regions of 
the embryo. 
o The medial end of each tubule enlarges to form the Bowman’s capsule into which a tuft of capillaries, or glomerulus, 
invaginates. 
o The lateral end of each tubule opens into the mesonephric (Wolffian) duct, an intermediate mesoderm derivative. The duct 
drains into the hindgut  
o Mesonephric tubules function temporarily and degenerate by the beginning of the third month. The 
mesonephric duct persists in the male as the ductus epididymidis, ductus deferens, and the 
ejaculatory duct. Where the mesonephric duct disappears in females 
 During week 5, the metanephros, or permanent kidney, develops from 2 sources:  
o the ureteric bud – a diverticulum of the mesonephric duct 
o the metanephric mass (blastema) – from intermediate mesoderm of the lumbar and sacral regions. 
 The ureteric bud penetrates the metanephric mass, which condenses around the diverticulum to form 
the metanephrogenic cap.  
o The bud dilates to form the renal pelvis, which subsequently splits into the cranial 
and caudal major calyces.  
 Each major calyx buds into the metanephric tissue to form the minor calyces. 
 One to 3 million collecting tubules then develops from the minor calyces 
forming the renal pyramids.  
 The ureteric bud forms the drainage components of the urinary system (calyces, 
pelvis, ureter). 
 Penetration of collecting tubules into the metanephric mass induces cells of the tissue cap to form nephrons, or excretory units. 
o Lengthening of the excretory tubule gives rise to the proximal convoluted tubule, the loop of Henle, and the distal 
convoluted tubule. 
 The kidneys develop in the pelvis but appear to ascend into the abdomen as a result of fetal growth of the lumbar and sacral 
regions between the 6‐9 weeks 
 With their ascent, the ureters elongate, and the kidneys become vascularized by arteries which arise from the abdominal aorta. 
 
Congenital Abnormalities of the Renal System 
Renal Agenesis   Results from failure of one or both kidneys to develop because of early degeneration of the ureteric bud. 
o Unilateral agenesis if fairly common 
 Where the remaining kidney undergoes compensatory hypertrophy.  
 Patients often have adequate renal function and are asymptomatic. 
o Bilateral agenesis is fatal and associated with Oligohydramnios  called Potter Sequence 
 Affected fetuses typically also have  
 Potter facies – flattened nose, low set ears ears, and recessed chin 
 Talipes equinovarus – talus [ankle] + pes [foot] and equino [heel] + varus [turned upward] = clubfoot 
 Pulmonary hypoplasia 
Hypoplasia   Hypoplasia is failure of a kidney (usually unilateral) to develop to normal weight; the hypoplastic kidney has a decreased number 
of calyces and lobes. 
Horseshoe Kidney   Horseshoe kidney is a common congenital anomaly that is found in 1:500‐1000 autopsies 
 The kidneys show fusion, usually at the lower pole  where the kidney is trapped by the Inferior Mesenteric Artery 
 Affected individuals have normal renal function but may be predisposed to renal calculi. 
Abnormal locations   The most common abnormal location is a pelvic kidney.  
(Ectopic)   The ectopic kidney usually has normal function.  
 Tortuosity of ureters may predispose to pyelonephritis. 
 
 
 
 
 
 
 
 
 
Cystic Lesions 
 
Childhood Autosomal   is a rare autosomal recessive disease that presents in infancy with progressive and often fatal renal or liver 
Recessive Polycystic Kidney  failure. 
Disease (ARPKD)  o perinatal, neonatal, infantile, juvenile subcategories 
(also called infantile   Epidemiology: incidence 1 in 20,000  rare 
polycystic kidney disease or   Pathogenesis: A mutation in the PKHD1 gene, Chromosome 6p, fibrocystin  
renal dysgenesis Potter type   The kidneys are  
1)  o bilaterally enlarged (reniform kidney‐like shape)  
o cysts in the cortex & medulla 
o cross‐sections have a sponge‐like appearance, saccular dilatation of collecting tubules 
 The liver may have  
o cysts + expanded portal area (fibrosis) + bile ducts proliferation (tortuous dilated) leading to 
congenital hepatic fibrosis 
 Diagnosis:  
o Radiology 
 Prognosis: 
o renal/liver failure at birth, infancy, juvenile  oliguric renal failure in utero can lead to Potter sequence 
o Beyond neonatal period include: systemic hypertension, progressive renal insufficiency, and portal hypertension 
 Treatment: transplantation 
Autosomal Dominant   is an autosomal dominant disease that affects 1 in 1,000  common 
Polycystic Kidney Disease  o There is most frequently a mutation of the PKD1 gene on chromosome 16 which produces a transmembrane 
(also called Adult Polycystic  protein called polycystin 1.  
Kidney Disease (APKD) or   85%, earlier onset of renal failure, more severe 
renal dysgenesis Potter type   polycystin 1 function  Cell‐cell, cell‐matrix interactions 
III)  o Other mutations involve PKD2 on chromosome 4 which produces polycystin 2. 
 later onset of renal failure 
 polycystin 2 function  regulation of intercellular Ca2+ levels 
o Abnormalities in cell differentiation 
o Ciliopathy (mechanosensors) – defects in mechanosensing 
 Clinically, patients are (40‐50 year old) 
o asymptomatic or pain 
o colic 
o abdominal mass 
o hemorrhage 
o hematuria 
o progressive renal failure 
o polyuria 
o hypertension 
o low proteinuria (<2g) 
 Diagnosis is established with U/S and CT scan 
o Most patients develop end‐stage renal failure by middle/later life depending on mutation  
 On gross pathologic examination, the kidneys have massive bilateral enlargement (reniform kidney‐
like shape) with large bulging cysts filled with serous, turbid, or hemorrhagic fluid.  
 Microscopic examination shows functioning nephrons present between the cysts; the cysts arise 
from the tubular epithelial cells of the kidney.  
 Extrarenal manifestations include: 
o Benign cysts of the liver, pancreas, and lungs (40%) 
o berry aneurysms of the circle of Willis (4‐10% of deaths) 
o mitral valve prolapse (20‐25%) 
o colonic diverticula 
 Treatment:  
o Transplantation; ACE inhibitors or ARBs 
   Dysplasia versus hypoplasia versus polycystic:  
o Dysplasia – abnormal development 
o Hypoplasia – small size but otherwise normally developed 
o Polycystic – cystic but without dysplastic elements 
Cystic Renal Dysplasia (also   Aberrant metanephric differentiation – persistence of immature elements, cartilage 
called Pediatric Dysplasia)  o NOT related to neoplasia but a developmental abnormality 
 The most common renal cystic disease in children (1:1000‐2000) 
 Associated abnormalities:  
o uretero‐pelvic junction obstruction 
o ureteral agenesis/atresia 
 Grossly: kidney(s) are enlarged irregular shaped renal mass (“bunch of grapes”) with cartilage and immature collecting ducts 
 Microscopy: disorganized renal parenchyma with immature tubules surrounded by 
collarets of condensed mesenchyme; cartilage 
o immature, persistent abnormal structures aberrant nephronic differentiation, 
biologically benign cysts enlarge but there is NO invasion of nearby tissues, no 
metastatic potential 
 It may progress clinically to renal failure. 
 
  Pediatric Dysplasia  ARPKD (Childhood)  APKD (Adult) 
Incidence  1:1,000‐2,000  1:20,000 (rare)  1:500‐1,000 (common) 
Bilateral  Yes/No  Yes (hereditary) Yes (hereditary)
Segmental  Yes/No  No  No
Shape  Irregular  Reniform  Reniform
Ureter abnormalities  Yes  No  No 
Liver abnormalities  No  Yes  Yes 
 
Acquired polycystic disease   Seen in renal dialysis patients:  
o Cortical and Medullary cysts 
o small risk (7%/10years) of developing renal adenomas and renal cell carcinoma. 

Simple cysts   Cysts are filled with ultrafiltrate and anechoic (black) on ultrasound  
 Very common in adults  
 Usually asymptomatic with multiple or single cysts 
 Can cause hematuria, pain, and calcification 
Medullary diseases with   Inherited disease causing tubulointerstitial fibrosis and progressive renal insufficiency with inability to concentration urine. 
cysts  Medullary cysts usually not visualized; shrunken kidneys on ultrasound. Poor Prognosis 
 Medullary sponge kidney  
o can cause nephrolithiasis 
o relatively common and innocuous 
 Nephronophthisis‐medullary cystic disease complex  
o presents as polyuria and polydipsia 
o Pediatric onset CKD (Chronic Kidney Disease) that can progress to chronic renal failure in young adults.  
o There are autosomal recessive forms.  
o The cysts are in the cortex and the medulla. 
 
 
   
Kidney Tumors 
 
Adult Benign Kidney Tumors 
Oncocytoma   Benign epithelial cell tumor arising from collecting ducts (distal nephron)  
 Presents: painless hematuria, flank pain, abdominal mass 
 Gross: well circumscribed mass (mahogany brown) with central scar 
 Microscopy: Large eosinophilic cells with abundant mitochondria without perinuclear clearing 

   
 Treatment: often resected to exclude malignancy  such as renal cell carcinoma 
Angiomyolipoma   Vessels/smooth muscle/fat (radiologic diagnosis) 
 Tuberous sclerosis, some sporadic  
 Tuberous Sclerosis is associated with two genes 
o TSC1 ‐ Hamartin 9q34 
o TSC2 ‐ Tuberin 16p13 
 benign tumors: brain, kidneys, heart, liver, eyes, lungs, skin 
 microscopic: fat, smooth muscle, poorly formed vessels benign, can rupture with bleeding  
o when tumor reaches 4 cm, the tumor is removed due to risk of rupture 
 
Renal Cell Carcinoma (RCC) 
   Most common 1° renal malignancy. 

  
o Associated with gene deletion on chromosome 3 (most sporadic or familial (4%) as von Hippel‐Lindau syndrome).  
o Clear Cell Carcinoma: 
 RCC = 3 letters = chromosome 3  3p deletion 
 somatic mutation/hypermethylation induced inactivation of the VHL gene (tumor suppressor gene) 
 Loss of VHL gene results in accumulation of the transcription factor  
 HIF‐1alpha and over‐expression of HIF‐1alpha target genes, which facilitate cellular adaptation to tissue hypoxia  
 VHL gene in development of both sporadic and familial clear cell RCC 
 Other kidney cancers:  
 chromosomal gains (papillary, met oncogene) 
 Chromosomal losses (chromophobe) 
o Associated with paraneoplastic syndromes:  
 Hypercalcemia 
 Hypertension 
 Polycythemia 
 Cushing syndrome 
 feminization/masculinization 
 Originates from PCT cells  polygonal clear cells filled with accumulated lipids and carbohydrates ‐‐‐‐‐‐‐‐‐‐‐‐‐‐‐‐‐‐‐‐‐‐> 
 
 Often golden‐yellow due to increased lipid content. ‐‐‐‐‐‐‐‐‐‐‐‐‐‐‐‐‐‐‐‐‐‐‐‐‐‐‐‐‐‐‐‐‐‐‐‐‐‐‐‐‐‐‐‐‐‐‐‐‐‐‐‐‐‐‐‐‐‐‐‐‐‐‐‐‐> 
 
 Most common in men 50–70 years old.  
 Risk factors:  
o Tobacco 
o Obesity 
o HTN 
o occupational exposure to heavy metals (eg cadmium) 
 Manifests clinically with: 
o Hematuria 
o palpable mass 
o 2° polycythemia 
o flank pain 
o fever 
o weight loss.  
 Invades renal vein (may develop varicocele if left sided) then IVC and spreads hematogenously; metastasizes to lung and bone. 
 Treatment:  
o surgery/ablation for localized disease.  
o Immunotherapy (eg, aldesleukin, IL‐2, IFN‐alpha) 
o Targeted therapy for metastatic disease (multikinase inhibitors = Sunitinib, Sorafenib; monoclonal antibodies to VEGF = 
Bevacizumab; mTOR inhibitors = Temsirolimus, Everolimus), rarely curative. Resistant to chemotherapy and radiation 
therapy. 
 
 
 
von Hippel‐Lindau (VHL)   1:40,000, autosomal dominant  
syndrome associated with   Age @ onset 37 years versus 61 years in sporadic RCC 
RCC clear cell type   Multiple bilateral cysts & tumors in 40‐60% of affected people 
 Germline mutation of the VHL gene 3p25, somatic mutation on second allele 
 Tumors with high vascularity + clear cells 
 Hemangioblastoma (cerebellum, retina) 
 Angiomas of the retina 
 Pheochromocytomas (some), other 
 Many different alterations in the VHL gene (3p25) with some correlation between specific inactivation and phenotype 
Control of HIF‐1alpha by   Under normoxic conditions VHL ubiquitinates HIF‐1alpha, leading to ubiquitin‐mediated proteolysis and 
VHL in Normoxic Conditions  degradation by the proteasome  

Control of HIF‐1alpha by   In hypoxic cells, such as those found in tumors, HIF‐1alpha ultimately initiates the transcription of hypoxia‐induced 
VHL in Hypoxic Conditions  genes, including those which promote 
o Cell survival under anaerobic conditions 
o Angiogenesis 
o Metastasis 
Clear Cell Type Pathology   Pathology dependent on carcinoma type:  
o RCC clear cell type, most common, 65% 
o Gross: yellow‐orange “adrenal gland‐like” with Invasion into renal vein at advanced stages 

o  
o Microscope: cells with clear cytoplasm, highly vascular, delicate, “chicken” wire‐like, blood vessels   

o  
Renal cell carcinoma –   Morphology + cytogenetic + molecular genetic studies 
adults classification   Clear cell carcinoma (aka conventional) ‐ 65%; loss 3p 
 Papillary renal cell carcinoma: 10‐15%; chromosomal gains 
 Chromophobe renal cell carcinoma: 5%; chromosomal losses 
Papillary RCC   10‐15%, better prognosis than clear cell RCC 
 Contains Papillary architecture 
 Can be Hereditary (Familial) ‐ multiple bilateral tumors  
 Frequiently cystic 
 Trisomies 7, 16, 17 
 Involvement of Chromosome 7 – MET oncogene 
 Gross: 

  
 Microscope: 

  
Chromophobe RCC   5% 
 Multiple chromosomal losses 
 Distal nephron 
 Morphologic overlap with oncocytoma 
 Histiology: Prominent cell membrane “vegetable‐like” 
 Better prognosis 
 Microscope: vegetable‐like cells 

  
 Electron Microscopy: vesicles 

  
 
mTOR: A Central Regulator   mTOR is an intracellular serine/threonine kinase in the PI3K/Akt signaling pathway 
of Cancer Cell Growth,   mTOR is a central regulator that senses changes in 
Angiogenesis, and  o Growth factor signaling  
Metabolism  o Nutrients and energy 
 mTOR activation promotes 
o Cell growth and proliferation 
o Angiogenesis 
o Cancer cell metabolism through increased nutrient uptake and utilization 
 mTOR = mammalian target of Rapamycin 
 Activation of mTOR signaling promotes the production of proteins that regulate progression through the cell cycle and 
angiogenesis  
 mTOR regulates HIF – activation of mTOR increases HIF activity  
 Activation of mTOR by mutations that disrupt the TSC 1 and 2 genes confers a predisposition to RCC and is associated with 
increased HIF activity  
Estimated Survival by Stage   5 year survival estimates based on stage  
o Stage I = 96%  tumon less than 4 cm in diameter and limited to the kidney 
o Stage II = 82% 
o Stage III = 64%  extention of tumor into the renal sinus 
o Stage IV = 23% 
Adult renal tumors ‐   Malignant until proven otherwise 
summary   Surgical treatment = mainstay, targeted therapies 
 Prognosis: 
o stage, tumor histology 
o presence of nodal metastases 
o evidence of metastatic disease at presentation 
o patterns of metastases: lung, bone, brain, liver and adrenal gland 
 Classification evolving, role of molecular studies 
 
Urothelial carcinoma   
   5‐10% of renal tumors = urothelial carcinoma 
 Adults 
 Renal pelvis, ureter, frequently also concomitant urinary bladder cancer 
 Hematuria 
 Analgesic nephropathy = risk factor 
 
Wilms Tumor 
Clinical   Typical   palpated mass 75% 
Presentation:   25% with other developmental abnormalities; syndromic in 10%: 
o Beckwith‐Wiedemann syndrome: Wilms tumor, macroglossia, organomegaly, hemihypertrophy 
(extremities) 
o WAGR complex: Wilms tumor, Aniridia, Genital abnormalities, mental Retardation 
o Denys‐Drash syndrome: Wilms tumor, gonadal dysgenesis, renal (mesangial sclerosis) 
 Epidemiology:   pediatric, most common kidney tumor  
 children <10 yo; 2‐5 yo 
 Pathogenesis   mutations WT1, WT2, loss of function mutation 
 recapitulates nephrogenesis 
 nephrogenic rests = putative precursor lesion of WT 
 Pathology:   recapitulates nephrogenesis 
o undifferentiated blastemal: “embryonal”, “small blue cells”, attempts to recapitulate nephrogenesis, 
biologically malignant; uncontrolled growth, can invade nearby tissues and metastasize 
Gross – very soft 

 
Wilms’ tumor triphasic histology  ‐undifferentiated blastema 
(astrix) (small blue cells) 
‐tubules (carrot) 
 
‐fibroblast‐like stroma (hashtag)  
 Diagnosis:   radiologic studies 
 Prognosis:   excellent, 90% five years survival 
 Treatment:   surgery with/without chemotherapy depending on stage 
 
 chemo sensitive 
 
Pediatric Malignancy: Age 0 to 4 years 
   Leukemia – blood 
 Retinoblastoma – eye, neuronal origin 
 Neuroblastoma – adrenal gland 
 Wilms’ tumor ‐ kidney 
 
Wilms Tumor Versus Cystic Dysplasia 
Feature  Wilms Tumor  Cystic Dysplasia 
Congenital Malformation  Increased risk  100% 
Histogenesis  Attempts to recapitulate  Aberrant Nephronic Differentiation 
Nephrogenesis 
Histology  Undifferentiated blastema: embryonal, small blue cells  Immature, persistent abnormal structures 
Behavior  Malignant  Benign 
Invasion of Nearby Tissues  Yes  No 
Metastatic Potential  Yes  No 
Cysts  Rare  Yes 
Genetics  Mutations: somatic, germline some  Sporadic 
Bilateral  5‐10%  Bilateral associated with Potter’s syndrome 
Synchronous (simultaneously) 
Metachronous (one after the other) 
 
 
A 2 yo boy is brought to pediatrician because his mother palpated “a bulging mass” while bathing him.  What best applies to the 
case? 
A. This tumor most likely is composed of clear cells 
B. This tumor typically shows loss of short arm of chromosome 3 
C. This tumors is most likely highly chemoresistant 
D. This tumor is chemosensitive 
E. Cysts, disorganized renal parenchyma with immature tubules and cartilage 
 
An infant is diagnosed with an enlarged left kidney – shown.  Most likely:  

   
A. This lesion will respond to chemotherapy 
B. This is autosomal dominant lesion  
C. This is a congenital disorder with recessive inheritance 
D. Cystic lesion, disorganized renal parenchyma with immature tubules and cartilage 
E. Kidney sections show undifferentiated blastema, tubules and stroma 
 
During a radiologic workup for gall bladder stones, a 65 yo male was found to have a 5 cm mass in his right kidney.   
A. This is most likely oncocytoma 
B. This is most likely a benign tumor 
C. Chemotherapy will be effective 
D. Surgery consultation will be scheduled 
E. Tumor’s morphology shows abortive glomeruli/tubules and stroma 
 
 
 
 
Chronic Kidney Disease (CKD) and End Stage Renal Disease (ESRD) 
 
CKD: Definition and   Presence of either kidney damage or decreased renal function 
Epidemiology  o Duration for > 3 months irrespective of cause 
o +/‐ Albuminuria 
 HTN, Diabetes Mellitus make up 72% of all cases 
 More common in African Americans 
 Prevalence overall on the rise 
o Aging population 
o Increased prevalence of HTN, DM, obesity 
 CKD risk factor for cardiovascular disease 
o Angina pectoris 
o ACS (Acute Coronary Syndrome) 
o Heart failure 
o Stroke 
o AKI (Acute Kidney Injury) 
o Peripheral Vascular Disease 
o Arrhythmias/Sudden Cardiac Death 
 < 2% of patients ultimately require RRT 
o Most die from CV causes before ESRD 
KDIGO CLASSIFICATION ‐ 
2012 


CALCULATION OF eGFR   MDRD (Modified Diet in Renal Disease) formula: widely used. 
(estimated GFR)   Current staging of CKD based on MDRD classification. 
 Some important considerations of MDRD: 
o Can’t be used in AKI, has to be in steady state 
o Underestimates GFR in the healthy 
o Not tested in pregnancy and the very elderly 
o Relies on creatinine and so all the pitfalls associated with it. 
 CKD EPI equation thought to be more accurate than MDRD  
Etiology of CKD   Diabetes Mellitus – 44.9% 
 Hypertension – 27.2% 
 Glomerulonephritis – 8.2% 
 
CKD Snapshots 
Diabetic Nephropathy   Most common cause of CKD/ESRD in US 
 Pathogenesis:  
o Hyperglycemia and glycation of tissue proteins leads to mesangial expansion  
o glomerular basement membrane thickening 
o podocyte injury 
o hyaline deposition in the glomerular arterioles (nodular glomerulosclerosis or Kimmelstiel‐Wilson (KW) lesions) 
Diabetic Nephropathy  Type 1 DM  Type 2 DM 
Onset  15 years  5‐10 years 
Retinopathy  Precedes nephropathy  Not always the case 
Albuminuria  Yes  Yes
Hematuria  Can be present Can be present
Treatment  Tight control of DM early in the diagnosis and ACEI/ARB for BP control 
 
 Progression of Diabetic Nephropathy – Not always the case and albuminuria can improve with tight DM and BP control. 


 
 
Hypertensive Nephropathy   2nd most common cause of CKD/ESRD in US. 
 Also known as “hypertensive nephrosclerosis” or “benign nephrosclerosis”. 
 African Americans have an 8‐fold increased risk even if good BP control is achieved.  
o Possible role of apolipoprotein ‐1 (APOL‐1) gene variation. 
 Generally seen in patients with long standing history of HTN with slow gradual decline in renal function. 
 Pathogenesis:  
o Hyaline arteriosclerosis of small arteries and arterioles in the kidney along with focal and segmental sclerosis and interstitial 
fibrosis. 
 Proteinuria is present but generally less than 1 gm/24 hours 
 Small to normal size kidneys on ultrasound imaging. 
 Treatment is control of BP. 
 
Glomerular Diseases: Glomerulonephritis 
Glomerular Diseases:   CLINICO‐PATHOLOGICAL DIAGNOSES: based on biopsy findings in appropriate clinical settings 
Glomerulonephritis  o IgA nephropathy is predominant in Asian populations 
o Post‐infectious glomerulonephritis 
o Membranoproliferative glomerulonephritis 
o Lupus nephritis 
o Rapidly progressive glomerulonephritis 
IgA Nephropathy   Most common glomerulonephritis worldwide 
 IgA deposits in glomeruli (mesangial) 
 Progresses to CKD over several decades in 25‐30% of cases 
 Typically presents as hematuria 1‐2 days after URI (Upper Respiratory Tract Infection) 
 Clinical features: 
o Microhematuria/proteinuria 
o AKI, nephrotic range proteinuria (rare) 
 Many associated systemic diseases: 
o Cirrhosis 
o RA 
o HIV 
o HSP 
Post‐infectious GN   Most often a complication of streptococcal skin infections (impetigo) or streptococcal pharyngitis 
 Immune complexes become lodged in the glomerular basement membrane (GBM) 
 Complement activation leads to GBM destruction 
 Onset of disease usually 2‐4 weeks after initial infection 
 High ASO titre, low serum C3 complement usual findings (C4 is normal) 
Membranoproliferative   Occurs most commonly in children and young adults 
Glomerulonephritis ‐ MPGN   Seen with Hepatitis C infection 
 Caused by deposits in the GBM and the mesangium 
 Complement activation leads to glomerular destruction 
 Not to be confused with membranous glomerulonephritis (does not affect mesangium) 
Lupus Nephritis   Auto‐immune disease with anti‐nuclear antibodies and complement activation leading to multi‐system organ involvement. 
 Kidney involvement seen in up to 50% at some point in time. 
 Six histologic subtypes of disease described 
 Stage 1: minimal   in its histology has a normal appearance under a light microscope, but mesangial deposits are visible under an electron 
mesangial  microscope. At this stage urinalysis is normal. 
nephritis 
 Stage 2: mesangial   is noted by mesangial hypercellularity and matrix expansion. Microscopic haematuria with or without proteinuria may 
proliferative  be seen. Hypertension, nephrotic syndrome, and acute kidney insufficiency are very rare at this stage 
nephritis 
 Stage 3: focal   is indicated by sclerotic lesions involving less than 50% of the glomeruli, which can be segmental or global, and active 
lupus nephritis  or chronic, with endocapillary or extracapillary proliferative lesions. Under the electron microscopy, subendothelial 
  deposits are noted, and some mesangial changes may be present. Immunofluorescence reveals positively for IgG, IgA, 
IgM, C3, and C1q. Clinically, haematuria and proteinuria are present, with or without nephrotic syndrome, 
hypertension, and elevated serum creatinine 
 Stage 4: diffuse   is both the most severe, and the most common subtype. More than 50% of glomeruli are involved. Lesions can be 
proliferative  segmental or global, and active or chronic, with endocapillary or extracapillary proliferative lesions. Under electron 
nephritis  microscopy, subendothelial deposits are noted, and some mesangial changes may be present. Clinically, haematuria 
  and proteinuria are present, frequently with nephrotic syndrome, hypertension, hypocomplementemia, elevated anti‐
dsDNA titres and elevated serum creatinine 
 Stage 5:   is characterized by diffuse thickening of the glomerular capillary wall (segmentally or globally), with diffuse membrane 
membranous  thickening, and subepithelial deposits seen under the electron microscope. Clinically, stage V presents with signs of 
nephritis  nephrotic syndrome. Microscopic haematuria and hypertension may also been seen. Stage V also can also lead to 
  thrombotic complications such as renal vein thromboses or pulmonary emboli. 
 Stage 6: advanced   is represented by global sclerosis involving more than 90% of glomeruli, and represents healing of prior inflammatory 
sclerosing lupus  injury. Active glomerulonephritis is not usually present. This stage is characterised by slowly progressive kidney 
nephritis  dysfunction, with relatively bland urine sediment. Response to immunotherapy is usually poor. A tubuloreticular 
  inclusion within capillary endothelial cells is also characteristic of lupus nephritis, and can be seen under an electron 
microscope in all stages. It is not diagnostic however, as it exists in other conditions such as HIV infection. It is thought 
to be due to the chronic interferon exposure 
 
 
 Lupus nephritis staging does NOT imply a chronological progression from stages I‐VI 
 Can see stage V with other stages 
 Stage IV is most common and most severe 
 Clinical sequelae 
o HTN 
o Hematuria 
o proteinuria/nephrosis 
o Progressive CKD 
o thrombophilia 
Rapidly Progressive   A.K.A. “crescentic glomerulonephritis”  
Glomerulonephritis‐ RPGN  o High numbers of crescents seen on renal biopsy histology 
 Rapid decline in renal funcion (in days/weeks/months) 
 Common causes 
o Goodpasture’s syndrome 
o Immunecomplex GN – Lupus, Ig A 
o Pauci‐immune as in ANCA Vasculitis 
 
Glomerular Diseases: Nephrotic Syndrome 
Glomerular Diseases:   Minimal change disease  highest prevalence with children 
Nephrotic Syndrome   Focal segmental glomerulosclerosis  highest prevalence with adults 
 Membranous nephropathy 
 Amyloidosis 
Minimal change disease   Most common cause of nephrotic syndrome in young children  
o Can also occur in older children and adults 
 More common in those with history of autoimmunity 
 Bland light microscopy findings 
o Hence the name of the disease 
 Usually treatable with steroids and ACE‐I therapy (for proteinuria) 
Focal segmental   Occurs in children and adolescents more often than in adults 
glomerulosclerosis (FSGS)  o Most common cause of nephrotic syndrome in adults, though 
 Can be primary disease or secondary to other conditions 
o HIV, heroin use, familial forms 
 Less responsive to steroid treatment compared to minimal change disease 
 Can be misdiagnosed as Minimal Change Disease as focal sclerosis can be missed. 
Membranous nephropathy   2nd most common cause of nephrotic syndrome in adults 
 Tendency to affect Caucasians 
 Most cases are primary, anti‐PLA2R (Phospholipase A2 Receptor) autoantibodies thought to play a role. 
 Increased risk of thromboses (DVT, PE) compared to other nephrotic syndromes 
 Secondary causes 
o SLE, infections (hepatitis B), drugs, tumors 
 Treatment: immunosuppression (mixed results) 
Amyloidosis   Variety of disorders in which amyloid proteins deposit in tissues/organs and cause damage 
 Primary disease: AL amyloidosis 
 Secondary disease: AA amyloidosis 
 Clinical features: CKD, heart disease, skin lesions, macroglossia (enlarged tongue), GI disease, polyneuropathy 
 
Cholesterol Atheroembolic   Seen in the presence of severe atherosclerotic disease. 
Disease   Occurs when cholesterol is released from an atheromatous plaque into the bloodstream. 
 Generally occurs after an intervention (CT contrast, angiogram, vascular surgery) but can occur spontaneously as well 
(hemodynamic stress). 
 Clinical manifestations: Fever, malaise, digital gangrene, characteristic rash (livedo reticularis), renal failure. 

  
 Low complements and peripheral eosinophilia can be present. 
 Definitive diagnosis is by tissue biopsy (Cholesterol clefts) ‐‐‐‐‐‐‐‐‐‐‐> 
 Treatment: Supportive medical management 
 
Polycystic Kidney Disease   Two types:  
o ADPKD – Autosomal Dominant Polycystic Kidney Disease and  
o ARPKD  ‐  Autosomal Recessive Polycystic Kidney Disease 
 ADPKD more common than ARPKD, approximately 1 in every 400 to 1000 live births 
 Mutations in PKD1 (85% cases, chromosome 16) or PKD2 (15% cases, chromosome 4) genes account for most cases of ADPKD.  
 Clinical presentation: Hematuria, flank pain, kidney stones,. Concomitant liver cysts, diverticulosis of the colon, mitral valve 
prolapse and intracranial aneurysms can be seen. 
 Diagnosis: Genetic testing available and presence of multiple cysts on imaging studies. 
 
 
CKD Management 
CONSEQUENCES OF RENAL  MAD HUNGER – PNEUMONIC 
FAILURE   Metabolic Acidosis 
  Dyslipidemia (especially high triglycerides) with nephrotic range proteinuria 
  Hyperkalemia 
  Uremia – clinical syndrome marked by an increase BUN leading to nausea, anorexia, asterixis, encephalopathy, pericarditis, 
platelet dysfunction. 
  Na+/H20 retention – HTN, Volume Overload, Heart failure 
  Growth retardation and developmental delay 
  Erythropoietin deficiency 
  Renal Osteodystrophy due to secondary hyperthyroid deficiency 
CKD: Stage II/III   Promote healthy living 
Management:  o Smoking cessation 
“Conservative  o Normal body weight, exercise 
Renoprotection”   BP, lipid control 
 Glycemic control (diabetics) 
 Optimize coexistent liver, cardiac disease 
 ACE‐I/ARB therapy 
o Especially with proteinuria 
 Medication dosage adjustment per eGFR 
 Avoid nephrotoxic agents (NSAIDs, e.g.) 
CKD Stage III‐V   Step One: Nephrology Referral 
Management 
CKD Stage III‐V   Pathophysiology:  
Management:  o Sodium retention due to decreased GFR 
Hypertension Control  o High renin state 
 Lifestyle changes play a huge role:  
o 2 gm sodium diet 
o Weight loss, smoking cessation                               
 Just as an example: One slice of bread can contain anywhere from 80 to 230 mg of sodium, and a slice of frozen pizza can contain 
between 370 and 730 mg. 
CKD Stage III‐IV   Goal is < 300mg – 500mg/day 
Management:   Slows CKD progression 
Proteinuria Reduction   Usually treatment is through ACE‐I +/‐ ARB 
 Reduce dietary protein intake 
o 0.8 – 1.0gm/kg/day 
CKD Stage IV‐V   In chronic kidney disease there is reduced renal phosphorous clearance 
Management:  o Leads to hyperphosphatemia and increased FGF‐23 
Mineral/Bone Disorders   Causes reduced 1,25(OH)2 vitamin D 
 Resulting in hypocalcemia and secondary hyperparathyroidism 

  
CKD Stage IV‐V   Restrict phosphate intake or reduce phosphate with phosphate binders 
Management:   Supplement 1,25(OH) Vit‐D 
MBD ‐ Treatment 

  
High Phosphorus Diet   All dairy products 
 Dried beans and lentils 
 Processed meat 
 Chocolate 
 Color Soda: Cola, Dr. Pepper and Beer 
 Muffins, Biscuits, or Pancakes from a Mix 
 Liver or Organ Meats 
 Nuts and Seeds 
 
 
 
 
 
 
CKD Stage IV‐V   Many contributing factors 
Management:  o Decrease Erythropoietin synthesis 
Anemia  o Iron deficiency and transport dysregulation 
o Decrease Erythrocyte half‐life 
o Blood loss in dialysis patients 
 Treatment goals 
o Target hemoglobin 9‐11 g/dL 
o Target transferrin saturation > 20%  
 Treatment options 
o Exogenous erythropoietin – subcutaneous or IV routes 
o Iron supplementation – oral or IV 
 
CKD Stage IV‐V   Early CKD: normal anion gap metabolic acidosis 
Management:  o Due to defective tubular hydrogen ion secretion 
Metabolic acidosis   Advanced CKD: increased anion gap acidosis 
o Due to defective sulfuric acid excretion 
 Long term effects if metabolic acidosis in CKD 
o Progression of CKD by accelerating interstitial fibrosis 
o Bone resorption and osteopenia 
 Treatment options 
o Sodium bicarbonate or sodium citrate. 
o Aim to keep serum bicarbonate in the normal range (22‐24). 
o Would avoid in volume overloaded patients. 
 
End Stage Renal Disease (ESRD) 
Renal Replacement Therapy   Kidney Transplantation: Living or deceased donor  
Options  o Kidney transplantation is the best option but not everyone is a candidate and not enough kidneys available. 
 Hemodialysis: In‐center or home HD 
 Peritoneal dialysis 
Dialysis   Dialysis is a process of by which the solute composition in one compartment (blood) is altered by exposing it to a solution in a 
second compartment (dialysate) through a semipermeable membrane. 
 It is a process of removing toxins and fluid from the extravascular compartment. 
o Simple diffusion for the removal of toxins and waste products 
o Convection for volume removal 
 The goal of dialysis is to remove accumulated fluid and toxins to maintain their concentrations below the levels at which they 
produce uremic symptoms 
Principle Mechanisms in   Simple diffusion for the removal of toxins and waste products 
Hemodialysis   Convection for volume removal 
Principle Mechanisms in   Simple diffusion for the removal of toxins and waste products 
Peritoneal dialysis   Osmosis for volume removal by osmotic gradient using dextrose 
Sequelae of ESRD   Uremic cardiovascular disease 
o Medial vascular calcification 
o Arterial stiffness 
o LV hypertrophy 
o Higher risk of cardiac arrest and heart failure 
 
Metabolic Acidosis or Alkalosis 
 
Introduction   An acid is a compound capable of donating a proton and a base is a compound capable of accepting a proton 
 pH is defined as the negative logarithm of the hydrogen ion concentration 
General Concepts   Daily Acid Production 
o Volatile mechanism – CO2 production which is excreted by the lungs 
o Non‐volatile mechanism – acid production from intermediary metabolism, specifically catabolism of sulfur‐containing amino 
acids in protein 
 There are 2 main homeostatic mechanisms which are responsible for maintaining acid‐base balance in response to non‐volatile 
acid production 
o Internal buffer systems – which neutralize acid 
o Excretion of hydrogen ion – which excretes acid 
 
Metabolic Acidosis 
Metabolic acidosis   Defined as a primary decrease in the bicarbonate concentration of the plasma 
o It is normally accompanied by a decrease in PCO2 (respiratory adaptation) in order to maintain near normal pH 
Henderson and Henderson‐  These equations depict the relationships between the three determining variables in acid‐base homeostasis 
Hasselbalch Equations   Henderson 
24 ∙ 2
 
3
 Henderson‐Hasselbalch 
3
 
2 3
 Acid‐Base Homeostasis is dependent on the following reactions: 
3 ↔ 2 3↔ 2 2  
o A primary change in either [HCO3‐] or PCO2 will produce an adaptive (compensatory) change in the other in the same 
direction in an attempt to keep pH constant (Tables 1 and 2) 
 Table 1. The Four Primary Acid‐Base Disorders and Their Compensatory Responses 
Acid‐Base Disorder  Primary Abnormality  Effect on pH  Compensatory Response 
Metabolic Acidosis  Low [HCO3]  Low pH  Decrease PCO2 
Metabolic alkalosis  High [HCO3] High pH Increase PCO2
Respiratory acidosis  High PCO2 Low pH High [HCO3]
Respiratory alkalosis  Low PCO3  High pH  Low [HCO3] 
 
 Table 2. Expected Compensation for Primary Acid‐Base Disorders 
Acid‐Base Disorder  Primary Change  Secondary Adaptation 
Metabolic Acidosis  Decrease in HCO3 of 1 mEq/L  Decrease in PCO2 of 1.2 mmHg 
 
Metabolic alkalosis  Increase in HCO3 of 1 mEq/L Increase in PCO2 of 0.7 mmHg
Respiratory acidosis  Increase in PCO2 of 10 mmHg  Acute: Increase in HCO3 of 1.0 mEq/L 
Chronic: Increase in HCO3 of 3.5 mEq/L 
Respiratory alkalosis  Decrease in PCO2 of 10 mmHg Acute: Decrease in HCO3 of 2 mEq/L
 
Chronic: Decrease in HCO3 of 4‐5 mEq/L 
Renal Regulation of Acid‐  Reabsorption of filtered bicarbonate. Most or all of bicarbonate filtered at the glomerulus is reabsorbed, primarily in the 
Base Balance  proximal tubule.  
o Failure of this reabsorption mechanism leads to proximal renal tubular acidosis. 
 Excretion of acid. The kidney excretes acid in 3 fashions: 
o Free hydrogen ion (H+) excretion (lowers urine pH) – this is quantitatively the least important mechanism 
o Titratable acid excretion (secreted H+ combines with poorly reabsorbable anions such as phosphate) 
o Excretion of ammonium ion (NH4+)  most important 
 The kidney generates ammonia (NH3) which combines with secreted H+ and is excreted as ammonium ion. 
 Failure of H+ secretion and/or ammonia production/ammonium excretion result in distal renal tubular acidosis 
Types of Metabolic Acidosis   Metabolic acidosis can occur due to overproduction of an endogenous weakly dissociable acid (such as lactic acid).  
o Such acids can be generically represented as H+A‐ (for lactic acid, A‐ would be the acid anion lactate).  
o In this case the excess H+ ions titrate extracellular bicarbonate as before, but now there is accumulation of the excess anion 
(A‐) rather than chloride, resulting in a high anion gap acidosis. 
 Metabolic acidosis can also occur due to decreased renal acid excretion leading to retention of acid (H+ ions), which titrate 
extracellular bicarbonate, or it can be caused by loss of bicarbonate from the body.  
o In either instance, the decrease in filtered bicarbonate results in decreased renal bicarbonate and increased renal chloride 
reabsorption (to maintain electroneutrality), resulting in a hyperchloremic acidosis.  
o The plasma chloride rises to the same extent as the plasma bicarbonate falls, and the anion gap remains normal. 
High Anion Gap Acidosis   The anion gap is a concept to give a clue as to the cause of metabolic acidosis (Figure 1). Electroneutrality demands that the 
number of positive charges equals the number of negative charges (ions) in body fluids. Therefore, in plasma: 
Na (142mEq/L) + K (5) + Ca (5) + Mg (3) = Cl (105) + HCO3 (25) + Alb (Albumin) (16) + OA (organic anions) (6) + HPO4 (2) + SO4 (1) 
 Simplifying this equation to include only certain monovalent measured anions and calling the other cations  
o unmeasured cations (UC) = K + Ca + Mg 
o unmeasured anions (UA) = Alb (Albumin) + OA (organic anions) + PO4 + SO4 
Na + UC = Cl + HCO3 + UA 
Anion gap = Na – Cl – HCO3 = (UA – UC) 
Normal AG = 10‐12 mEq/L 
 Overproduction of an endogenous organic acid (H+A‐), where A‐ is the acid anion 
o leads to high anion gap metabolic acidosis (for example with lactic acidosis A‐ = lactate).  
 When lactic acid (produced in Sepsis) is produced, the H+ is titrated by bicarbonate as follows: 
HA + HCO3  H2CO3  H2O + CO2 + A 
 This reaction consumes 1 mol of bicarbonate for each mole of acid produced.  
o If lactate (A) is not excreted or metabolized, it will accumulate in plasma, resulting in an “anion gap”. 
Increased Anion Gap   Increased organic acid production (with retention of organic acids) 
Acidosis  o Ketoacidosis – acetoacetate, beta‐hydroxybutyrate 
o Lactic acidosis – lactate 
o Toxin ingestion 
 Salicylate (Aspirin) – mostly lactate 
 Methanol (wood alcohol) – formate and formaldehyde 
 Ethylene glycol (antifreeze) – glyoxylate, oxalate 
 Failure to excrete inorganic anions: 
o Renal failure leading to increased AG – phosphate, sulfate 
Normal Anion Gap   Gastrointestinal loss of bicarbonate: 
(Hyperchloremia) Acidosis  o diarrhea 
 Renal loss of bicarbonate: 
o Proximal renal tubular acidosis (RTA) (Type 2) 
o Carbonic anhydrase inhibitors (prevent proximal HCO3 reabsorption) 
 Failure to excrete acid: 
o Distal renal tubular acidosis (RTA) (Type 1, Type 4) 
o Renal failure – leads to high and normal anion gap acidosis 
 Administration of acid: 
o Infusion of HCl or its congeners (e.g., ammonium chloride, TPN) 
 Administration of large amounts of saline: 
o dilutional acidosis 
 Type 2 RTA (Proximal)   In normal kidneys, there is complete reabsorption of filtered bicarbonate (by the proximal 
  tubule) until the plasma bicarbonate level exceeds normal (i.e., 25 mEq/L).  
 If there is dysfunction of the proximal tubule, there is a defect in bicarbonate reabsorption 
 Proximal RTA Type 2 
o leading to a decrease in the plasma bicarbonate level to the point that the kidneys are 
again able to reabsorb all of the filtered bicarbonate.  
o Often accompanied by other features of Fanconi Syndrome  an increase excretion of 
nearly all amino acids, glucose, bicarbonate, and phosphate 
 In the example given in the Figure, 
o this is a plasma HCO3 of 15 mEq/L. Since filtered bicarbonate is equal to the plasma 
HCO3 x GFR, in this case, the kidneys ability to reabsorb bicarbonate would be 15 to 25 
OR 60% of normal. 
 Type 1 RTA (Distal)   In normal kidneys, acid (H+) is excreted by the distal tubule alpha‐intercalated cells, 
  predominantly in the form of ammonium ion (NH4+).   
o When H+ is excreted, an equimolar amount of HCO3 is returned to the blood,  
 thus regenerating the bicarbonate that has been titrated by acid produced from protein 
metabolism.  
 Type 1 distal RTA can be due to  
o failure of the Proton ATP pump on the lumen side of the cell 
o failure of the HCO3‐Cl exchanger on the blood side of the cell 
o backleak of protons from the lumen into the cell 
 In addition, an increased secretion of potassium also occurs, resulting in hypokalemic 
hyperchloremia acidosis 
 The most common causes of Type 1 Distal RTA are: 
o Hereditary; Autoimmune disease; Tubulointerstitial disease; Dysproteinemias 
 Urine pH is increased (>5.3) despite systemic acidosis. 
 There is decreased ammonium ion (NH4) resulting in a decreased excretion of chloride ion 
Type 4 RTA (Distal)   Usually due to an Aldosterone deficiency (Hypoaldosteronism) which decreases the activity of the Proton ATPase. 
o The resulting hyperkalemia leads to decreased ammonium ion excretion  
 Due to inhibition of ammonia formation 
 Most common causes are: 
o Diabetes – due to the glycation of prorenin with impaired activation to renin 
o Drugs – RAAS axis inhibitors 
o Aldosterone synthesis inhibitors such as Heparin 
o Inhibitors of tubular potassium secretion such as Trimethoprim and Calcineurin inhibitors 
o NSAIDs inhibit the RAAS axis as well as tubular potassium secretion 
Summary of Major  RTA  Type 1  Type 2 Type 4
Laboratory Findings in RTA  Plasma Potassium  Low  Low High
 
Urine pH  >5.3  <5.3 <5.3
 
Treatment of Metabolic   Treat underlying cause(s)  sepsis, shock 
Acidosis   Bicarbonate (especially with normal anion gap acidosis) – Na or K bicarbonate (or bicarbonate former) depending on etiology 
and electrolyte values. 
 
Metabolic Alkalosis 
 Metabolic alkalosis is defined as a primary increase in the bicarbonate concentration, i.e. [HCO3‐], of the plasma.  
o In primary metabolic alkalosis: 
 the pH of the blood will be elevated (alkalemia) 
 An increase in [HCO3‐] of 1.0 mEq/L should be accompanied by an increase in PCO2 of 0.7 mm Hg (this is termed respiratory adaptation).  
 The rise in PCO2 will keep the blood pH near normal (although mild alkalemia will be present). 
Generation of metabolic   There are three ways to generate metabolic alkalosis: 
alkalosis  o Net loss of hydrogen ions (H+) from extracellular fluid (ECF) 
o Net addition of HCO3‐ to ECF 
o Loss of fluid containing chloride in excess of bicarbonate ("contraction alkalosis") 
Net loss of hydrogen ions  Causes of H+ loss 
(H+) from extracellular fluid   Gastrointestinal ‐ loss of HCl   HCl secretion by parietal cells of stomach does not normally cause metabolic alkalosis 
(ECF)  from stomach (vomiting, gastric  because HCl is titrated by pancreatic sodium bicarbonate: 
drainage)  HCl (stomach) + NaHCO3 (pancreases) ‐‐> NaCl + CO2 + H2O. 
   However, if HCl is lost from the body because of vomiting or gastric drainage, there is a 
net gain of bicarbonate.  
 Metabolic alkalosis is then maintained by  
o increased renal bicarbonate reabsorption and decreased bicarbonate secretion  
 due to depletion of ECF and chloride 
 Renal ‐ loss of H+ into urine   Hydrogen ions generated by protein metabolism are normally excreted by the kidneys 
(mineralocorticoid excess  primarily in the form of ammonium (NH4+) ions.  
states)   In certain conditions, an inappropriate increase in renal H+ excretion leads to metabolic 
  alkalosis.  
o This is characteristic of primary mineralocorticoid excess states.  
 Because, Aldosterone (the naturally occurring mineralocorticoid hormone)  
 increases sodium reabsorption and potassium and H+ secretion in the cortical 
collecting tubule.  
 Sodium retention expands ECF volume and decreases proximal tubule sodium 
reabsorption.  
 Metabolic alkalosis then results from the combination of excess mineralocorticoid 
hormone effect and increased distal delivery of sodium  
 (in the presence of aldosterone, sodium in the lumen will enter the cells of the 
collecting tubule and potassium and H+ will exit the cells into the tubule lumen).  
 Administration of diuretic drugs that impair ion transport in the loop of Henle or distal 
convoluted tubule also results in stimulation of aldosterone secretion (secondary to 
renin release in response to hypovolemia) and increased sodium delivery to the 
collecting tubule. 
 Of note, high levels of aldosterone may not result in metabolic alkalosis in the absence 
of adequate distal sodium delivery.  
o For instance, in congestive heart failure and liver cirrhosis,  
 there is secondary hyperaldosteronism (due to decreased renal perfusion and 
increased renin secretion) 
 however, distal sodium delivery is decreased and metabolic alkalosis does not 
normally occur unless diuretics are administered. 
 Shift into cells ‐ severe K+   With very severe potassium deficiency 
deficiency  o K+ will shift out of cells into the ECF in exchange for H+.  
  o Therefore H+ is "lost" into the cells, causing metabolic alkalosis 
o The increase in intracellular H+ in renal tubular cells results in increased H+ secretion 
and bicarbonate reabsorption 
 
 thus also maintaining metabolic alkalosis 
Net addition of HCO3‐ to  Causes of HCO3‐ gain: 
ECF   Exogenous alkali administration (bicarbonate, lactate, citrate, acetate) 
 This only occurs with either massive administration of alkali (usually iatrogenic) or in the presence of impaired renal function. 
Loss of fluid containing  Causes of Cl‐‐rich fluid loss: 
chloride in excess of   Gastrointestinal ‐ villous   Villous adenomas are tumors that secrete chloride into the stool; K+ depletion also contributes 
bicarbonate ("contraction  adenoma, congenital  to the generation of alkalosis.  
alkalosis")  chloridorrhea.   Congenital chloridorrhea is a rare disorder in which there is a failure of gut reabsorption of 
  chloride secreted by the stomach. 
 Renal ‐ diuretics,   Diuretics are a very common cause of metabolic alkalosis  
Bartter's and Gitelman's  o because impaired chloride reabsorption by the kidney plus stimulation of the renin‐
syndromes.  angiotensin‐aldosterone axis by volume contraction  called secondary hyperaldosteronism 
   Bartter's syndrome is a genetic defect of the loop diuretic‐sensitive Na‐K‐2Cl co‐transporter in 
the loop of Henle ("endogenous loop diuretic").  
 Gitelman's syndrome is a genetic defect of the thiazide‐sensitive Na‐Cl co‐transporter in the 
distal tubule ("endogenous thiazide") 
 During chronic hypercapnia (seen frequently in chronic obstructive pulmonary disease) 
o there is an appropriate adaptive increase in renal H+ secretion and thus bicarbonate 
reabsorption.  
o This is accompanied by loss of chloride in the urine (sodium is reabsorbed preferentially with 
bicarbonate rather than chloride).  
o Rapid restoration of PCO2 to normal with mechanical ventilation is not accompanied by a 
similarly rapid change in bicarbonate handling by the kidney and may result in severe 
alkalemia.  
o Because of previous chloride depletion, post‐hypercapnic metabolic alkalosis is typically 
associated with a low urine chloride concentration and improves with saline administration. 
 Skin ‐ cystic fibrosis   Metabolic alkalosis has been described in children with cystic fibrosis  
 
  o due to loss of chloride in excess of bicarbonate in sweat. 
Maintenance of Metabolic   Under normal physiologic conditions,  
Alkalosis  o bicarbonate is filtered by the glomerulus (the filtered load is the product of the GFR and the plasma bicarbonate 
concentration).  
o Virtually all of the filtered bicarbonate is then reabsorbed, 
primarily at proximal nephron sites.  
o Since normally about 1 mEq/kg of protons are generated by the 
body each day 
 this amount of bicarbonate buffer is titrated in the ECF and 
needs to be regenerated in the distal nephron.  
 Both reabsorption of filtered bicarbonate and regeneration of 
bicarbonate occur by tubular secretion of H+.  
 H+ is formed in the tubular cell from the splitting of H2O into H+ 
and OH‐.  
 After the H+ is secreted into the tubular lumen, OH‐ then 
combines with CO2 to form HCO3‐, which is reabsorbed into the 
peritubular capillary.  
 Reabsorption of filtered bicarbonate occurs by titration of 
filtered bicarbonate by H+ in the proximal tubular lumen with addition of bicarbonate 
formed in the tubular cell into the peritubular capillary.  
 In the distal nephron, luminal bicarbonate is usually absent;  
 therefore H+ secretion results in urinary H+ loss (primarily in the form of ammonium 
ions or NH4+) and thus bicarbonate regeneration. 
 Mechanism of bicarbonate reabsorption and regeneration by tubular cells.  
o The figure depicts the major cellular and luminal events in bicarbonate reabsorption in the 
proximal tubule (top panel) and regeneration in collecting tubules (bottom panel).  
o In the proximal tubule 
 H+ ions are secreted into the lumen by the Na‐H exchanger, whereas HCO3‐ ions are 
returned to the systemic circulation primarily via a Na‐HCO3 co‐transporter.  
 Secreted H+ combines with filtered HCO3‐ to form CO2 and H2O (this reaction is facilitated 
by carbonic anhydrase (CA) in the brush border). 
o In the collecting tubule 
 a Proton‐ATPase pump and a Cl‐‐HCO3 exchanger mediate these processes.  
 H+ secretion will then result in regeneration of bicarbonate.  
 Note: the collecting tubule cell shown is a type A intercalated cell; there also are type B intercalated cells 
 Once metabolic alkalosis has occurred 
o its maintenance must indicate a failure of the kidneys to excrete the excess bicarbonate.  
 This can occur either because of a  
 decreased filtered load of bicarbonate (due to a decrease in GFR) 
 an increase in tubular bicarbonate reabsorption (or decrease in bicarbonate secretion) 
o In the absence of renal failure 
 inability of the kidneys to excrete the excess bicarbonate implies the presence of a factor (or factors) that either  
 increase bicarbonate reabsorption or decrease its secretion 
o such as ECF or chloride depletion, potassium depletion, and hypercapnia.  
 The mechanism(s) by   ECF depletion 
which these factors  o Decrease GFR and Increases proximal tubular Na+ and HCO3‐ reabsorption  
maintain metabolic   since there is hypochloremia, decreased filtration of chloride leads to bicarbonate reabsorption with sodium in an attempt 
alkalosis.  to maintain ECF volume 
  o Stimulates renin secretion leading to secondary hyperaldosteronism  
 increases H+ secretion and HCO3‐ generation in the collecting tubule 
 Chloride depletion (recent data suggest that this is the most important mechanism) 
o Increases distal tubular HCO3‐ reabsorption 
 decreased tubular fluid chloride concentration promotes H+ secretion and chloride‐bicarbonate exchange by type A 
intercalated cells in the distal tubule 
o Decreases distal tubular HCO3‐ secretion  
 decreased tubular fluid chloride concentration inhibits chloride‐bicarbonate exchange by type B intercalated cells in the 
distal tubule 
 Note that type B cells have the opposite configuration to type A cells, i.e. the location of the H+‐ATPase and chloride‐
bicarbonate exchangers are reversed 
o Directly stimulates renin production 
 leading to secondary hyperaldosteronism and increased H+ excretion 
 K+ depletion 
o Decreases intracellular pH and Increases tubular HCO3‐ reabsorption 
 Hypercapnia (increased PCO2) 
o Decreases intracellular pH and Increases tubular HCO3‐ reabsorption 
 Note: K+ depletion and hypercapnia will both lead to intracellular acidosis, which increases H+ secretion (K+ depletion leads to 
shift of K+ out of cells and H+ into cells; CO2 movement into cells results in acidosis because CO2 combines with OH‐ [formed 
from splitting of H2O into H+ and OH‐]). 
Clinical Features of   History 
Metabolic Alkalosis  o vomiting 
o gastric drainage 
o diuretics 
 Symptoms 
o often none 
o sometimes cramps 
 Signs 
o hypertension (in primary mineralocorticoid excess states)  called Kussmaul sign 
 Kussmaul sign is a paradoxical rise in jugular venous pressure (JVP) on inspiration, or a failure in the appropriate fall of the 
JVP with inspiration. It can be seen in some forms of heart disease and is usually indicative of limited right ventricular filling 
due to right heart dysfunction. 
o hypoventilation (usually not evident on physical exam) 
o tetany and/or increased deep tendon reflexes (metabolic alkalosis results in increased negative charges on serum albumin, 
thus increasing binding of calcium to albumin and decreasing the concentration of free or ionized calcium) 
o cardiac arrhythmias (esp. if pH > 7.6) 
Laboratory Findings in   Arterial blood gases:  
Metabolic Alkalosis  o increased pH, [HCO3‐], and PCO2 
   Electrolytes:  
o elevated [HCO3‐]  
o decreased [Cl‐] 
o usually low [K+] 
o slight increase in anion gap (increased negative charges on albumin, increased lactate due to increased intracellular pH) 
 Blood urea nitrogen (BUN):  
o frequently increased (due to volume depletion) 
 Hematocrit:  
o frequently increased (volume depletion) 
 Urine chloride [Cl‐]:  
o very helpful in differential diagnosis (value < 10 mEq/L indicates volume/chloride depletion) 
 Note: Urine [Na+] may not be low despite volume depletion  
o because urinary loss of bicarbonate forces urinary cation (Na+ and K+) excretion. 
Differential Diagnosis of   Chloride‐responsive type (urine Cl‐ low, i.e. < 10 mEq/L): 
Metabolic Alkalosis  o In these conditions, the kidney is avidly reabsorbing chloride because of persistent volume (and chloride) depletion that 
  developed during the generation of metabolic alkalosis. 
 GI ‐ vomiting, gastric drainage, villous adenoma, chloride diarrhea 
 Renal – continued diuretics use after drug cessation 
 Skin ‐ cystic fibrosis 
 Chloride‐resistant type (urine Cl‐ high, i.e., > 20 mEq/L) 
o Renal ‐ Mineralocorticoid excess states ‐‐ can be either primary (low renin) of secondary (high renin) 
Primary  Secondary (Increase levels of Renin  stimulate aldosterone) 
Primary hyperaldosteronism  Renal artery stenosis 
Cushing’s syndrome  Accelerated hypertension 
Licorice ingestion (stimulates hyperaldosteronism)  Renin‐secreting tumor 
Liddle syndrome (stimulates hyperaldosteronism)  Estrogen therapy 
  Bartter’s syndrome 
  Gitelman syndrome 
  Diuretics (during drug administration) 
o Other – Profound potassium depletion 
Treatment of Metabolic   In general, metabolic alkalosis in the presence of ECF excess (as occurs in mineralocorticoid excess states) is mild and does not 
Alkalosis  require treatment.  
 Alkalosis of sufficient severity to require treatment is generally associated with volume depletion and can be corrected with 
sodium chloride administration.  
o Volume expansion will decrease HCO3‐ reabsorption.  
o Administration of potassium is indicated if hypokalemia is present, as entry of K+ into cells in exchange for H+ will buffer 
excess ECF HCO3‐. 
 Some patients, such as those with severe congestive heart failure, may have metabolic alkalosis  
o due to relative plasma volume depletion (due to loop diuretics) but still have increased ECF volume (edema).  
 In this setting, administration of a carbonic anhydrase inhibitor such as acetazolamide may be beneficial.  
 Carbonic anhydrase inhibitors are proximally acting diuretics which result in decreased bicarbonate reabsorption and 
bicarbonaturia. 
 In the presence of life‐threatening alkalosis 
o intravenous HCl or ammonium chloride (providing the patient does not have hepatic or renal failure) can be given.  
 Ammonium chloride administration titrates bicarbonate by the following reaction: 
NH4Cl + NaHCO3  NaCl + NH3 + CO2 + H2O 
 Finally, prevention of metabolic alkalosis in patients undergoing gastric drainage by drugs that inhibit gastric acid secretion (H2 
blockers, omeprazole) is indicated. 
 
 
Primary Disorder  Predicted Compensation 
Metabolic acidosis  An decrease in [HCO3‐] of 1.0 mEq/L  an decrease in PCO2 of 1.2 mm Hg 
Metabolic alkalosis  An increase in [HCO3‐] of 1.0 mEq/L  an increase in PCO2 of 0.7 mm Hg
Respiratory acidosis  Acute  An increase in [HCO3‐] of 0.1 mEq/L  an increase in PCO2 of 1 mm Hg
Chronic  An increase in [HCO3‐] of 0.4 mEq/L  an increase in PCO2 of 1 mm Hg
Respiratory alkalosis  Acute  An decrease in [HCO3‐] of 0.2 mEq/L  an decrease in PCO2 of 1 mm Hg 
Chronic  An decrease in [HCO3‐] of 0.4 mEq/L  an decrease in PCO2 of 1 mm Hg 
 
 
Acid‐Base Problems: 
 
Patient 1. A 70‐year‐old man presents with weakness. Blood chemistries reveal (in mmol/L): sodium 145, potassium 3.0, chloride 
120, total CO2 16; (in mg/dL): urea nitrogen 30, creatinine 1.2. Arterial blood gases (ABG) reveal: pH 7.35, PCO2 30 mmHg, HCO3 16 
mEq/L. 
 
Normal Blood Chemistries Values:  Normal ABG Values: Normal Urine Chemistries:
Sodium = 136‐144 mmol/L  pH = 7.36‐7.46  Sodium = >40 mmol/L 
Potassium = 3.5‐5.3 mmol/L  PCO2 = 32‐46 mmHg  Potassium = >20 mmol/L 
Chloride = 98‐108 mmol/L  PO2 = 74‐108 mmHg  Chloride = >10 mmol/L 
Total CO2 = 23‐27 mmol/L  HCO3 = 21‐29 mmol/L
BUN = 7‐22 mg/dL   
Creatinine = 0.7‐1.4 mg/dL 
Glucose, fasting = 70‐100 mg/dL 
 
Q1A: What is the acid base disturbance? 
A. Metabolic acidosis 
B. Metabolic alkalosis 
C. Mixed metabolic acidosis/metabolic alkalosis 
D. No metabolic disturbance 
 
ABG, blood pH is below normal  mild acidemia. The expected respiratory compensation is a decreased of PCO2. Since the delta 
bicarbonate is 24‐16=8, the delta PCO2 should be 40 ‐ 1.2*8 = 30.4 +/‐ 2, which is within the range of the ABG PCO2. Therefore, this 
is metabolic acidosis with respiratory compensation 
 
Q1B: How would you further characterize it? 
A. High‐anion gap acidosis 
B. Normal anion gap (hyperchloremic) acidosis 
 
Bicarbonate is low and the AG = 145 – 120 – 16 = 9 (normal = 10‐12). Hence, this is a non‐AG or hyperchloremic metabolic acidosis. 
Another way to tell is that the serum chloride is elevated relative to the serum sodium level. The normal ratio of sodium to chloride 
is 138/100 = 1.38, but in this case is 145/120 = 1.21.  
 
Q2: What is the differential diagnosis and most likely cause? 
A. Chronic kidney disease 
B. RTA (Renal Tubular Acidosis) 
C. Diarrhea (due to increased stool bicarbonate loss) 
D. Hydrochloric acid ingestion 
 
 In patients with normal or near‐normal renal function, the cause of hyperchloremic acidosis will usually be RTA or diarrhea.  
o Classic (distal) RTA and proximal RTA as well as diarrhea typically present with hypokalemic hyperchloremic metabolic 
acidosis.  
o A urine pH>5.5 suggests distal RTA, although it can also be elevated with laxative abuse leading to diarrhea, volume 
depletion, and impaired distal sodium delivery.  
o In classic RTA: The urine AG or net charge (Na + K – Cl) is positive due to impairment of ammonium excretion and thus 
decreased urine chloride 
o in diarrhea: The renal proton excretion is enhanced, and thus there is a large amount of ammonium chloride in the urine, 
and the net charge is typically negative. 
o In proximal RTA: the net charge is variable 
 In this cause, the urine net charge is 40 + 5 ‐ 100 = ‐55 (values given in the book for this problem)  therefore, diagnosis is diarrhea 
Patient 2. A 30‐year‐old woman presents with fatigue and weakness. She denies vomiting or diarrhea and takes no medications.  Her 
physical examination is normal except for mild orthostatic hypotension (volume depleted).  Blood chemistries reveal (in mmol/L): 
sodium 138, potassium 2.9, chloride 90, total CO2 40.  Arterial blood gases (ABG) reveal: pH 7.50; PCO2 53 mmHg; HCO3 40 mEq/L.  
The urine pH is 7.5.  Urine chemistries reveal (in mmol/L): sodium 30, potassium 30, chloride 5.   
 
Normal Blood Chemistries Values:  Normal ABG Values: Normal Urine Chemistries:
Sodium = 136‐144 mmol/L  pH = 7.36‐7.46  Sodium = >40 mmol/L 
Potassium = 3.5‐5.3 mmol/L  PCO2 = 32‐46 mmHg  Potassium = >20 mmol/L 
Chloride = 98‐108 mmol/L  PO2 = 74‐108 mmHg  Chloride = >10 mmol/L 
Total CO2 = 23‐27 mmol/L  HCO3 = 21‐29 mmol/L
BUN = 7‐22 mg/dL   
Creatinine = 0.7‐1.4 mg/dL 
Glucose, fasting = 70‐100 mg/dL 
 
Q1:  What is the acid base disorder? 
A.  Metabolic acidosis 
B.  Metabolic alkalosis 
C.  Mixed metabolic acidosis/metabolic alkalosis 
D.  No metabolic disturbance 
 
On the ABG, the pH is high (7.5) which indicates alkalemia, and bicarbonate is high (40), indicating metabolic alkalosis. Since the 
delta bicarbonate is 24‐40=16, the delta PCO2 = 0.7*16 = 11.2 +/‐ 2, where the actual delta PCO2 = 53‐40 = 13. Therefore, this is a 
simple metabolic alkalosis. 
 
On the ABG: 
Step 1. pH = 7.5 which is >7.4 = Alkalemia 
Step 2. CO2 = 53 which is >40 = Metabolic alkalosis 
Step 3. Calculate AG = 138 – 90 – 40 = 8 which is <12 = Non‐Anion Gap = Simple Metabolic alkalosis 
 
Q2: What is the most likely diagnosis? 
A.  Diuretic intake 
B.  Bartter’s syndrome 
C.  Gitelman’s syndrome 
D.  Surreptitious vomiting 
E.  Hypokalemic periodic paralysis 
 
Since we have a Metabolic alkalosis, we need to check the urine Cl = 5 which is <10 so this could be from Diuretics, Dehydration, 
Emesis, or Nasogastric suction. Then we look at the urine K = 30 which is >20 so this is due to vomiting.  
 Diuretic intake would cause an increase in chloride, sodium, and potassium excretion 
 Bartter’s and Gitelman’s Syndrome are tubular reabsorptive disorders (similar to endogenous diuretics) and would also 
cause increased chloride, sodium, and potassium excretion 
 Hypokalemic periodic paralysis is not associated with metabolic alkalosis 
 
 
   
Patient 3. A 60‐year‐old female patient presents to the emergency room (ER) with fever and confusion.  She has known chronic 
obstructive pulmonary disease (COPD) and is on home oxygen.  
Serum chemistries reveal (in mmol/L): sodium = 136; potassium = 3.9; chloride = 101; total CO2 = 5.  
ABG: pH = 6.8; PCO2 = 33 mm Hg; HCO3 = 5 mEq/L. 
 
Q1: What is the acid‐base disturbance? 
A. Metabolic acidosis 
B. Respiratory acidosis 
C. Respiratory alkalosis 
D. Mixed metabolic acidosis/respiratory alkalosis 
E. Mixed metabolic acidosis/respiratory acidosis 
 
On the ABG: 
Step 1. pH = 6.8 which is <7.4 = Acidemia 
Step 2. CO2 = 33 which is <40 = Metabolic acidosis 
Step 3. Calculate AG = 136 – 101 – 5 = 30 which is >12 = Anion Gap Metabolic Acidosis 
Step 4. Is PCO2 appropriate?  
Winter’s = 1.5*HCO3 + 8+/‐2 = 1.5*5 + 8 = 15.5 +/‐ 2.  
Since the given PCO2 = 33 is greater than expected = Respiratory Acidosis. 
Step 5. Is there a HCO3 problem?  
Delta AG = 12 – 30 = 18.  
Add the delta AG to the HCO3 = 5 + 18 = 23 which is less than normal HCO3 = 24. Thus this is a Metabolic Acidosis. 
 
Q2: Why do you think the patient developed it? 
 
This patient had sepsis leading to lactic acidosis.  She was unable to achieve appropriate respiratory compensation due to her 
underlying COPD. Thus, the very profound acidemia should raise the possibility to a combined acidosis, i.e. both metabolic and 
respiratory acidosis. 
 
Patient 4. A 30‐year‐old male patient presents to the emergency room (ER) with confusion.  
Serum chemistries reveal (in mmol/L): sodium = 136; potassium = 3.9; chloride = 101; total CO2 = 15.  
ABG: pH = 7.5; PCO2 = 20 mm Hg; HCO3 = 15 mEq/L. 
 
Q1: What is the acid‐base disturbance? 
A. Metabolic acidosis 
B. Respiratory acidosis 
C. Respiratory alkalosis 
D. Mixed metabolic acidosis/respiratory alkalosis 
E. Mixed metabolic acidosis/respiratory acidosis 
 
On the ABG: 
Step 1. pH = 7.5 which is >7.4 = Alkalemia 
Step 2. CO2 = 20 which is <40 = Respiratory alkalosis 
Step 3. Calculate AG = 136 – 101 – 15 = 20 which is >12 = High Anion Gap Metabolic Acidosis 
Step 4. Is PCO2 appropriate?  
Winter’s = 1.5*HCO3 + 8+/‐2 = 1.5*15 + 8 = 30.5 +/‐ 2.  
Since the given PCO2 = 20 is smaller than expected = Respiratory Alkalosis  
Step 5. Is there a HCO3 problem?  
Delta AG = 12 (normal) – 20 = 8.  
Add the delta AG to determine the expected HCO3 = 15 + 8 = 23.  
The delta HCO3 = 24 – 23 = 1 which is <2  Acute Respiratory Alkalosis.  
Since the given HCO3 (15) < expected HCO3 (23)  coexisting Anion Gap Metabolic Acidosis 
 
Q2: Why do you think the patient developed it? 
 
Salicylate toxicity causes an AG metabolic acidosis and an acute respiratory alkalosis. 
Hematuria and Nephrolithiasis 
 
Hematuria 
Overview   Can be gross (visible) or microscopic (invisible) 
 Can be transient or persistent 
 Can occur from any site along urinary tract 
 Broad differential diagnosis 
o Entirely benign causes 
o Malignant, potentially lethal causes 
Gross Hematuria   From pink to dark maroon +/‐ clots 
 Normally Painless  
 Sequelae 
o Acute urinary retention 
o Anemia (when severe) 
 Associated with a higher risk of urologic cancer 
 
Detection of Hematuria 
Dipstick testing for   A dipstick detects ‘heme’ 
hematuria  o Red blood cells (RBCs) 
o Free hemoglobin 
o Free myoglobin 
o “heme”+ from Semen 
 Beware of:  
o Contamination from menstrual bleeding or GI bleeding 
Detecting Hematuria   Positive Dipstick testing  
o Requires microscopic confirmation to check for presence of RBCs 
o Micrscopic hematuria is defined as > 3 RBCs per hpf 
 Imposters:  
o Positive dipstick for blood, BUT negative microscopy for RBCs 
 Imposters 
o Free hemoglobinuria (hemolysis, sickle cell disease) 
o Myoglobinuria (rhabdomyolysis from crush injury) = positive dipstick with no red blood cells 
o Semen 
 NOT due anticoagulation or over‐anticoagulation 
o Exception: marked overdose of warfarin 
 Search for cause is usually warranted if > 3 RBCs/hpf in the urine  
Microscopic Hematuria:   Review Microscopic Analysis of Urine 
Evaluation  o Glomerular hematuria 
 RBC casts 
 Dysmorphic RBC’s, acanthocytes 
 Proteinuria (ratio > 0.3 or >300mg/day) 
 Elevated renal indices (BUN, Creatinine) 
 Hypertension 
o Non‐glomerular hematuria 
 Isomorphic RBC’s  same size 
 No proteinuria 
 Normal renal indices 
 Glomerular source suspected 
o Refer to Nephrology 
 Non‐glomerular source suspected 
o Refer to Urology 
 
Causes of Microscopic Hematuria 
Glomerular   Causes 
Age < 50 years old  Age > 50 years old 
IgA Nephropathy  IgA Nephropathy 
Thin Basement Membrane Disease (also called  Glomerulonephritis (also called Goodpastures, Churg‐Strauss, ANCA or 
Benign Familial Hematuria)  Wegener’s Vasculitis, Lupus Nephritis) 
Hereditary Nephritis (also called Alport’s   
 
Syndrome) 
   Uncertain Causes 
Age < 50 years old  Age > 50 years old 
Exercise Hematuria  Exercise Hematuria
Benign Hematuria (Unexplained Microscopic  Over‐Anticoagulation (usually due to Warfarin – rare, but if something is to 
Hematuria)  bleed, it will bleed on Warfarin) 
Over‐Anticoagulation (elevated INR usually   
due to Warfarin) 
Factitious Hematuria (usually presents as   
 
Gross Hematuria 
 
Non‐Glomerular   Upper urinary tract Causes 
Age < 50 years old  Age > 50 years old 
Nephrolithiasis  Nephrolithiasis 
Pyelonephritis or UTI  Renal Cell Carcinoma 
Medullary Sponge Kidney Disease due to kidney stones Pyelonephritis 
Renal Tuberculosis (endemic areas, HIV infection) Ureteral Cancer 
Renal Trauma  Papillary Necrosis 
Renal Infarction or AVM  Renal Infarction 
Ureteral Stricture  Ureteral Stricture and Hydronephrosis 
 
Sickle Cell Disease or Trait  Renal Tuberculosis 
   Lower urinary tract Causes 
Age < 50 years old  Age > 50 years old
Cystitis, Prostatitis, Urethritis or UTI  Cystitis, Prostatitis, Urethritis or UTI 
Benign Bladder and Ureteral Polyps and Tumors  Bladder Cancer 
Bladder Cancer  Prostate Cancer 
Prostate Cancer  Benign Bladder and Ureteral Polyps and Tumors 
Urethral and Meatal Strictures   
 
Schistosoma haematobium (North Africa)   
 
Urologic Cancer 
Incidence   Approximately 5% of cases of microscopic hematuria  
o Higher incidence in gross hematuria 
 Risk increases with age 
o Especially ≥ 65 y/o 
Risk factors   Cigarette smoking 
 Occupational exposures  Leather, dye, rubber/tire manufacturing industries 
 Phenacetin use  analgesic 
 Aristolochic acid  herbal supplement agent that causes urologic cancers and kidney failure in Asia and Japan 
Testing to Detect Urologic   Evaluation of the Upper Tract 
Cancers  o CT scan 
 Evaluation of the Lower Tract 
o Cystoscopy 
 Cytologic Studies of Urine 
o First AM void, three consecutive days ideally 
Upper Tract Imaging  CT 
Modalities 

Ultrasound 

X‐ray 

Microscopic Hematuria:   Imaging of Upper Tract Positive 
Evaluation  o Mass or Cyst suspected: Urology referral to rule out and treat cancer 
o Polycystic kidneys: Nephrology referral to follow for progression and development of kidney disease that may need transplant 
and/or dialysis  
 Imaging of Upper Tract Negative 
o About 70% of all cases 
o Further workup depends upon age, risk factors for urologic cancer 
o Exact age cutoff for/against further workup is controversial 
o Cystoscopy and ureteroscopy 
 Age < 40‐50 y/o, no risk factors for urologic cancer 
o Perform cytology of voided urine x 3 
o Workup ends if negative (consider f/u annual UA) 
o Cystoscopy if neoplastic cells suggested on cytology 
 Age > 40‐50 or risk factors for urologic cancer 
o Urine cytology and cystoscopy 
o Workup ends if both negative 
o No f/u studies necessary unless symptoms develop 
 BUT if there is gross hematuria  should always get full workup due to higher risk of urologic cancer 
Gross Hematuria:   Large bore Foley catheterization +/‐ traction 
Additional Management   Bladder irrigation 
 Intravesicle therapy 
 Cystoscopic/surgical treatments 
 
 
Nephrolithias – Kidney Stones 
Epidemiology   Very Common 
o Lifetime risk: 10‐20% (1:5 people) 
o Male more likely to have kidney stones than Females 
 Lifetime prevalence of 12% (male) and 7% (female) 
o More common in the South where the southeast is more common than the northwest 
o Affects Caucasians > Hispanics and Asians > Blacks (very rare) 
o Recurrence is variable: 
 5 ‐10% within one year (rare) 
 35% in five years 
 50% in 10 years 
o Incidence of Stones 
 70‐80% Calcium oxalate (most common) or calcium phosphate 
 10‐15% Uric acid 
 10‐15% Magnesium ammonium phosphate (a.k.a. Struvite) 
 1% Cystine 
 1% Medication crystals (indinavir, triamterene) 
Clinical Presentation   Most patients with moderate/severe colic 
o With or without gross or microscopic hematuria 
 Stone in upper ureter 
o Flank pain, upper anterior abdominal pain 
 Stone in lower ureter 
o Groin pain, ipsilateral testicular/labial pain 
 Stone lodged in the kidney does not hurt 
o Painless hematuria 
o Persistent urinary tract infection (UTI) 
Pathophysiology of stone   Factors leading to supersaturation 
formation  o Increased ions or solutes – calcium 
o Decreased inhibitors – citrate  
o Increased promoters 
o Low urine flow or volume 
Calcium Oxalate Stones   Stone Shape: envelope and dumbbell 
o Most are a one time or rare occurrence (once every 5‐10 years)  
o Most are idiopathic  
o But some can be associated with medical conditions and can be recurrent (every couple of months) 
 Common Causes: 
o Idiopathic familial hypercalciuria 
o Primary hyperparathyroidism (increased PTH)  “Stones, moans, and groans” 
o Sarcoidosis (overproduction of 1,25‐Dihydroxy vitamin D) 
o Hypercalcemia of malignancy 
o Renal Tubular Acidosis (RTA) 
o Vitamin C abuse (ascorbic acid) 
 Excess vitamin C is excreted in the oxalate form, and this may bind calcium in the urine 
leading calcium oxalate crystals  
 People take vitamin C to prevent and treat colds (more than 4‐5 tabs a day daily) AVOID! 
 Other Unique and Rare Causes: 
o Enteric hyperoxaluria: 
 Dietary calcium binds to FFAs in small bowel  
 Unbound oxalate is absorbed in the colon and then filtered into the urine 
 In the urine, oxalate binds to calcium, resulting in oxalate nephrolithiasis 
 Complication of short bowel syndrome 
 Large missing portion of small bowel in cases of (a.k.a. Crohn disease) 
 Fat malabsorption 
o Primary hyperoxaluria: 
 Autosomal recessive enzymatic defects in glyoxylate metabolism that result in enhanced oxalate overproduction  
 PH type 1 
 PH type 2 
 PH type 3 
 Diagnosed in infants and children and can lead to kidney failure, transplantation and/or dialysis 
o Antifreeze poisoning (ethylene glycol): 
 Intentional with suicide attempt 
 Unintentional with experimenting with ‘drugs’ 
 Unintentional ingestion with kids or pets 
 Calcium Oxalate stones and Hypocitraturia 
o Citrate inhibits calcium stone formation by forming a soluble complex with calcium (inhibitor) 
o Metabolic acidosis (diarrhea or RTA) diminishes citrate excretion  
 due to enhanced citrate reabsorption in the proximal tubule (promoter) 
 Hypocitrituria 
o Medullary sponge kidney 
o Distal RTA 
o Idiopathic or high animal protein diet   
 
Calcium Phosphate Stones   Stone Shape: Wedge shaped prisms and needles 
o Not as common as Calcium Oxalate 
o Form in alkaline urine pH 
o Common complication of distal RTA 

Struvite Stones (also called   Stone Shape: Coffin Lid 
Staghorn)   Causes and Mechanisms 
Ammonium, Magnesium,  o Chronic upper urinary tract infection with urease producing bacteria 
Phosphate   Proteus species 
 Haemophilus species 
 Klebsiella species 
 Ureaplasma urealyticum 
 Staphylococcus saprophyticus 
o Hydrolyzes urea to ammonia leads to persistently alkaline urine 
o More common in women, chronic urinary obstruction (neurogenic bladder, e.g.) 
Uric Acid Stones   Stone Shape: pleomorphic, most often appearing as rhombic plates or rosettes 
with either a yellow or reddish‐brown appearance and form ONLY in acidic urine  
o Radiolucent on Xray (can’t see) 
o Visible on CT 
o Rhomboid or rosettes on micro 
 Caused by: 
o Red meat, obesity, soda 
 Metabolic Syndrome and Diabetes mellites 
o Obesity is a global epidemic, prevalence of 30% in the USA 
 Diet is associated with both a more acid urine pH and increases in urinary uric acid excretion 
 Obesity also increases risk of gout 
 Predisposing Risk Factors 
o Hot and arid climate: 
 dehydration, low urine volume and acidic pH 
o Chronic diarrhea: 
 leads to systemic acidosis and low urine pH as well as dehydration and low urine volumes 
o Hyperproducers of uric acid: 
 Conditions with high cell turn over (PV, AML, TLS) 
o Gout can have high elevated uric acid levels  
o Uric acid solubility decreases with acidic pH 
 Mechanisms 
o Acid urine pH + uric acid  
 Uric acid crystals are much less soluble to an acid pH than alkaline pH 
 Uric acid solubility increases >10x when urine pH increases from 5 to 7 
 Most uric acid stone formers have a urine pH of < 6 
Cystine Stones   Stone Shape: Hexagonal  
o Very Rare 
o Radiolucent on Xray (can’t see) 
o Radiopaqye on CT 
 Cystinuria: autosomal recessive condition 
 Proximal tubule dysfunction of reabsorption of Cystine AND 
o Ornithine 
o Lysine 
o Arginine 
 Cystine 
o Only soluble in low concentrations or in alkaline urine 
 Screening: 
o Cyanide‐nitroprusside screen indicates a urinary cystine concentration >75 mg/L (yellow to purple‐red color) 
Evaluation of Stones   History and physical 
o Prior stone history 
o Related medical illnesses 
 IBD (Crohn disease), thyroid/parathyroid disease, sarcoid, Renal Tubular Acidosis (RTA), small bowl resection 
 UTI’s, urinary obstruction, cancer 
o Family history 
o Medications 
o Diet 
o Daily volume intake 
 Imaging 
o CT scan without contrast preferred 
 Requires no contrast to detect stones 
 Displays proximal and distal ureters 
 Can detect small stones 
 Can detect radiolucent stones  uric acid and cystine stones does not show up on x‐ray 
 Can detect renal disorders other than stones 
 Can detect other intra‐abdominal disorders that may be source of symptoms 
o Renal US, IVP less preferred 
 
 Stone Evaluation 
o Retrieving stone for analysis is CRUCIAL 
 Calcium oxalate or calcium phosphate; Uric acid; Struvite; Cystine 
 Blood tests 
o Routine serum chemistries 
 Look for: 
 Serum calcium; Acidosis; Hypokalemia; Uric acid 
o iPTH level if hypercalcemia or hypercalciuria detected/known 
o Vitamin D 25, Vitamin D 1,25 
 Urine Analysis 
o UA look for pH and crystal presence 
o Some experts advocate 24 hour urine collection in ALL patients 
o Others advocate workup in certain settings 
 After 2nd stone event 
 1st stone in patients under 20 y/o 
 Suspected related medical illness 
 Patient willing to make lifestyle changes in response to findings 
 24 Urine Analysis 
o Two consecutive 24 hour urine collections 
 At least six weeks after stone has passed 
o Variables measured 
 Total volume (2 L/day), creatinine, pH 
 Calcium, oxalate, citrate, uric acid 
 Sodium, potassium, phosphorus 
 Certain circumstances: Cystine 
Active Stone Management   Medical therapy reasonable 
o Small stones (< 5mm) – pass on their own 
o Distal ureteral location 
o Adequate pain control with analgesics 
o No associated UTI or localized obstruction 
 Interventional therapy reasonable 
o Larger stones (>6mm), especially proximal 
o Not passing after 4 weeks of medical therapy  Figure 1. Extracorporeal  Figure 2. Ureteroscopy 
o Intractable pain  Shock Wave Lithotripsy 
o Associated UTI or localized obstruction 
 Medical therapy 
o Analgesia (NSAIDS = opiates in RCTs)  
o Increase Oral intake of 2‐3L/day 
o Tamsulosin – alpha antagonist to help the ureter relax 
 Interventional therapy:  
o Extracorporeal shock wave lithotripsy (ESWL)  Figure 1 
o Ureteroscopy  Figure 2 
Future Stone Prevention   For all stones: 
o Increase daily fluid  2‐3L/day, preferably water 
o Double urine output or achieve > 2L/day 
 Specific stone management strategies 
o Calcium stones: 
 Low sodium diet – avoid can food and boxed dinners 
 NORMAL dietary calcium 
 Restriction will enhance oxalate absorption 
 Medication:  
 Thiazide diuretic: Decreases calcium excretion in urine 
 Citrate for calcium oxalate: alkalinizes urine pH 
 Correct low Citrate/Hypocitraturia 
 Citrate increases urinary pH and increases calcium solubility 
o Lemon juice is an effective source of citrate 
o 4 ounces of lemon juice concentrate per day mixed with tap water as lemonade for a total volume of 2 liters 
 
o Uric Acid stones: 
 Insoluble in low urine pH (pH <6.5)  give citrate to raise pH 
 Medications: 
 First line:  
o potassium citrate 
o Alkalinize urine, maintain urine pH > 6.5 
 Second line: Allopurinol, Losartan 
o Useful with hyperuricemia, marked hyperuricosuria 
o Also useful if patient cannot maintain urine pH > 6.5 
 Diet: 
 Lose weight!   Low purine diet thought to be not very palatable 
 
 
 
o Struvite stones 
 Urine pH between 8‐9  normal is 6.5 
 Eradicate infection with antibiotic therapy 
 Complete removal of all stones 
 Including residual fragments 
 Often requires surgical intervention 
 Prevent future UTIs 
Directed Responses of   Hyperoxaluria  to much oxalate 
Future Stone Prevention  o Low oxalate diet 
o Increase calcium intake  
o Medications: 
 calcium carbonate (tums) 
o Restrict vitamin C 
 Facilitates urine oxalate excretion  
 Cystine stone treatment options: 
o Reduce supersaturation: 
 Increase fluid intake to decrease [cystine] 
 Restrict protein and sodium intake to decrease [cystine] 
 Alkalanize urine (increase cystine solubility) 
 Cystine solubility increases by 3X in an alkaline urine, but only if the urine pH is greater than 7 
o Chelating agents containing thiol: 
 Penicillamine – poorly tolerated 
 Tiopronin – really expensive 
 Thiol‐containing drugs have sulfhydryl groups reduce disulfide bond of cystine, producing mixed disulfides with cysteine that 
are more soluble than the homodimer cystine 
Dietary Review   Foods Rich in Calcium 
o Dairy products 
o Yogurt 
o Sardines 
o Dark leafy greens ‐ Spinach, kale 
o Fortified cereals 
o Fortified Orange juice 
o Soybeans 
o Fortified soymilk 
o Enriched breads/grains 
 Food Rich in Oxalate 
o Black tea 
o Chocolate 
o Soymilk 
o Nuts 
o Berries 
o Beans, spinach, okra 
o Beets, rhubarb 
o Carrots 
o Celery, Swiss chard 
o Sweet potatoes 
 Low Sodium Diet 
 
58 yo man with obesity and diabetes who presents to your office with history of multiple stones. He has had five stones passed in the last year and 
three stones a year ago. He strained his urine and brought the stones in for pathology evaluation. In addition, PMH is + for h/o gout. urine and do a 
dipstick which shows a pH of 5.5 (acidic). You also look at the urine under the microscope and see the following crystals:  

 
Which of the following is TRUE? 
A. Xray conducted will likely show stones 
B. 24‐hour urine will likely show elevated calcium 
C. Treatment may include citrate or lemon juice 
D. Recommended daily fluid intake is one liter 
 
55 year old man with history of small bowel resection due to difficult to manage Crohn disease. Since the age of 30 he has been passing stones 
every few months. Which crystals are you more likely to see on microscopy? Answer is A 

 
 
63 year old woman presents to the office after having passed a stone for which she was medically treated in the emergency room. CT scan showed 
staghorn calculi on the left kidney.  

 
Which of the following is TRUE? 
A. Urine pH will be acidic 
B. She likely has chronic UTIs due to Proteus 
C. Micro will show dumbbell crystals 
D. Her stones will not be visible on Xrays 
 
The formation of renal stones is a consequence of all of the following events EXCEPT: 
A. Supersaturation 
B. Nucleation 
C. Imbalance between promoters and inhibitors 
D. Excessive urine flow 
Disorders of Plasma Sodium Concentration 
 
Introduction   Approximately 60% of body weight is salt water.  
 The blood sodium concentration ([Na+]) is approximated by the ratio of total body sodium to total body water, i.e. 
Blood (plasma or serum) [Na+] = Total body sodium/Total body water 
 Most disorders of sodium concentration are primarily due to alterations in water balance  Usually due to total body water 
Definitions   Hyponatremia: 
o defined as a plasma (or serum) sodium concentration [Na+] < 135 mEq/L.  
 Hypernatremia: 
o defined as a plasma sodium concentration [Na+] > 145 mEq/L. 
Physiologic Regulation of   In steady state, water intake equals water output. 
Plasma Sodium   Water excess without a change in total body sodium content leads to Hyponatremia. 
Concentration [Na+]   Water deficit without a change in total body sodium content leads to Hypernatremia. 
 Increase water intake 
o Causes a slight decrease plasma [Na+] 
o Causes a decrease antidiuretic hormone (ADH) release and loss of thirst 
o Causes an increase renal water excretion and decrease water intake 
o Resulting in the normalization of plasma [Na+] 
 Decrease water intake 
o Causes an increase plasma [Na+] 
o Causes an increase ADH and thirst 
o Causes a decrease renal water excretion and an increase water intake 
o Resulting in the normalization of plasma [Na+] 
Regulation of ADH   ADH is arginine vasopressin (AVP) 
o an octapeptide  
o synthesized by the supraoptic and paraventricular nuclei in the Hypothalamus 
o stored and secreted by the posterior pituitary 
 ADH acts on the collecting duct to increase water reabsorption and thus decrease renal water excretion.  
o In the presence of substantial amounts of ADH 
 the urine will be concentrated i.e. urine osmolality (Uosm) > plasma osmolality (Posm). 
 ADH secretion is physiologically regulated by  
o osmotic (most sensitive) and nonosmotic factors  
o A small (1% to 2%) increase in effective osmolality ([Na+ ] + [glucose]) will increase ADH release.  
 Both thirst and ADH are triggered at a plasma osmolality = 280 mmol/kg and will shut off at lower plasma osmolality 
o A large (∼10%) decrease in blood volume or blood pressure will also increase ADH release and can override the effect of 
osmolality  called non‐osmotic stimulation of ADH release 
o ADH release can also be affected by other non‐osmotic stimuli (e.g., drugs, pain, stress). 
 Assuming normal renal response to its effects,  
o when ADH is completely suppressed, urine osmolality is very low (< 150 to ∼50 mmol per kg) 
o when ADH is maximally active, urine osmolality is high (>700 to ∼1,200 mmol per kg). 
 
Hyponatremia 
Types of Hyponatremia   Pseudo‐Hyponatremia: 
o plasma [Na+] is low due to an 
 increase in the protein and/or lipid content of plasma 
 plasma water [Na+] is normal 
 plasma osmolality is thus normal 
 Hypertonic Hyponatremia (plasma osmolality is high)  Serum Osmolality > 295 mOsm/kg 
o Hyperglycemia: 
 Elevated glucose in plasma osmotically draws water from cells  lowering the plasma [Na+].  
 Corrected [Na+] = measured [Na+] + [1.6 × (glucose / 100 − 1)] 
 where glucose concentration is expressed in mg/dL 
 Note that the correction factor is larger (up to 4) when there is extreme Hyperglycemia 
o Exogenous solutes: 
 Hypertonic mannitol used to treat cerebral edema 
 Isotonic Hyponatremia  Serum Osmolality is Normal (280‐295 mOsm/kg) 
o Addition of an isosmotic but non–sodium‐containing fluid to the extracellular space  such as glycine or sorbitol used as 
endoscopic irrigant solutions 
 Hypotonic Hyponatremia (plasma osmolality is low)  Serum Osmolality < 280 mOsm/kg 
o which can be further subdivided into 3 types based on clinical and laboratory assessment of plasma/extracellular volume 
 Euvolemic 
 Hypovolemic 
 Hypervolemic 
o Most Hyponatremia falls into this category. 
 
 
 
 
 
Pathogenesis of Hypotonic   Plasma sodium concentration [Na+] reflects the relationship between total body cations (sodium plus potassium) and total body 
Hyponatremia  water (TBW) 
o [Na+] = alpha (Nae + Ke ) / TBW + beta  
 where  
 Nae = isotopically measured exchangeable sodium 
 Ke = isotopically measured exchangeable potassium. 
 Thus, plasma [Na+] only gives an indication of the relative amount of total body cations and TBW and not the absolute amount 
of either.  
 In the absence of severe potassium depletion: 
o plasma [Na+] is determined by the ratio of [Na+] = TBNa+ / TBW. 
 
 Hyponatremia = decrease plasma sodium concentration = excess of TBW relative to total body sodium (TBNa+) 
 Increase in TBW with normal   This typically occurs when there is an inappropriate increase in ADH secretion,  
TBNa+   syndrome of inappropriate antidiuretic hormone (SIADH) where the kidneys are 
  reabsorbing water 
 For example,  
o if a tumor is secreting ADH 
 ADH secretion is not being regulated by the plasma sodium concentration 
 Then if water is ingested  it will be retained 
 Causing persistent Hyponatremia. 
 Decrease in TBW with a large   This usually occurs when there are sodium and water losses, with replacement of water 
decrease in TBNa+  but not sodium.  
   For example: 
o some patients taking diuretics (which cause renal loss of sodium and water) will ingest 
a large amount of water possibly due to stimulation of thirst by volume depletion 
 volume depletion, if severe, can also (appropriately) cause ADH release  which 
contributes to Hyponatremia. 
 Large increase in TBW with   This may occur in edematous disorders, that is, 
normal or increased TBNa+  o congestive heart failure (CHF) 
  o liver cirrhosis 
o nephrotic syndrome 
o renal failure 
 For example,  
o in severe CHF, there is a decrease in “effective blood volume” or “effective circulatory 
volume” due to impaired cardiac output 
o since there is decreased renal perfusion, the kidney perceives volume depletion and 
retains sodium and water.  
 ADH release is increased due to the decrease in effective blood volume and blood 
pressure which contributes to Hyponatremia. 
 
o Plasma sodium concentration gives no information about the TBNa+ 
 Estimation of TBNa+ requires physical examination.  
 Signs of low TBNa+  
 flat neck veins 
 decreased skin turgor 
 dry mucous membranes 
 absence of edema 
 orthostatic changes in pulse and blood pressure 
 The principal sign of high TBNa+ 
 edema 
Pathophysiology of Hyponatremia 
 Normal kidneys can excrete a large amount of water  
o because of the great ability of the kidneys to form dilute urine 
 In the absence of ADH 
o urine osmolality can be as low as 50 mmol per kg.  
o The daily solute load is generally 600 to 1,200 mmol per day.  
o Even if urine osmole excretion is 600 mmol per day, 12 L of dilute urine can be excreted.  
o Thus sustained Hyponatremia due to fluid ingestion ALONE is very rare, providing solute intake is maintained (kidney’s are not 
able to excrete distilled water) 
Therefore, Hyponatremia   Decreased solute excretion: 
usually indicates impaired  o If osmole (solute) excretion is lower than normal 
renal water excretion due   Hyponatremia may ensue as a result of smaller fluid intakes.  
to the following:   This has been described in beer drinkers, who ingest much fluid but very little solute (i.e., 100 to 200 mmol per day) solute 
(beer drinker’s potomania). 
ADH  Urine Osmolality (mmol/kg)  Urine volume (L/day)  If patient drinks 6 L of beer, will hold onto __ L of fluid 
0  50  4  2 
++  400  0.5   
 
++++  1200  0.125   
 
 
 
   Impaired urinary dilution:  
o Excess ADH production: 
 either appropriate, i.e., in response to a physiologic stimulus, or inappropriate 
o Intrarenal factors (independent of ADH).  
 Normal renal ability to excrete water depends on three factors:  
 Filtration of solute by the glomeruli  
 Delivery of solute to distal (diluting) nephron sites  
 Water impermeability of diluting nephron sites, which occurs providing ADH is absent.  
Cause  Decreased filtration  Increased solute reabsorption in proximal  Decreased solute reabsorption 
of solute  nephron  in distal nephron 
Mechanism  Decreased GFR  Reduced renal blood flow resulting in stimulations  Inhibitors of solute reabsorption 
of proximal solute reabsorption  in distal nephron 
 
Example  Renal failure  Congestive heart failure  Thiazide diuretics 
 
Volume Status   Total body water is divided into intracellular fluid (ICF) and extracellular fluid (ECF) 
o Sodium is present in HIGH concentration in the ECF but only in LOW concentration in the ICF 
o Clinically, it is usually the ECF volume that is detected, and thus when we say “volume”, we generally mean ECF volume, which 
is dependent on Total Body Sodium Content 
 Hypovolemia: low total body sodium (ECF volume) 
o Flat neck veins 
o Decreased skin turgor 
o Orthostatic changes in heart rate and blood pressure 
 Associated with an increase in heart rate and a decrease in blood pressure when standing 
 Hypervolemia: high total body sodium (ECF) volume 
o Edema 
 Euvolemia: no findings of either Hypovolemia or hypervolemia 
 Water depletion alone will only lead to clinically‐evident Hypovolemia if it is very severe  not very common 
 
Etiology of Hypotonic Hyponatremia 
 Hypovolemic Hyponatremia = a decrease in TBNa+:   Renal Na+ losses—Urine Na > 20 mEq/L 
  o Dehydration 
o Diuretics 
o primary adrenal insufficiency (Addison disease) 
o isolated mineralocorticoid deficiency 
o salt‐wasting nephropathies 
 Extrarenal Na+ losses— Urine Na < 10 mEq/L 
o Diarrhea 
o Vomiting 
o excessive sweating 
 Euvolemic Hyponatremia = normal TBNa+:   SIADH—most commonly due to  
  o (1) tumor 
o (2) pulmonary disease 
o (3) central nervous system disease 
o In some cases, the set point for ADH secretion is altered (“reset osmostat”).  
o Medications associated with SIADH or SIADH‐like syndrome include  
 Chlorpropamide 
 carbamazepine  
 cyclophosphamide 
 vinca alkaloids 
 amitriptyline 
 haloperidol 
 selective serotonin reuptake inhibitors (SSRIs) 
 monoamine oxidase (MAO) inhibitors 
 thiazide‐type diuretics 
 Nephrogenic syndrome of inappropriate antidiuresis (NSIAD).  
o Looks similar to SIADH except that ADH levels are zero.  
o ADH receptors are mutated and constitutively activated 
 Severe Hypothyroidism (may be due to decreased cardiac output and GFR) 
 Psychogenic polydipsia (increased water intake) 
 Beer drinker’s potomania (decreased solute intake) 
 Exercise Hyponatremia (increased water intake plus ADH release) 
 Postoperative Hyponatremia (increase Hypotonic fluid administration plus ADH release) 
 Hypervolemic Hyponatremia = increased TBNa+:   Examples include the following edema states: 
  o CHF 
o Liver cirrhosis 
o Nephrotic syndrome 
o Renal failure 
 
 
Laboratory Evaluation of   Plasma osmolality: 
Hyponatremia  o low in Hypotonic Hyponatremia  < 280 mOsm/kg 
 Urine osmolality: 
o urine should be maximally dilute  Uosm ~50 mmol per kg in the presence of Hypoosmolality 
o however, generally urine is inappropriately concentrated 
 except in: 
 psychogenic polydipsia 
 reset osmostat 
 beer drinker’s syndrome 
 Low UNa+ < 10 mmol/L or FENa+ < 1  
o suggests extrarenal loss of Na+ or an edematous disorder  
 in which kidneys are sodium avid, and thus causing edema 
 due to a decrease in effective circulatory volume) 
 “Normal” UNa+ > 20 mmol/L or FENa+ >1  
o suggests renal loss of Na+ or excess ADH in the absence of renal sodium avidity, as in SIADH 
 BUN levels are typically increased in  
o Hypovolemic or Hypervolemic Hyponatremia 
 owing to increased proximal tubule urea reabsorption  
 BUN levels are typically low or normal in  
o Euvolemic Hyponatremia 
 Plasma uric acid levels are typically  
o reduced in SIADH and cerebral salt wasting 
o elevated in Hypovolemic or Hypervolemic Hyponatremia  due to increased proximal tubule urate reabsorption 
 Thyroid‐stimulating hormone (TSH) may indicate Hypothyroidism 
 plasma cortisol levels may indicate Hypoadrenalism 
 Computed tomography (CT) of the head and radiography of the chest can be helpful if SIADH or cerebral salt wasting is 
suspected. 
Treatment of Hyponatremia   Rate of correction is dependent on severity of problem, that is, whether Hyponatremia is deemed to be acute or chronic. 
 Hypovolemic Hyponatremia:   treated with physiologic saline. 
 Hypervolemic Hyponatremia:   Treat with fluid restriction and diuretics (to treat salt overload) 
 Euvolemic Hyponatremia (e.g.,   Mild asymptomatic Hyponatremia (plasma sodium < 136 mmol/L) 
SIADH) is treated as follows:  o should be considered a diagnostic clue but does not mandate treatment. 
 More severe asymptomatic Hyponatremia (plasma sodium < 125 mmol/L)  
o should be treated with water restriction 
 Symptomatic Hyponatremia: 
o Signs and Symptoms:  
 confusion, seizures, coma is considered a medical emergency 
o Initial treatment with Hypertonic (3%) saline is appropriate.  
 However, the magnitude of correction should not exceed 10 mmol/L from 
baseline (e.g., from 100 to 110 mmol/L).  
 The rate of correction should not exceed 2 mmol/L/h  
 In patients in whom Hyponatremia is known to be chronic and who have only 
 
mild symptoms, a rate of correction should not exceed 0.5 mmol/L/h 
 
Hypernatremia 
Etiology of Hypernatremia   Sodium is the primary determinant of plasma osmolality 
 Hypernatremia is thus always a Hypertonic or Hyperosmolar condition 
 To calculate the Plasma osmolality 
o POsm = 2 x Na (mmol/L) + urea nitrogen (mg/dL) / 2.8 + glucose (mg/dL) / 18 
 Hypernatremia occurs ONLY when Hyperosmolality is accompanied by an impaired thirst mechanism or when water ingestion is 
restricted. 
 In response to Hypernatremia 
o water moves out of cells with a resulting decrease in brain volume 
 The brain responds by intracellular uptake of electrolytes, amino acids, and other organic solutes 
 Therefore, rapid hydration can cause cerebral edema. 
Pathogenesis of   Hypernatremia = increased [Na+] = decrease in TBW relative to TBNa+ .  
Hypernatremia   Decrease TBW with normal   This is typical of two clinical conditions:  
TBNa+   o (1) patients in nursing homes with decreased thirst or inability to drink water 
  o (2) diabetes insipidus (DI) where 
 Central DI  ADH release is impaired or absent 
 Nephrogenic DI  the kidney does not respond to ADH 
 Decrease TBW with decreased   This can occur when water losses exceed sodium losses from sweat, GI tract, or 
TBNa+  kidneys. 
 
 Normal TBW with increased   This is usually iatrogenic  
TBNa+  o due to administration of Hypertonic fluids or isotonic fluids in the setting of renal 
  water losses  
 when the concentration of sodium in administered fluid is greater than urinary 
concentration 
 
 
 In the presence of ADH 
o normal kidneys can concentrate urine to a urine osmolality of 1,200 mmol/kg  
 Older patients, urine osmolality generally will increase only to the 700 to 1,000 mmol/kg range 
o Thus, if the daily solute load is 600 mmol/day 
 The urine output can be as low as 500 ml/day (with lesser daily solute loads, daily urine output can be theoretically even 
lower when maximally concentrated).  
 These low urinary volumes will minimize renal water loss if water intake is impaired.  
 However, insensible water loss (primarily via respiration) is about 500 to 700 mL/day.  
 Therefore, even when there is maximum antidiuresis, total cessation of water intake will lead to Hypernatremia over a 
period of hours to a few days  
Etiology of Hypernatremia   Hypernatremia is always associated with Hyperosmolality 
o can be divided into Hypovolemic, euvolemic, and Hypervolemic Hyponatremia depending on the clinical evaluation 
 Hypovolemic Hypernatremia = decrease in TBNa+    Renal Na+ losses: 
  o diuretics (with inadequate water intake) 
o osmotic diuresis (due to Hyperglycemia, mannitol, urea) 
o postobstructive diuresis 
o tubular injury (recovery phase of ATN) 
 Extrarenal Na+ losses: 
o Sweating 
o Diarrhea 
o vomiting (with inadequate water intake) 
 Euvolemic Hypernatremia = decrease in TBH2O   Central Diabetes insipidus: 
  o trauma, idiopathic tumor 
 Nephrogenic Diabetes insipidus: 
o congenital, drugs, Hypercalcemia, tubular disease 
 Decreased water intake (“nursing home syndrome”) 
 Hypervolemic Hypernatremia = increase in TBNa+   Iatrogenic administration of Hypertonic fluid (Hypertonic 
  saline, bicarbonate, etc.) 
 Mineralocorticoid excess states (e.g., 
Hyperaldosteronism)— causes mild Hypernatremia 
 Salt poisoning (and seawater ingestion) 
 Clinical evaluation involves history and physical examination.  
o Urine chemistries (especially osmolality)  
 in central DI 
 urine osmolality < plasma osmolality (usually < 100 mmol per kg) 
 osmotic diuresis 
 urine osmolality > plasma osmolality. 
Treatment of   Hypovolemic Hypernatremia:   treated with Hypotonic fluids. 
Hypernatremia   Euvolemic Hypernatremia   treated with water administration (+ ADH in central DI) 
 Hypervolemic Hypernatremia   If severe, may require both water administration and either 
  diuretics or dialysis to remove the excess sodium. 
 Rate of correction should not exceed 0.5 mmol/L/h 
o Chronic Hypernatremia (i.e., developing over days) 
 A too rapid a reduction in plasma sodium and osmolality may result in shift of water into the brain  causing brain edema.  
o Acute Hypernatremia is very rare, so caution in rate of correction is usually in order.  
o However, if Hypernatremia is known to have occurred over a period of minutes to hours (salt poisoning), more rapid 
correction is indicated.  
 This is because the brain has not had time to accumulate idiogenic osmoles, and therefore there is a little risk of 
development of brain edema with more rapid correction of Hypernatremia 
 
   
Patient 1 
A 60 year old man with known lung cancer presents with fatigue and cough.  On exam his vital signs are normal.  He is not orthostatic.  His jugular 
veins are visible but not distended, skin turgor is normal, and he has no edema.  He is alert and oriented and answers questions appropriately.  
Chest X‐ray shows a R lower lobe infiltrate.  Plasma electrolytes (in mmol/L) are:  Na = 114, K = 4, Cl = 80, CO2 = 24.  The plasma urea nitrogen and 
creatinine are 6 and 0.6 mg/dL, respectively.  The urine osmolality is 500 mmol/L (High)   Hyponatremia with concentrating urine 
Q1. What is the most likely diagnosis?  
A. Beer drinker’s syndrome 
B. Psychogenic polydipsia 
C. Addison’s disease 
D. SIADH  
 
Beer drinker’s syndrome and psychogenic polydipsia are excluded by the presence of 
concentrated urine (Uosm > Posm). Adison’s disease is due to a hypovolemic state. 
SIADH from the lung cancer is the most likely diagnosis. 
 
Q2. How do you treat the hyponatremia? 
A. Isotonic saline 
B. Hypertonic saline 
C. Tolvaptan – blocks ADH 
D. Fluid restriction 
 
Since he is relatively asymptomatic (fatigue is non‐specific and he has normal mental status), fluid restriction is the appropriate initial therapy. 
Isotonic saline will not be effective in a euvolemic patient. There is no indication for hypertonic saline. Tolvaptan would be effective but is 
unnecessary in this setting and is very expensive. 
 
Patient 2 
An 85‐year‐old woman with Alzheimer’s dementia is admitted from a nursing home for obtundation.  On examination she has hypotension (BP 
80/50 mmHg), flat neck veins, clear chest, and no edema.  Chest X‐ray is clear.   
Plasma electrolytes (in mmol/L) are:  Na = 164 (high, Normal = 140), K = 4, Cl = 130, CO2 = 24.   
The plasma urea nitrogen and creatinine are 16 and 1.2 mg/dL, respectively.   
The urine osmolality is 500 mmol/L. 
 
Q1. What is the your diagnosis? 
A. Dehydration  water depletion 
B. Volume depletion  sodium depletion 
C. Both 
D. Neither 
 
This patient has clinical findings of volume depletion (hypotension, flat neck veins = low 
TBNa+) and has hypernatremia. Therefore, both TBNa+ and TBH2O are low, but the decrement 
in TBH2O is greater than the decrement in TBNa+. This is termed hypovolemic hypernatremia. 
She is in a appropriate antidiuretic state (Uosm > Posm) which is helping to prevent further water losses and thus one can exclude diabetes 
insipidus. She has not been adequately hydrated in the nursing home. 
 
Plasma [Na+] = TBNa+ / TBH2O 
 
Q2. How do you initially treat this patient? 
A. Isotonic saline 
B. Hypotonic saline 
C. Dextrose in water 
D. Oral water replacement 
 
Since she is hypotensive, the initial treatment should be isotonic saline, which will restore ECF and plasma volume and raise blood pressure. 
However, after BP is raised, it is essential to change the IV solution to either hypotonic saline or dextrose in water, which will slowly correct the 
hyponatremia. Oral water as initial therapy is a poor choice as she is obtunded and it will not effectively replenish ECF volume. 
 
    
Clinical Vignette – Hyponatremia 
A 45‐year‐old male with Hypertension and Hyperlipidemia presents for evaluation of shortness of breath and cough for the past month. He takes 
hydrochlorothiazide for blood pressure control. He denies chest pain or palpitations. He has what he thinks are migraines twice a week, which 
resolve with ibuprofen. He has smoked ½ ppd for the past 25 years. On a recent visit to the emergency room, he was noted to have Hyponatremia. 
Vitals signs and physical examination are normal. 
 
Blood chemistry:   Lipid profile: 
Sodium, 125 mmol/L  Total cholesterol, 200 mg/dL (5.2 mmol/L) 
Potassium, 4.0 mmol/L  Triglycerides, 450 mg/dL (5.1 mmol/L) 
Chloride, 104 mmol/L   
Total CO2, 24 mmol/L  Chest X‐ray, 2‐cm lung in right upper lobe nodule 
Urea nitrogen, 10 mg/dL (Urea, 3.6 mmol/L)   
Creatinine, 1.2 mg/dL (106 mcmol/L)  MRI Head with and without (1768 mcmol/L) FENa, 2.9% 
Glucose, 100 mg/dL (5.6 mmol/L)  Plasma ADH, undetectable 
Plasma osmolality (measured), 260 mmol/kg  TSH, 2.0 mIU/L (normal) 
Plasma osmolality (calculated), 263 mmol/kg  Fasting AM cortisol, 10 mcg/dL (276 nmol/L) (normal) 
Urine chemistry:   
Osmolality, 900 mmol/kg 
Sodium, 60 mmol/L 
Creatinine, 20 mg/dL (1768 mcmol/L) 
Complete blood count:   
White blood cells, 4,500/mm3 
Hemoglobin, 12 g/dL (120 g/L) 
Hematocrit, 40% 
Platelets, 250,000/mm3 
 
With low serum sodium (hyponatremia) and high serum osmolality  Hypotonic Hyponatremia 
Urine sodium > 20 
 
Q: What is the next step?. 
A. Fluid restriction 
B. Lung nodule biopsy 
C. Stop the ibuprofen 
D. Stop the hydrochlorothiazide 
E. Administer gemfibrozil 
F. Give intravenous isotonic saline 
G. Give intravenous Hypertonic saline 
 
A: first step in the evaluation of Hyponatremia is to obtain a plasma osmolality, urine osmolality, and urine electrolytes. He is clinically euvolemic, 
and plasma Hypoosmolality coupled with high urine sodium and high urine osmolality suggests SIADH. The plasma sodium is not extremely low 
and he is asymptomatic, so emergent intravenous isotonic or Hypertonic saline is not necessary. The FENa may be elevated due to the thiazide 
diuretic. Thiazides have also been implicated in SIADH as well as impaired urinary dilution due to ADH‐independent effects of the drug (impairment 
of urinary dilution). The triglycerides are high but pseudoHyponatremia is not present since plasma osmolality is low. The lung nodule is 
concerning for an ADH‐secreting malignant tumor; however, the plasma ADH is undetectable. Hence, we have an SIADH picture without ADH, 
which is consistent with nephrogenic syndrome of inappropriate antidiuresis (NSIAD). NSIAD is usually caused by a mutated ADH (V2) receptor, 
which is constitutively active (“gain of function mutation”). It is usually diagnosed in infants, but can present for the first time in adulthood. It is 
treated with fluid restriction and, if necessary, oral urea. Incidentally, the patient later underwent a needle biopsy of the lung nodule, which 
revealed a benign hamartoma. 
 
   
Clinical Vignette – Hypernatremia 
 
A 65‐year‐old male with CHF, diabetes mellitus (DM), and a recent subdural hematoma presents for evaluation of new‐onset seizures. Currently, 
he is tired and has a headache. He complains of nausea and has vomited once. His family reports that he drinks a lot of water. Medications include 
furosemide 20 mg daily, lisinopril 10 mg daily, aspirin 81 mg daily, and insulin. He does not smoke or use alcohol or illicit drugs. Vitals are normal 
except for a mild fever of 38C. Physical examination is significant for irritability and restlessness. There is no papilledema, but there is mild nuchal 
rigidity. He does not have peripheral edema, jugular venous distension, or rales on pulmonary auscultation. He weighs 100 kg. 
 
Blood chemistry:  Lipid profile: 
Sodium, 105 mmol/L  Total cholesterol, 200 mg/dL (5.2 mmol/L) 
Potassium, 4.0 mmol/L  Triglycerides, 210 mg/dL (5.1 mmol/L) 
Chloride, 71 mmol/L   
Total CO2, 24 mmol/L  Chest X‐ray, normal 
Urea nitrogen, 10 mg/dL (Urea, 3.6 mmol/L)   
Creatinine, 1.2 mg/dL (106 mcmol/L)  CT of the head, stable subdural hematoma (similar to CT done 2 weeks 
Glucose, 100 mg/dL (5.6 mmol/L)  ago) 
Plasma osmolality, 220 mmol/kg  Lumbar puncture, normal 
TSH, 2.0 mIU/L (normal) 
Fasting AM cortisol, 10 mcg/dL (276 nmol/L) (normal) 
Urine chemistry:   
Osmolality, pending 
Sodium, pending 
Drug screen, negative 
Complete blood count:   
White blood cells, 4,500/mm3 
Hemoglobin, 12 g/dL (120 g/L) 
Hematocrit, 40% 
Platelets, 250,000/mm3 
 
Q: What is the next step in treatment? 
A. Fluid restriction 2 L per day 
B. Stop furosemide and lisinopril 
C. Give isotonic saline 2.5 L over 5 hours 
D. Give 3% Hypertonic saline 1.2 L over 5 hours 
 
 The patient’s primary problem is new‐onset seizures. His history of subdural hematoma prompts us to evaluate him for intracranial 
pathology. His CT of the head is negative for worsening of subdural hematoma or other intracranial lesions.  
 He has mild fever but lumbar puncture findings are normal.  
 There is no evidence of toxic ingestion, and he is not Hypoglycemic or Hypoxic.  
 His mental status changes and seizures are attributed to severe Hyponatremia.  
 He appears euvolemic as he has neither lower extremity edema nor jugular venous distension or pulmonary rales.  
 Even without urine osmolality and urine electrolytes 
o it seems likely that SIADH is the cause of the Hyponatremia.  
o This can be further delineated by the laboratory findings when available (high urine sodium and osmolality are expected).  
 Because he is very symptomatic, raising the plasma sodium to ~115 mmol/L at a rate of 2 mmol/L/h is indicated.  
o The amount of sodium required to bring the plasma sodium from 105 to 115 is calculated to be 600 mmol  
 [(115 − 105) × 100 kg × 0.6] 
o Administration of 1.2 L of 3% Hypertonic saline (which contains ~500 mmol/L of sodium) over 5 hours is reasonable.  
o However, one should monitor plasma sodium every 1 to 2 hours to assure an appropriate rate and magnitude of correction.  
o For instance, if the urine osmolality is unexpectedly low, or the stimulus for ADH release wanes and the urine becomes dilute 
during treatment, spontaneous water excretion may lead to a more rapid increase in plasma sodium level than predicted on the 
basis of the above formula. It is especially important not to overcorrect the plasma sodium due to the risk of osmotic 
demyelination syndrome (ODS). Because >8% to 10% acute increase in brain water will lead to herniation and death, there is a 
limit to the amount of brain edema that can occur. Therefore, it is unnecessary and potentially dangerous to acutely raise the 
plasma sodium level more than 8% to 10% even in severe symptomatic Hyponatremia.  
 In SIADH, isotonic saline is ineffective because administered sodium will be excreted and some of the administered water retained due to 
high ADH levels. 
 
   
Clinical Vignette – Hypernatremia 
 
A 92‐year‐old man with multi‐infarct dementia, coronary artery disease, prostate cancer, benign prostatic Hypertrophy, and DM presents from a 
nursing home with decreased oral intake, decreased urination, and increasing confusion. He has also had generalized weakness, dizziness, and 
palpitations. He now does not recognize his son, who visits every day. Medications include glyburide 10 mg daily, aspirin 81 mg daily, metoprolol 25 
mg bid, lisinopril 10 mg daily, and Tamsulosin (Flomax) 0.4 mg daily. Vitals are as follows: P, 120/min; BP, 90/70 mm Hg; R, 22/min; T, 35oC. 
Physical examination is significant for confusion, irritability, and lack of cooperation. There is decreased turgor of the skin, dry oral mucosae, and 
weakness in all extremities. There are no neurologic deficits. 
 
Blood chemistry:  Urinalysis: 
Sodium, 150 mmol/L  Color, dark yellow 
Potassium, 4.5 mmol/L  pH, 6 
Chloride, 115 mmol/L  Specific gravity, 1.025 
Total CO2, 25 mmol/L  Protein, negative 
Urea nitrogen, 60 mg/dL (Urea, 21 mol/L)  Blood, negative 
Creatinine, 2.0 mg/dL (177 mcmol/L)  Glucose, negative 
Glucose, 110 mg/dL (6.1 mmol/L)  Ketones, 2+ 
Osmolality, 330 mmol/kg  Bilirubin, negative 
Creatine kinase, 100 U/L  Urobilinogen, negative 
TSH, 2.3 mIU/L  Leukocyte esterase, negative 
Nitrite, negative 
WBC, 2/hpf 
RBC, 0/hpf 
Bacteria, rare 
Urine chemistry:  CT of the head—generalized cortical atrophy 
Osmolality, 800 mmol/kg  MRI brain—cortical atrophy with multiple cerebral infarcts and a 
Sodium, 5 mmol/L  Hypothalamic infarct, age undetermined 
Potassium, 20 mmol/L   
Chloride, 10 mmol/L 
Complete blood count:   
White blood cells, 4,000/mm3 
Hemoglobin, 16 g/dL (160 g/L) 
Hematocrit, 50% 
Platelets, 300,000/mm3 
 
Q: Which of the following is least likely to be contributing to Hypernatremia? 
A. Dementia 
B. Altered thirst sensation 
C. Decreased access to water 
D. Central DI 
 
 In any case of altered mental status, acute intracranial abnormalities should be excluded.  
o In the elderly, especially with underlying dementia, Hypoxia, Hypotension, and sepsis should be considered.  
o Electrolyte disorders, especially Hyponatremia and Hypernatremia, need to be excluded, and one should test thyroid, kidney, 
and liver function.  
o Vitamin deficiencies (such as deficiency in vitamin B12, thiamine, or niacin) and psychiatric conditions should be considered.  
 This patient has multi‐infarct dementia and has decreased oral intake at the nursing home.  
o Both altered thirst sensation and decreased access to water are common underlying factors resulting in decreased water intake 
and Hypernatremia in such patients. 
o The physical exam (tachycardia, Hypotension, decreased skin turgor, and dry mucosae) is consistent with volume depletion.  
o Hypernatremia and acute prerenal failure (low urinary sodium, concentrated urine) are the important laboratory findings.  
o Starvation ketosis resulting in ketonuria is also present. 
 This patient thus has evidence of volume depletion (based on physical exam, prerenal failure, and hemoconcentration) as well as 
dehydration (based on Hypernatremia).  
 Both plasma and urine osmolalities (as well as urine specific gravity) are high 
o indicating appropriate ADH release, as Hyperosmolality, Hypovolemia, and Hypotension are all stimuli for the secretion of ADH.  
 ADH causes the insertion of water channels in the collecting duct, which allows for the conservation of water.  
 Though this patient has had a Hypothalamic stroke, raising the possibility of central DI, ADH production is appropriate, as reflected by the 
appropriately high urine specific gravity and osmolality. 
 
   
Clinical Vignette – Hypernatremia 
 
A 51‐year‐old female with dilated cardiomyopathy and chronic obstructive pulmonary disease presents for evaluation of shortness of breath. She 
quit smoking 3 months ago. She has been using her albuterol nebulizer four times a day as opposed to her usual usage of once per day or less. She 
thinks her lower extremity edema is worse. She also reports three pillow orthopnea and daily paroxysmal nocturnal dyspnea. She denies excess 
sodium intake. She denies fever, chills, or dysuria. She does have cough with yellow sputum. Medications include bupropion, prednisone (started 1 
week ago by her primary physician), Combivent (ipatropium/albuterol), Flovent (fluticasone), albuterol via nebulizer prn, furosemide, lisinopril, 
carvedilol, baby aspirin, and simvastatin. Vitals are as follows: P, 90/min; BP, 115/85 mm Hg; R, 20/min; T, 36.5oC. Positive findings on physical 
examination include jugular venous distension, a third heart sound (S3), and bibasilar rales halfway up the lungs. There is also shifting dullness on 
abdominal percussion and 3+ edema in the lower extremities. The neurologic examination is normal. 
 
Blood chemistry:  Complete blood count: 
Sodium, 150 mmol/L  White blood cells, 4,000/mm3 
Potassium, 5.5 mmol/L  Hemoglobin, 12 g/dL (120 g/L) 
Chloride, 115 mmol/L  Hematocrit, 36% 
Total CO2, 25 mmol/L  Platelets, 220,000/mm3 
Urea nitrogen, 30 mg/dL (Urea, 10.7 mmol/L)   
Creatinine, 1.7 mg/dL (150 mcmol/L) 
Glucose, 133 mg/dL (7.4 mmol/L) 
Osmolality, 320 mmol/kg 
Urine chemistry:  Chest X‐ray, enlarged heart, pulmonary edema 
Osmolality, 350 mmol/kg  D‐dimer, 1.0 μg/L 
Sodium, 40 mmol/L  CT scan chest, no pulmonary embolus, pulmonary edema, pleural 
Potassium, 10 mmol/L  effusions 
Chloride, 60 mmol/L   
 
Q: Which of the following is not a potential treatment for the Hypernatremia? 
A. 5% dextrose in water (D5W) 
B. Loop diuretics 
C. Dialysis 
D. Isotonic saline 
 
 The history indicates a dyspneic patient who is not getting relief through nebulizer treatments and who complains of orthopnea and 
paroxysmal nocturnal dyspnea.  
 In addition, the patient has recently started prednisone 
o which probably caused worsening of fluid overload due to renal sodium retention 
 Jugular venous distension, the presence of an S3, diffuse rales on lung auscultation, and lower extremity edema as well as possible ascites 
on abdominal percussion all confirm Hypervolemia.  
 Hypernatremia indicates that total body sodium is in excess of TBW 
o probably due to the effects of prednisone (renal sodium retention) and furosemide (renal water loss in excess of renal sodium 
loss).  
 Plasma osmolality is high; although urine osmolality is higher than plasma osmolality, it should be higher with this degree of 
Hyperosmolar Hypernatremia.  
o This is probably due to diuretics (which limit urinary concentrating ability).  
 There is no evidence for an acute MI or pulmonary embolism.  
 Treatment of Hypervolemic Hypernatremia is often problematic.  
o This patient needs diuretics to treat the pulmonary edema (the cause of the initial presentiation, i.e., dyspnea), but this can 
lead to further worsening of Hypernatremia.  
o Isotonic dextrose in water (D5W) can be used simultaneously to dilute the plasma sodium; however, it should not be given until 
Hypervolemia is improved.  
o If the kidneys fail to diurese, dialysis should be implemented.  
o Isotonic saline is not appropriate for Hypervolemic Hypernatremia. 
Pathophysiology of Hypertension 
 
Blood Pressure   Blood Pressure is the product of Cardiac Output (CO) and Systemic Vascular resistance (SVR) 
 Blood Pressure is generated by cardiac contraction against Vascular resistance;  
 MAP = CO X SVR. 
o MAP (mean arterial pressure) = DBP + (SBP‐DBP)/3 
o CO (Cardiac Output) = Stroke Volume x Heart Rate (aka contractility) 
o SVR (Systemic Vascular Resistance is also known as Total Peripheral Resistance) 
 If one component increases the other must decrease proportionately to maintain normal BP or hypertension will result. 
Blood pressure modulation  Cardiac Output  Peripheral Resistance 
by effects   o Blood Volume  o Hemoral Factors:  o Neural Factors:  o Local Factors: 
on cardiac output and   Sodium   Constrictors:   Constrictors:   Autoregulation 
peripheral resistance   Mineralocorticoids   Angiotensin II   Alpha‐adrenergic   Ionic (pH and hypoxia) 
 Atrial Natriuretic   Catecholamines   Dilators: 
Peptide   Thromboxane   Beta‐adrenergic 
o Cardiac Factors   Leukotrienes   
 Heart rate   Endothelin  
 contractility   Dilators: 
 Prostaglandins 
 Kinins 
 Nitric Oxide 
 
 
Cardiac Output   Stroke volume is affected by pre‐load, after‐load, and contractility 
 The primary determinant of CO in normal individuals is volume status (sodium content) 
 An increase in CO is rarely the cause of persistent hypertension 
Systemic Vascular   SVR is affected by humoral and local factors:  
Resistance  o Humoral factors: 
 Sensors: baroreceptors, JG apparatus atrium 
 Mediators: BP, distal tubule chloride delivery, atrial stretch 
 Balance of vasoconstrictors and vasodilators 
 Angiotensin II and norepinephrine are two of the more important factors.  
o Local factors: 
 The vessel wall, endothelium monolayer, vascular smooth muscle cell 
 Signals‐blood pressure, shear stress, generated by blood flow (viscosity), humoral factors 
 Effectors: 
 Vasoconstrictors: myogenic response, prostaglandins, leukotrienes, endothelin, endothelium‐derived constricting factor 
(EDCF), angiotensin, endothelial cationic channels, oubain‐like factor 
 Vasodilators: endothelium‐derived relating factor (EDRF, nitric oxide) prostaglandins, endothelial ionic channels 
 Local mechanism allows for auto regulation of blood flow and capillary pressure to various organs (brain and kidney most 
prominently) 
Hypothetical scheme for the 
pathogenesis of essential 
(primary) hypertension 

  


Renin Angiotensin System in   Renin angiotensin system has major effect in pathogenesis and maintenance of hypertension via angiotensin II 
Hypertension   Direct vasoconstriction and increased SVR  
 Enhanced Na reabsorption by the proximal tubule 
 Stimulates aldosterone release which increases Na reabsorption by the collecting tubule 
Sympathetic Nervous   Increased adrenergic tone leads to hypertension. 
System  o Where Adrenergic tone increases 
 Vascular tone 
 Na+ retention 
 Cardiac inotropy  
 Blockade of the sympathetic nervous system reduces BP 
Primary or Essential   No known etiology accounts for about 90% of hypertension. 
Hypertension   Occurs in cluster of families but it is not hereditary 
 Polygenic, no single gene identify 
 Monogenic hypertension is defined when a single gene is identified as the cause for hypertension 
 
Guyton Hypothesis and    Fundamental mechanism of long term control of BP is the fluid volume feedback mechanism 
Renal Function Curves   Kidneys regulate arterial pressure by altering renal excretion of Na (and H2O)  called Renal Pressure Natriuresis  
o thereby controlling circulatory volume and cardiac output 

  

  Hypertension causes a shift in the curve to the right 

  salt sensitive and non‐salt sensitive can both occur in essential hypertension 
Renal Pressure Natriuresis   Elevated BP  
o raises sodium excretion  
 is usually accompanied by increased water excretion  called Renal Pressure Diuresis 
 In hypertension 
o sodium excretion (equal to sodium intake) is maintained at higher BP levels than that would normally result in natriuresis and 
diuresis 
 Thus, in hypertensive subjects, pressure and diuresis are reset 
Derived Curves of Renal 
Handling of Salt and Water 
Intake in Normal People and 
in Patients with Essential 
Hypertension 


Renal Pressure Natriuresis   Renal pressure natriuresis is impaired and shifted in all forms of hypertension studied 
 in Hypertension  o It may be due to either intra‐renal or extra‐renal factors yet to be fully described 
 Intra‐renal factors: 
 lead to reduced renal blood flow, decreased glomerular filtration rate or increased tubular reabsorption 
 Extra‐renal factors: 
 include increase sympathetic nervous system activity or increased anti‐natriuretic hormones 
Kidneys in Pathogenesis    No obvious renal defects identified in hypertensive patients so far but almost all experimental hypertension is caused by insult to 
of Essential Hypertension  kidneys altering either renal homodynamic or tubular reabsorption 
 Renal pressure natriuresis mechanism is abnormal in all types of experimental and clinical hypertension 
Kidney Regulation of   Maintenance of volume by regulating sodium excretion 
Normal Blood Pressure   Pressure natriuresis an essential function of kidneys 
 Resetting of pressure natriuresis at high level occurs in essential hypertension 
 Most monogenic hypertension due to increase sodium retention 
Monogenic Hypertension   Glucocorticoid remediable aldosteronism (GRM): 
o also describable as aldosterone synthase hyperactivity 
o is an autosomal dominant disorder in which the increase in aldosterone secretion produced by ACTH is no longer transient. 
o It is a cause of primary hyperaldosteronism 
 Syndrome of apparent mineralocorticoid excess (AME): 
o is an autosomal recessive disorder causing hypertension (high blood pressure) and hypokalemia (abnormally low levels of 
potassium). 
 Mineralocorticoid receptor mutation (increases Na Ca activity): 
o Mutations in the mineralocorticoid receptor gene cause autosomal dominant pseudohypoaldosteronism type I 
 Liddle Syndrome: gain of function ENac 
o is a genetic disorder inherited in an autosomal dominant manner that is characterized by early, and frequently severe, high 
blood pressure associated with low plasma renin activity, metabolic alkalosis, low blood potassium, and normal to low levels 
of aldosterone. 
 T594M mutation African‐American subtype salt sensitive (increases ENac activity) 
o The T594M allele of the epithelial sodium channel‐subunit has been proposed as a gain‐of‐function mutation leading to salt‐
sensitive hypertension in blacks that is particularly responsive to the specific sodium channel antagonist amiloride 
 
 
 
Secondary Hypertension   An explainable cause for hypertension is present 
 May be superimposed on pre‐existing essential hypertension 
 Overall incidence is about 5‐10% although in specific populations it may be >15‐20% 
Primarily Volume Mediated  Primarily Vasoconstrictor Mediated 
 Non‐renal causes   Pheochromocytoma: tumor of adrenal medulla which 
o Primary hyperaldosteronism: A tumor or hyperplasia of  produces excessive amounts of epinephrine and 
adrenal cortex leads to increased production of  norepinephrine. This increased level of catecholamine 
aldosterone. Aldosterone leads to sodium retention and  produces severe hypertension due to intense 
expanded volume leading to severe hypertension. This is  vasoconstriction. 
pure volume expansion. The plasma levels of aldosterone   Unilateral renal artery stenosis 
are increased and that of plasma renin are suppressed.   Hypercalcemia 
o Cushing’s syndrome:   
o Mineralocorticoid producing tumors: 
 Renal causes 
o Renal failure 
 
o Acute glomerulonephritis 
   Represents about 10% of all hypertension  
 Identifiable underlying cause, has specific therapy and potentially curable, major causes: 
o Kidney disease. 
o Renal artery stenosis  
o Hyperaldosteronism 
o Pheochromocytoma 
  Renal   Acute glomerulonephritis 
 Chronic kidney disease 
 Polycystic disease 
 Renal artery stenosis 
 Renal vasculitis 
 Renin‐producing tumors 
Endocrine   Adrenocortical hyperfunction 
 Exogenous hormones 
 Pheochromocytoma 
 Acromegaly 
 Hypothryoidism (myxedema) 
 Hyperthryoidism (thyrotoxicosis) 
 Pregnancy‐induced 
Cardiovascular   Coarctation of the aorta 
 Polyarteritis nodosa 
Neurologic   Psychogenic 
 Increased intracranial pressure 
 Sleep apnea 
 
 Acute stress, including surgery 
Renovascular Hypertension   Defined as resulting from stenosis of unilateral or bilateral renal artery stenosis 
 Stenosis doesn’t necessarily mean hypertension 
 Degree of stenosis is critical 
Renal Artery Stenosis   Clinical presentations 
o Severe and difficult to control hypertension 
o Kidney failure 
o Flash pulmonary edema 
 Two disease processes 
o Atherosclerotic renal artery stenosis 
o Fibromuscular dysplasia 
Goldblatt Model of   A clip is applied to one renal artery in an animal with two functioning kidneys. That results renal artery stenosis on clipped 
Unilateral Renal Artery  artery. 
Stenosis with two kidneys   Results in hypo perfusion in clipped kidney, leads to increase secretion of renin–angiotensin. 
 Increase BP results in pressure natriuresis in contralateral normal kidney, keeping near normal volume. 
 Increase blood pressure maintained by increased TPR and some impaired pressure natriuresis. 
Goldblatt Model II   A clip is applied to one renal artery in an animal with one functioning kidney, a model of renal artery 
stenosis in a single kidney (such as renal artery stenosis in transplanted kidney). 
  Bilateral renal artery stenosis: 
o Similar mechanism of hypertension. 
o Total renal mass is hypo‐perfused 
o Impaired clearance 
o No off‐setting pressure natriuresis (in contralateral kidney, thus absence or marked impairment 
of pressure natriuresis.) 
Two Kidney‐One Clip   BP increased 
(Unilateral Renal Artery   Blood Volume did not change 
Stenosis)   Plasma Renin increased 
 Response to ACE‐I was positive to control the BP 
One Kidney‐One Clip   BP increased  
(Renal Artery Stenosis of a   Blood Volume increased 
Solitary Kidney)   Plasma Renin did not change 
 Response to ACE‐I was negative and would need to add a diuretic 
Two Kidney‐Two Clip   BP increased  
(Bilateral Renal Artery   Blood Volume increased 
Stenosis)   Plasma Renin did not change 
 Response to ACE‐I was negative and would need to add a diuretic 
Renovascular Hypertension:     Unilateral renal artery stenosis with presence of normal contralateral kidney is vasoconstrictive maintained by increased RAS 
Pathogenesis  system 
 Renal artery stenosis in solitary kidney or bilateral renal artery stenosis is volume dependent 
Renal Artery Stenosis   Therapy 
o Medical – antihypertensive +/‐ ACE inhibitors 
o Interventional – renal artery angioplasty +/‐ stenting 
o Surgical – renal artery bypass 
 
Aldosterone and   Aldosterone, a mineralocorticoid, is a sodium retainer 
Hypertension   Excess aldosterone  
o leads to sodium and volume retention  
 resulting in hypertension 
 Primary aldosterone tumor is relatively rare but primary aldosteronism may result from adrenal hyperplasia and may be more 
common 
Aldosterone   Excess aldosterone  
o leads to hypertension 
 Aldosterone results in  
o Na retention and hypervolemia 
 Excess potassium excretion  
o leads of hypokalemia 
 Volume expansion may be clinically absent due to “aldo‐escape” 
Chronic Kidney Disease and   As kidney function falls (decrease in GFR), its ability to excrete sodium and water is impaired 
Hypertension   This leads to excess volume  
o which causes and worsens hypertension 
Impaired Sodium Excretion   BP incidence increases with patients with chronic kidney disease 
 BP is very responsive to manipulations of volume status 
 Seems to be mediated by abnormal vasoregulation and increased SVR 
Kidney Failure   With the loss of kidney function, virtually 100% of patients become hypertensive 
 Chronic kidney disease is the most common form of secondary hypertension 
 Hypertension can be controlled with diuretics and other hypotensive agents and in End Stage Renal Disease (ESRD) by 
hemodialysis and ultrafiltration. 

  
   Effect of Decrease in Body Fluid Volume by Dialysis Therapy Upon BP of a Patient With Chronic Renal Disease 
0
Bodyweight (kg)

-2
Change in

-4
-6
-8
-10
180
Blood Pressure (mmHg)

Mean
160
140
120
Diastolic
100
80 Start Dialysis
Treatment

   5 10 14 18 22 26 30

Hypertensive Renal Injury   Two patterns of Hypertension injury to Kidneys 
o Essential hypertension 
 Benign nephrosclerosis. 
o Malignant hypertension 
 Intimal thickening 
 Proliferative arteritis 
 Fibrinoid necrosis 
o Renal auto regulation. 
Essential Hypertension and   Contracted kidneys in essential hypertension 
Nephrosclerosis   Progressive reduction in size 
 Cortical atrophy and diffuse fibrosis 
 Biopsy shows afferent arteriolar hyaline arteriosclerosis 
 Subintimal hyaline homogenous eosinophilic deposits 
 Ischemic atrophy 
Nephrosclerosis 


“Benign” hypertension –  
hyaline arteriolopathy 

  
Malignant Hypertension   Marked elevation of BP 
 Evidence of widespread acute arteriolar injury 
 Fundoscopic finding of hypertensive neuroretinopathy 
 

  


Autoregulation   Autoregulation of glomerular filtration rate (GFR) and renal blood flow (RBF) is  
o due to pre‐glomerular vasoconstriction that impedes the transmission of elevated systemic arterial pressure to glomerulus 
and peritubular capillaries of the kidneys 
 Autoregulation protects kidneys from hypertensive injury 
o  but may be altered in chronic kidney disease and diabetic kidney disease 
Autoregulation of Renal   Ensures that renal blood flow and GFR remain relatively constant even while systemic arterial pressures fluctuate 
Blood Flow & GFR   Depends primarily on two mechanisms: 
o Myogenic Response 
o Tubuloglomerular Feedback 

o  

o
Myogenic Response   Response to increased afferent arteriolar pressure: 
o stimulates reflexive vasoconstriction  
 by stimulating smooth muscle cell contraction 
o minimizes increase in PGC that would otherwise occur in response to increased systemic arterial pressure 
o prevents damage to glomerular capillaries 
 which already function at hydrostatic pressures that are much greater than those in the systemic capillaries 
 Response to decreased afferent arteriolar pressure: 
o stimulates reflexive vasodilation  
 by stimulating vascular smooth muscle relaxation 
o increases blood flow and filtration pressure in the glomerulus 
 thereby helping maintain GFR 
Renal autoregulation  Autoregulation
prevents transmission of 
systemic blood pressure to 
BP N
glomerular capillaries 

A E
G

Systemic hypertension 
would increase glomerular 
capillary pressure in  BP
absence of autoregulation 

A E
G

Dihydropyridine calcium  DCA
antagonists (DCA) abolish 
renal autoregulation and 
BP
prevent normalization of 
glomerular capillary 
pressure  A E
G

Kidney Autoregulation:   Autoregulation of kidneys modulate blood flow and pressure transmission to glomerular tufts and protects them against injury. 
Therapeutic role   In CKD or diabetes: 
o autoregulation is impaired making kidney more prone to hypertensive injury. 
 Some antihypertensive drugs may abolish autoregulation and may worsen renal injury. 
Treatment of Hypertension:    Lowering BP: 
Renal Considerations  o will minimize damage to kidneys and slow down progression of renal failure  
 In presence of chronic kidney disease 
o sodium and water are retained and volume becomes an important factor 
 Diuretics are necessary in most instances of hypertension with CKD 
 Drugs which preserve renal autoregulation help preserve renal functions 
Summary   Kidneys play a central role in long term regulation of blood pressure and hypertension 
 There is shifting of pressure natriuresis curve to right in essential hypertension or reset at higher BP level 
 Unilateral renal artery stenosis with a normal contralateral kidney is renin dependent and blocking the renin‐angiotensin system 
is an effective treatment 
 In bilateral renal artery stenosis and in stenotic solitary kidney, the hypertension is volume related 
 Interaction of volume and vasoconstriction is responsible for the vast majority of essential hypertension 
 Kidneys are prone to hypertensive injury and keeping blood pressure under control prevents development of renal failure 
 
Urinary Tract Infection 
 
Definition of UTI   Lower UTI 
o Cystitis, urethritis, prostatitis 
o Infection of the lower genitourinary tract characterized by: 
 Dysuria 
 Increased frequency of urination 
 Urinary urgency 
 Suprapubic pain 
 Hematuria (occasionally) 
 Upper UTI 
o Infection of the kidney (pyelonephritis) 
o Characterized by: 
 Fever 
 +/‐ Chills 
 Flank pain/tenderness 
o May or may not be associated with signs and symptoms of lower UTI 
 Uncomplicated UTI 
o Simple cystitis 
o Absence of: 
 Pyelonephritis 
 Pregnancy 
 Presence of urologic abnormalities including stones 
 Presence of indwelling foley catheter, stent, nephrostomy tube or urinary diversion 
 Immunosuppression 
 Bacteremia 
 Complicated UTI 
o Infection in men, pregnancy and children 
o Hospital‐acquired infection 
o Infection in the presence of factors that predispose to persistent or relapsing infection: 
 Calculi or other obstruction 
 Indwelling catheters or other drainage devices  
 Immunosuppression 
 Renal failure 
 Immunosuppression: renal transplantation, chemotherapy, steroids, AIDS 
 Urinary retention from neurological disease 
Pathogenesis of UTI   Route of infection: 
o Ascending 
 GUT flora reach bladder via urethra 
 Perineal/periurethral colonization in females 
 Massaging action of intercourse 
 Catheterization or other instrumentation 
o Hematogenous 
 Common in Staphylococcal bacteremias 
o Lymphatic  
 Direct extension from adjacent organs via lymphatic system: retroperitoneal abscess or severe bowel obstruction 
 Increased pressure in the bladder can cause lymphatic flow to be directed towards the kidneys. 
   Microbial Factors 
o Adherence factors: pili or fimbriae 
o Motility‐ ascend against urine flow 
o Hemolysin‐ induces pores in cell membrane 
o Cytotoxin‐ Cytotoxic necrotizing factor‐1 
o Aerobactin‐ a siderophore that scavenges iron 
o Urease‐ elevated urinary pH by breakdown of urea (Proteus); associated with staghorn calculi 
o Serum resistance 
o Biofilm 
Adhesin  Receptor  Site 
Type 1 FimH (MS) most common  D‐mannose on many epithelial cells  Bladder and lower tract 
in E coli 
PAP – P fimbriae  most common  P blood group antigens – digalacoside on  Renal pelvis and kidney 
in E coli  RBC and Urothelium 
AFA – not fimbria  RBC  Kidney 
SFA – S fimbriae  Human and bovine RBC  Kidney 
 
DR adhesins  DR blood group  Renal pelvis and kidney 
 


   Host Factors 
o Behavior 
 Frequency of sexual intercourse 
 Use of diaphragm/vaginal spermicide 
 Hygiene: direction of wiping  front to back 
o Susceptibility to local colonization 
 Receptors: women who are of P1 blood group have epithelial receptors that mediate attachment of bacteria 
 pH of urine 
o Urinary stasis 
 Diabetes, neurological disorders, spinal cord injury, etc 
o Anatomic/physiologic changes:    
 Urinary obstruction 
 Reflux 
 Pregnancy 
o Instrumentation/Foreign bodies 
 Presence of catheters, stents, nephrostomy tubes, etc 
o Protective properties of urine 
 IgA and IgG 
 Tamm‐Horsfall protein‐ decrease bacterial attachment 
Epidemiology of UTI   Organisms 
 Acute uncomplicated   Complicated or Recurrent 
o E.coli‐ 80%  o E.coli 
o S. saprophyticus  o Proteus 
o Klebsiella  o Providentia 
o Proteus  o Klebsiella 
o Other  o Pseudomonas 
o Serratia 
o Enterococcus 
o Staphylococcus 
 
o Yeast 
   Prevalence‐ Age and Sex Distribution 
o 8.6 million ambulatory visits (84% women) in 2007 
o Self reported annual incidence in women 12% 
o By age 32, one half of all women report at least 1 UTI 
 2‐5% have recurrent UTIs with genetic predisposition 
o Among healthy women with cystitis, 25% recur within six months 
o Acute uncomplicated pyelonephritis less common 
 Estimated one episode per 28 episodes of cystitis 
Age  Prevalence (%) Sex Factors
Neonates  1  M>F GU abnormalities
Children  4‐5  F>>M   
Sexually active adults  10  F  Sexual activity and use of contraception (spermicide) 
 
Elderly  3‐40  M = F  Prostate hypertrophy, bladder prolapse, incontinence 
Clinical Features of UTI   Cystitis 
o Acute onset of dysuria, frequency, and urgency 
o Suprapubic pain and tenderness 
o Typically young sexually active women 
o Hematuria (about half of the time) 
o Pyuria (WBC in urine) 
o Positive culture ‐ may see lower numbers of bacteria 
 Major causes of acute dysuria in women 
o Cystitis – E. coli, S. saprophyticus, Proteus species, Klebsiella species 
o Urethritis – causes STDs: N. gonorrhoeae, C. trachomatis, HSV 
o Vaginitis – Candida, Trichomonas vaginalis, vaginal flora (BV) 
 
 
 
   Pyelonephritis 
o Acute onset of flank pain, fever and frequently lower tract symptoms (dysuria, frequency, urgency) 
o Costovertebral angle tenderness 
o Bacteremia and septic shock may occur 
o Complication ‐ papillary necrosis (diabetes, SS disease, obstruction) 
o Risk of pyelonephritis is greater in pregnancy 
o WBC casts on UA 
 Prostatitis 
o Pain in perineal area, lower back 
 Acute prostatitis ‐ fever, chills, dysuria, urinary retention 
 Boggy, tender prostate 
 Chronic prostatitis ‐ asymptomatic or perineal pain or mild dysuria 
 Urine cultures positive 
Diagnosis of Acute   Women with symptoms of lower UTI, no vaginal discharge, had >90% probability of acute cystitis 
Uncomplicated Cystitis   Do not need to do a UA 
 Do not need a urine culture 
 Can start antibiotics 
   Specimen collection 
o Urine specimen must be collected in a manner that avoids contamination 
 Clean‐voided, midstream urine 
 Catheterization 
 Suprapubic aspiration 
 Microscopic examination 
o Pyuria = more than 10 WBC's/microliter (10,000/ml) of urine OR more than 2 ‐ 5 WBC's/high power field on slide prepared 
from sediment of centrifuged urine 
 Sensitivity 95%; specificity 71% 
o White blood cell casts ‐ indicates pyelonephritis 
o Gram stain of uncentrifuged urine 
 One or more bacteria per oil‐immersion field correlates with >10^5 bacteria/ml of urine 
 Less sensitive, more specific 
 Chemical screening tests 
o Leukocyte esterase ‐ detects pyuria 
o Nitrite ‐ detects action of bacterial nitrate reductase on urinary nitrates 
 GNR: E.coli, Klebsiella, Proteus 
o Comparable to urinalysis but false negative tests occur. 
 Microbiological tests 
o Quantitative urine culture –  
 >10^5 bacteria/ml usually (80% correlation) indicates infection  
 <10^3 bacteria/ml usually indicates contamination 
o Lower numbers, i.e., 10^2 – 10^4/ml, may be significant in young women with cystitis, males, and patients with indwelling 
catheters 
o Blood culture ‐ may be positive in acute pyelonephritis or acute prostatitis 
WBC (Pyuria) 


WBC Cast   Microscopic cylindrical structures 
 Associated with the presence of WBCs and bacteria in the urine sediment  
 Formed in the distal convoluted tubule and collecting ducts of nephrons 
 Indicative of tubulointerstitial disease: pyelonephritis, acute interstitial nephritis (AIN), lupus nephritis, acute 
papillary necrosis 

LUMC: UA with Reflex to   Although pyuria alone without other signs or symptoms of UTI should not be used to 
Urine Culture  confirm a UTI diagnosis, the absence of pyuria strongly suggests the patient does not have 
  a UTI. Therefore, urine cultures will only be processed if the urinalysis shows >6 WBCs/hpf.  
 Exemptions to this protocol include: 
o Neutropenic patients (WBC < 1K/UL) 
o Pregnancy 
o Neonatal status 
o Pediatric patients with known congenital anomalies of the urinary tract 
o Patients scheduled for transurethral resection of the prostate 
o Patients scheduled for urologic procedure for which mucosal bleeding is anticipated 
Pure plate of >100,000 
col/ml 

  
Mixed bacterial growth due 
to contamination 

  
Management of UTI   Uncomplicated Cystitis (IDSA guidelines) 
o First line regimens: 
 Nitrofurantoin: 100 mg PO BID x 5 days 
 Trimethoprim‐sulfamethoxazole (Bactrim): 160/800 mg BID x 3 days (avoid if resistance prevalence is >20% or if used in 
previous 3 months) 
 Fosfomycin: 3 gm x single dose 
o Alternative regimens: 
 Fluoroquinolones (ciprofloxacin, levofloxacin) x 3 days 
 Cautions, black box warning, increasing resistance 
o Oral beta‐lactams: 
 Amoxicillin‐ clavulanate 
 Cephalexin 
 Cefuroxime 
 Cefdinir  3rd gen  
 Duration: 3‐7 days 
 Decreased rates of cure 
 Close follow‐up 
 Pyelonephritis ‐ uncomplicated 
o Fluoroquinoline 
 Ciprofloxacin: 7 days 
 Levofloxacin: 5 days 
 May give parenteral cephalosporin (ceftriaxone) 
o Trimethoprim/sulfamethoxazole – 14 days 
 If organism sensitive 
o Complete 7 days of therapy if patient responded promptly, otherwise, 10‐14 days is recommended when there is delay in 
response or with severe infection 
Asymptomatic Bacteriuria   Positive urine culture without any signs and symptoms of UTI 
o Screening only recommended for: 
 Pregnant women: urine culture at 1st and 3rd trimester 
 Patients undergoing transurethral resection of the prostate 
 Patients undergoing urologic procedures for which mucosal bleeding is anticipated 
Catheter‐Associated UTI   Patients with indwelling urethral, suprapubic, or intermittent catheterization with signs and symptoms compatible with UTI 
(CAUTI)  along with ≥ 10^3 CFU/mL of bacterial sp from urine sample 
 Above in patients whose catheters have been removed within 48 hours 
Best Way to Prevent CAUTI   Best way: 
o Limit unnecessary foley catheterization  if you don’t need it, then remove it 
o Discontinuation of catheterization 
 Methods with no data for recommendations: 
o Antimicrobial coated catheters 
o Prophylaxis with systemic abx 
o Prophylaxis with methenamine salts 
o Prophylaxis with cranberry products 
o Enhanced meatal care 
o Catheter irrigation 
o Antimicrobials in the drainage bag 
 
   
Question # 1 
A 35 year‐old woman comes to your clinic with a 5‐day history of burning on urination. She is married, monogamous.  
 2 days later, she develops a fever of 101.5 associated with chills and right flank pain.  
What is your diagnosis? 
 
 A UTI defined by the location (lower or upper UTI), complicating factors 
 
Question #2 
On further history‐taking, she reports being sexually active with her husband.  
She uses a diaphragm and occasionally a spermicide. 
She admits to being stressed with work lately. She works for a very busy company that she rarely takes a break.  
What are her risk factors for developing UTI? 
 
 Sexually active, spermicide (inhibits lactobacillus), female, stressed 
 
Question #3 
What organism do you suspect is the cause of her UTI? 
A. Klebsiella 
B. Proteus 
C. E.coli 
D. S. saprophyticus 
 
Question #4 
If she saw you 2 days ago before developing fevers, chills and right flank pain, what would have been your next step in 
management? 
A. Advise cranberry juice 
B. Obtain a urine dipstick in your office 
C. Send to the lab for urine culture 
D. Start her on nitrofurantoin 
 
Question #5 
You instruct her to submit a urine specimen to your medical assistant for urinalysis and urine culture. Later that day, you 
review the results of her urinalysis that showed: 
pH of 6 
No proteins 
No glucose 
WBC >180 = pyuria 
Nitrites + 
Leukocyte esterase + 
WBC cast + 
RBC >10 
Which of the above parameters is highly suggestive of pyelonephritis? 
 
 WBC cast + 
 
Question #6 
After submitting her urine specimen, you discuss with her your diagnosis. Because of the fevers, you also ordered for 2 
sets of blood cultures.  
She then asks you, what do I do now, doctor? 
A. Send her to the ED for admission. 
B. Start her on nitrofurantoin. 
C. Start her on ciprofloxacin 
D. Advise cranberry juice while waiting for urine culture.  
 
Question #7 
You send her home on a 7‐day Rx of ciprofloxacin. Her urine culture later grew >100,000 col of E.coli sensitive to 
ciprofloxacin. Her blood cultures were negative. 
At the completion of her therapy, she calls your office and asks you if there is anything else she needs to do. Her 
symptoms have resolved. 
A. Send her to the lab for repeat UA. 
B. Send her to the lab for repeat UA and urine culture. 
C. No follow‐up needed. 
D. Cranberry juice to prevent recurrent UTI 
 

 
 
Question #8 
1 year later, she comes to your office for a wellness check‐up. She reports noticing foul‐smelling urine (depends on food 
you eat and your hydration level) yesterday when she woke up. She denies other symptoms. She requests for a UA and 
urine culture. What will you do? 
A. Obtain UA only to prove she has no UTI. 
B. Obtain UA AND urine culture due to her history of UTI in the past. 
C. Start her on nitrofurantoin. 
D. Explain “gently” that there is no indication for any further testing at this time.  
 
 
 
 
 
 
 
 
Clinical Obstructive Uropathy 
 
Obstructive Uropathy   Location of the pt’s pain corresponds to the location of the kidney stone –  
  o Ureterovesical junction (UVJ) – most common location for obstruction b/c it is the narrowest part of the ureter;  
 Presents with gonadal, testicular, vaginal pain  
o Crossing of iliac artery (mid‐ureter) – 2nd most common location  
o Ureteropelvic junction (UPJ) – 3rd most common location; flank pain in back 
Causes of obstruction:    Intraluminal 
  o Renal calculi – most common cause 
o Blood clots  
o Fungal ball  
o Sloughed renal papillae  
 papilla can become necrotic; slough off & obstruct the ureter 
 disease processes implicated in this/any chronic illness that can causes vascular disease:  
 diabetes 
 sickle cell 
 pyelonephritis  
 phenacetin abuse  
o Urothelial tumors  
 can get transitional cell cancer anywhere in the urinary tract, renal pelvis, ureter, bladder;  
 most common in the bladder  
 tumor emanates into the tubular portion of the ureter causing an obstruction  
 Intramural (intrinsic) 
o UPJ obstruction  
o Structural lesions  
 Primarily seen in pediatric population; normal ureteral anatomy cannot facilitate peristalsis  
 Extramural (anything strangling ureter or urethra that can cause obstruction)  
o Prostatic disease – lower UT obstruction; most common reason  
o Gynecologic, colorectal or retroperitoneal malignancy (lymphoma, breast cancer, cervical cancer) 
 anatomy of ureter can be markedly distorted by retroperitoneal processes (bleed, tumor, or infection) 
o Crossing vessel to the lower pole of the kidney  
 IVC & aorta lie anteriorly to the ureter (posterior) Idiopathic retroperitoneal fibrosis (excessive collagen)   
Pathophysiology   Clinical Definitions 
o Hydronephrosis: 
 extra water in kidney, doesn’t always mean obstruction is present 
 dilatation of the pelvis & calyces  
o Obstructive uropathy:  
 structural impedance to the flow of urine anywhere along the tract  
o Obstructive nephropathy:  
 functional damage &/or anatomic damage to the renal parenchyma 
 results from an obstruction of urine flow 
Renal Functional Changes   Acute obstruction: 
o kidney stones move from pelvis to ureter 
o triphasic changes in:  
 Renal blood flow  
 Ureteral pressure  
 GFR   
 Chronic: 
o might be a malignancy in the retroperitoneum, growing prostate 
Unilateral obstruction:   Phase I: 0‐2 hours 
  o Ureteral pressure increases  
o Renal blood flow increases  
 due to:  
 afferent arteriolar dilation; tubuloglomerular feedback: prostaglandins, iNOS  
 Phase II: 2‐5 hours 
o Ureteral pressures continues to rise due to efferent arteriolar vasoconstriction  
o Renal blood flow starts to decrease  
 afferent arteriolar constriction – angiotensin II via RAAS 
 shift of regional flow from outer to inner cortex  
 Phase III: >5 hours 
o Ureteral pressure declines to baseline by 24h  
 due to decrease in GFR 
 urine production starts to decrease  
o Renal blood flow continues to decrease  
 afferent arteriolar constriction  
 angiotensin II via RAAS 
 endothelin & thromboxane A2  
o This process will chronically manifest as obstructive nephropathy 
 
 
 
Treatment:    Percutaneous extraction techniques  
   Laser treatment  
 Bilateral ureteral stents 
o if there is extramural obstruction or intramural obstruction, this can be used to alleviate obstruction  
 Nephrostomy tubes:  
o allow urine to drain out through the back 
 
Lower Urinary Tract   Primarily a male phenomenon Bulky tissue of the prostate obstructs the urethra 
Obstruction 
   Urethral Stricture Disease (USD)  
o A urethral stricture is scarring in or around the urethra that narrows or blocks the passageway through which urine flows from 
the bladder.  
o The stricture results from inflammation, infection or injury, and is much more common in men than in women. 
o USD: w/ urethritis (from gono, chlam)  
 causes dysuria; infection can manifest 10‐15y later as scar tissue in the urethra & cause obstruction  
o Pts w/ pelvic straddle disease can have USD as well  
o Urethral lumen is narrowed 
o Laser treatment of prostate enlargement  
o Treatment for USD: dilation or reconstructive surgery 
Bilateral Obstruction   Common etiologies:  
o BPH (lower UT)  
o Retroperitoneal disease (upper UT)  
 Distinct/additional/adjunctive pathophysiological effects 
 Tends to present w/ a chronic picture of kidney failure reflected in abnormal blood test levels  
o Elevated Creatine 
 Remember, unilateral obstruction presents w/ symptoms 
   Effect of Obstruction on Urinary Concentrating Ability:  
o Normal concentrating ability requires –  
 Medullary interstitial gradient  
 Water permeability in the collecting duct by ADH  
 Following release of bilateral urinary obstruction (BUO) 
o there is decreased expression of aquaporin channels, which causes polyuria & impaired concentrating ability 
 b/c medullary gradient has been dispersed by a chronic disease process, & now you are not expressing aquaporin channels 
so you can’t absorb water 
Post‐obstructive diuresis:    Marked polyuria can be seen after relief of BUO or obstruction of a solitary kidney  
  o Physiologic – retained urea, sodium, water 
 diuresis ends when fluid homeostasis achieved  
o Pathologic – impairment of concentrating ability or sodium reabsorption  
 down regulation of sodium transporters 
 loss of medullary interstitial solute gradient (solute is lost) 
 poor response to ADH (no receptors or channels for it to act on)  
 Main point: you cannot concentrate your urine 
Review of Obstructive   When obstruction cause functional (acute) or anatomic (chronic) renal damage 
Nephropathy   o Functional: effect is on GFR (understanding triphasic response to the acute urinary obstruction)  
  o Anatomic: Renal damage from chronic obstruction (thinning parenchyma on ultrasound) 
Review of Anatomy   The course of the ureter as it relates to intraluminal obstruction by kidney stones 
 The anterior and posterior relationships of the renal collecting system and vasculature 
 Intramural structure of the ureter 
 LOWER urinary tract obstruction in the male affects both renal units 
Review of Acute versus   Acute:  
Chronic  o Most often unilateral & stones are the most common  
o Common presentation is pain or infection & the creatinine may not be affected b/c the other kidney is normal/compensating  
o Effects on GFR may be reversible 
 Chronic:  
o More likely to be bilateral from  
 prostatic disease (b/c of its loction, affects both kidneys) 
 malignancies causing extramular obstruction (extramural = outside the wall, squeezing it)  
o UPJ obstruction would be an example of unilateral  
o Common presentation is renal failure (elevated creatinine)  
o Post obstructive diuresis can be seen (when obstruction is relieved)  
o Renal anatomic damage is irreversible 
 
Autoimmune 
 
Definition of Autoimmunity   Normal persons are unresponsive (tolerant) to their own (self) antigens. 
 Autoimmunity results from a failure of self‐tolerance.  
 Immune reactions to self antigens (i.e., autoimmunity)  
 Specific immune responses are directed against one particular organ or cell type   localized tissue damage. 
 Can be divided into 2 categories: 
o Organ specific (e.g. Grave’s disease, type l DM…) 
o Systemic (SLE, systemic sclerosis…) 
Immunologic Tolerance   Central tolerance.  
o The principal mechanism of central tolerance is the antigen‐induced deletion 
(death) of self‐reactive T and B lymphocytes during their maturation in central 
(generative) lymphoid organs (i.e., in the thymus for T cells and in the bone marrow 
for B cells).  
 In the thymus, many autologous (self) protein antigens are processed and 
presented by thymic APCs in association with self MHC.  
 Any immature T cell that encounters such a self‐antigen undergoes apoptosis (a 
process called deletion, or negative selection), and the T cells that complete their 
maturation are thereby depleted of self‐reactive cells.  
 An exciting advance has been the identification of putative transcription factors that induce the expression of peripheral 
tissue antigens in the thymus,  
 thus making the thymus an immunologic mirror of self.  
o One such factor is called the autoimmune regulator (AIRE); mutations in the AIRE gene are responsible for an 
autoimmune polyendocrine syndrome in which T cells specific for multiple self‐antigens escape deletion (presumably 
because these self‐antigens are not expressed in the thymus), and attack tissues expressing the self‐antigens. Some T 
cells that encounter self‐antigens in the thymus are not killed but differentiate into regulatory T cells 
o Immature B cells that recognize self‐antigens with high affinity in the bone marrow also may die by apoptosis.  
 Some self‐reactive B cells may not be deleted but may undergo a second round of rearrangement of antigen receptor genes 
and then express new receptors that are no longer self‐reactive (a process called “receptor editing”).  
o Unfortunately, the process of deletion of self‐reactive lymphocytes is not perfect.  
 Many self‐antigens may not be present in the thymus, so T cells bearing receptors for such autoantigens can escape into the 
periphery.  
 There is similar “slippage” in the B cell system as well, and B cells that bear receptors for a variety of self‐antigens, including 
thyroglobulin, collagen, and DNA, can be found in healthy persons. 
 Peripheral tolerance.  
o Self‐reactive T cells that escape negative selection in the thymus can potentially wreak havoc unless they are deleted or 
effectively muzzled.  
o Several mechanisms in the peripheral tissues that silence such potentially autoreactive T cells have been identified: 
 Anergy: This term refers to functional inactivation (rather than death) of lymphocytes induced by encounter with antigens 
under certain conditions. 
 Activation of T cells requires two signals:  
o recognition of peptide antigen in association with self MHC molecules on APCs 
o a set of second costimulatory signals (e.g., through B7 molecules) provided by the APCs 
 If the second costimulatory signals are not delivered, or if an inhibitory receptor on the T cell (rather than the 
costimulatory receptor) is engaged when the cell encounters self‐antigen, the T cell becomes anergic and cannot respond 
to the antigen.  
 Because costimulatory molecules are not strongly expressed on most normal tissues, the encounter between autoreactive 
T cells and self‐antigens in tissues may result in anergy.  
 B cells can also become anergic if they encounter antigen in the absence of specific helper T cells. 
 Suppression by regulatory T cells: The responses of T lymphocytes to self‐antigens may be actively suppressed by regulatory 
T cells.  
 The best‐defined populations of regulatory T cells express CD25, one of the chains of the receptor for IL‐2, and require IL‐2 
for their generation and survival.  
 These cells also express a unique transcription factor called FoxP3.  
o This protein is necessary for the development of regulatory cells, and mutations in the FOXP3 gene are responsible 
for a systemic autoimmune disease called IPEX (immune dysregulation, polyendocrinopathy, enteropathy, X‐linked 
syndrome) 
 which is associated with deficiency of regulatory T cells.  
 Several mechanisms have been proposed to explain how regulatory T cells control immune responses, including 
secretion of immunosuppressive cytokines (e.g., IL‐10, transforming growth factor‐β [TGF‐β]), which can dampen a 
variety of T cell responses, and competitive blocking of B7 molecules on APCs. 
 Activation‐induced cell death: Another mechanism of peripheral tolerance involves apoptosis of mature lymphocytes as a 
result of self‐antigen recognition.  
 One mechanism of apoptosis involves the death receptor Fas (a member of the TNF receptor family), which can be 
engaged by its ligand co‐expressed on the same or neighboring cells.  
 The same pathway is important for the deletion of self‐reactive B cells by Fas ligand expressed on T cells.  
 The importance of this pathway of self‐tolerance is illustrated by the discovery that mutations in the FAS gene are 
responsible for an autoimmune disease called the autoimmune lymphoproliferative syndrome (ALPS), characterized by 
lymphadenopathy and multiple autoantibodies including anti‐DNA.  
 Defects in Fas and Fas ligand are also the cause of similar autoimmune diseases in mice.  
 The mitochondrial pathway of apoptosis, which does not depend on death receptors, may also be involved in the 
elimination of self‐reactive lymphocytes. 
Pathogenesis of   Autoimmunity arises from the inheritance of susceptibility genes that may interfere with self‐tolerance, in 
autoimmunity  association with environmental triggers (infection, tissue injury, inflammation) that alter the display of self‐
antigens, promote lymphocyte entry into tissues, and enhance the activation of self‐reactive lymphocytes. 

Genetic factors in   Autoimmune diseases have a tendency to run in families, and there is a greater incidence of the same disease in monozygotic 
autoimmunity  than in dizygotic twins.  
 Several autoimmune diseases are linked with the HLA locus, especially class II alleles 


Gender Influence   There is a strong gender bias of autoimmunity 
 Much more common in Women than Men 
o If males are affected, the disease is much more severe 
 Not well understood 
 Hormones?  
 Other factors? 
Role of Infections and   Several hypotheses:  
Tissue Injury  o Viruses and other microbes may share cross‐reacting epitopes with self antigens 
o Microbial infections with resultant tissue necrosis and inflammation 
 
Systemic Lupus Erythematosus 
History   The word ‘lupus’ is derived from the Latin word for “wolf”. 
 Named in the 13th century by the physician “Rogerius”. 
 He described the rash as erosive facial lesions as a consequence of a wolf's bite.  
 The facial rash is also called “Butterfly rash” because it resembles a butterfly shape. 
Definition   An inflammatory, multisystem, autoimmune disease of unknown etiology with variable clinical and laboratory manifestations 
and a variable course and prognosis  
 Lupus can be a mild disease, a severe and life‐threatening illness, or anything in between  
 The diversity of clinical symptoms in SLE is great, and all organ systems are vulnerable 
 Characterized by  
o periods of flare (when disease is active) 
o remission (when disease is inactive) 
o can culminate in irreversible end‐organ damage 
 Characterized by a variety of antibodies that are important for diagnosis and responsible for clinical manifestations. 
Epidemiology   Every year there are more than 16,000 new cases of lupus in North America.  
 2–140/100,000 worldwide but as high as 207/100,000  
 Female:male ratio is 9:1 
 15‐45 years old of age of females 
 Affects minorities (African Americans and Hispanics) more commonly and more severely. 
Health disparities in SLE   Young female 
 Specific racial/ethnic minorities  
 Low income  
 Poverty 
Pathogenesis   Failure to maintain self‐tolerance  autoantibody formation  Tissue damage  
 Model for the pathogenesis of systemic lupus erythematosus. 
o Genetic susceptibility and exposure result in failure of self‐tolerance and persistence of nuclear antigens.  
o Autoantibodies serve to internalize nuclear components, which engage TLRs and stimulate IFN production.  
o IFN may stimulate B and T cell responses to the nuclear antigens.  
 IFN, interferon; IgG, immunoglobulin G; MHC, major histocompatibility complex; TLRs, Toll‐like receptors; 
UV, ultraviolet. 
Clinical manifestations   Great masquerader  
 Symptoms vary by organ system affected 
 Can initially present just with fatigue, rashes, joint achiness  
 Lupus intangibles 
o Fatigue 
o Brain fog 
o Achiness  
o Depression  
o Joint pain 
 
 
 
Lupus on the outside   Malar rash 

 Raynaud 

 Discoid rash 

 Sub‐acute cutaneous rash 

 Alopecia 

 Jaccoud deformity 

Lupus on the Inside   Lupus nephritis 

 Pleurisy 

 Pericarditis 

 
 Hemorrhagic stroke 

Libman‐Sacks Endocarditis   Non‐infectious, verrucous thrombi  
 Usually on mitral or aortic valve  
 Can be associated with anti‐phospholipid syndrome  
 Remember: LSE in SLE 
ACR Classification Criteria   Patient must have 4 of the following 11 criteria at any time during the disease history to be diagnosed with SLE 
   Mnemonic: SOAP BRAIN MD 
o Serositis 
o Oral ulcers 
o Arthritis 
o Photosensitivity 
o Blood disorder 
o Renal disorder 
o ANA 
o Immunologic abnormalities: (anti‐Smith antibody, anti‐double stranded DNA, anti‐phospholipid antibodies) 
o Neurologic symptoms  
o Malar Rash 
o Discoid Rash 
Subtypes of Lupus   There are five types: 
o Systemic lupus erythematosus; Discoid lupus; Drug induced lupus; Neonatal lupus; Overlap syndrome (SLE associated with 
another AI disease) 
Diagnosis of SLE    A thorough history and physical exam  Antibodies  Specificity for SLE  Clinical associations 
 Blood work based on pre‐test probability  Anti‐DsDNA  High   Lupus nephritis 
 Correlates with disease activity 
 Anti‐nuclear antibody (ANA): Useful screening test in  Anti‐Smith  High   None 
symptomatic patients; Reported as a titer (i.e. 1/160,  Anti‐SSA  Low   Dry eyes/dry mouth 
1/320, etc.…); Non‐specific but very sensitive (useful   Subacute cutaneous lupus 
 Neonatal lupus 
when negative)   Congenital heart block 
 Other laboratory tests  Anti‐SSB  Low   Dry eyes/dry mouth 
o White cell count   Subacute cutaneous lupus 
 Neonatal lupus 
o Hemoglobin   Congenital heart block 
o Platelet count  Anti‐RNP  Low   Raynaud, myositis, lung disease 
o Creatinine level  Anti‐phospholipid antibodies  Low   Clotting disorder 
o Proteinuria 
o Complement 3 and 4 
o DsDNA 
 
Treatment of SLE   NSAIDs 
 Anti‐malarials  
 Corticosteroids  
 Immunosuppressive medications 
Adjunctive measures   Avoid sun exposure 
 NO smoking 
 Healthy diet 
 Regular exercise 
Mortality in SLE   Cardiovascular disease: major cause of mortality  
 Other factors: 
o High disease severity at diagnosis 
o Younger age at diagnosis 
o Ethnicity: Black, Hispanic, Asian, and Native American populations 
o Male gender 
o Low socioeconomic status 
o Poor patient adherence 
o Poor social support 
o Low education 
 
Sjogren’s syndrome 
Definition   Autoimmune disease characterized by exocrine glandular dysfunction due to lymphocytic infiltration. 
 Female gender 
 Can be primary or secondary to another autoimmune disease (RA, SLE…) 
Clinical manifestations   Dry eyes (xerophthamia)  
 Dry mouth (xerostomia) 
 But also: 
o dry skin, chronic cough, vaginal dryness, numbness in the arms and legs, feeling tired, muscle 
and joint pains, and thyroid problems 

Laboratory abnormalities   Positive Anti‐SSA (anti‐Ro) and/or anti‐SSB (anti‐La) antibody 
 Positive Rheumatoid factor 
 Renal tubular acidosis (RTA) 
Prognosis   Can cause congenital heart block in newborn of mother with Sjogren. Will need a permanent pacemaker placement. 
 Caution: increased risk of lymphoma (40 times increased risk, esp. mucosa‐associated lymphoid tissue (MALT) 
 
Systemic Sclerosis 
Scleroderma, or systemic   Define: is an autoimmune disease characterized by: 
sclerosis (SSc)  o Vasculopathy  
o Fibrosis   
 Significant morbidity and high rates of mortality. 
Clinical manifestations   Raynaud 
 Skin tightening (no wrinkles) 
 Finger tip ulcers 
 Joint contractures 
 Renal hypertension 
 Dyspnea  
 GERD 
 Diarrhea/malabsorption 
 Weight loss 
Definition of Raynaud   Decreased blood flow to the skin secondary to arteriolar vasospasm in reaction to cold or stress (emotions). 
 Color change: 
o White (ischemia)  blue (hypoxia)  red (reperfusion) 
 Two types: 
o Raynaud disease: when primary/idiopathic  
o Raynaud phenomenon: when associated with another autoimmune disease (e.g. Scleroderma, SLE, Mixed connective tissue 
disease …) 
 Treatment: vasodilators such as calcium channel blockers. 
Subtypes of SSc   Diffuse Scleroderma: 
o Extensive skin involvement 
o Interstitial lung disease 
o Anti‐topoisomerase l (Scl70) 
 
 Limited scleroderma: CREST syndrome 
o Limited skin involvement (fingers and face) 
o Calcinosis, Raynaud, Esophageal dysmotility (difficulty in swallowing), Sclerodactyly and Telangiectasia 
o Pulmonary hypertension  
o Anti‐centromere antibody 
 
 Scleroderma renal crisis 
o typically presents with acute kidney injury in combination with  
 worsening symptoms of scleroderma 
 new‐onset hypertension 
o Investigations often reveal a bland urine sediment, and evidence of microangiopathic hemolytic anemia. 
o Early warning signs include: 
 the onset of the diffuse‐skin‐disease subtype of scleroderma within the past 4 years 
 rapidly progressive skin disease 
 presence of RNA‐polymerase antibodies 
o Recent intensification of glucocorticoid therapy (to doses of prednisone >15 mg daily) can also predispose to the onset of 
scleroderma renal crisis. 
Clinical manifestations  Raynaud 

Sclerodactyly 

Calcinosis 

Decreased mouth aperture 

Telangiectasia 

 
 
Scleroderma antibodies  Antibodies  Clinical Associations 
 Anti‐centromere antibody  o CREST syndrome 
o Pulmonary hypertension 
 Anti‐topoisomerase (Scl70)  o Diffuse skin involvement 
o Interstitial lung disease 
 Anti‐RNA‐polymerase 3  o Scleroderma renal crisis 
 
o malignancy 
 
Anti‐phospholipid syndrome 
   Can be primary or secondary (most commonly associated with SLE) 
 Diagnosis is based on a clinical event of thrombosis (arterial or venous) or a pregnancy morbidity along with abnormal laboratory 
test (clotting antibody). 
Anti‐phospholipid   Anticardiolipin antibodies (aCL) 
antibodies   Anti‐beta2‐Glycoprotein I antibodies 
 Lupus anticoagulant (LAC) 
 PS: Can cause false positive VDRL/RPR test.  
        LAC can prolong PTT. 
 
 
 
 
 
 
 
 
 
 
 
 
 
 
 
 
 
 
Clinical case 1 
20 year old African American female presents to the ED with 3 months of fatigue, joint pain (fingers and wrists) and a 
rash (picture) that started after she was vacationing in the Caribbean‘s. On exam: vitals stable. Has mild anemia and 
proteinuria on urine dipstick. 
Which of the following is the most likely diagnosis? 
A. Sun burn 
B. Rheumatoid arthritis 
C. Psoriasis 
D. Systemic lupus erythematous 
E. Sjogren’s disease 
 
Which of the following laboratory screening tests is most appropriate as initial work up? 
A. Chest x‐ray 
B. Lumbar puncture 
C. Antinuclear antibody 
D. Lyme serology  
E. DsDNA 
 
Clinical case 2 
65 year old female with 20 year history of Sjogren’s disease comes for follow up. She noticed recent onset of a lump 
behind her right jaw. Dry eyes and dry mouth are stable 
Exam: decreased salivary pooling and right parotid gland enlargement. 
Labs: Platelets: 80,000 (were 200,000 a year ago) 
 
Which of the following condition is likely to occur? 
A. Sinusitis 
B. Parotid gland lymphoma  
C. Otitis media 
D. Mumps infection 
  
 Clinical case 3 
36 year old female presents with 6 months of skin tightening, GERD, change in the color of her fingers.  
On physical exam, her BP is 190/100, HR 80, RR 16, T 98, sclerodactyly and few telangiectasia on her face. 
What process is most likely to explain the cause of her hypertension? 
A. Atherosclerosis  
B. Hyperaldosteronism 
C. Cocaine use  
D. Licorice use 
E. Scleroderma renal crisis 
 
what antibody has been shown to be associated with this condition? 
A. Anti‐SSA 
B. Anti‐topoisomerase (anti‐Scl70) 
C. Anti‐RNA polymerase III 
D. Anti‐centromere antibody 
E. Anti‐nuclear antibody 
 

Potrebbero piacerti anche